Third Test i had to fucking redo because they changed units Flashcards

1
Q

In assessing the knowledge of a pregestational woman with type 1 diabetes concerning changing insulin needs during pregnancy, the nurse recognizes that further teaching is warranted when the patient states:

a. “I will need to increase my insulin dosage during the first 3 months of pregnancy.”
b. “Insulin dosage will likely need to be increased during the second and third trimesters.”
c. “Episodes of hypoglycemia are more likely to occur during the first 3 months.”
d. “Insulin needs should return to normal within 7 to 10 days after birth if I am bottle-feeding.”

A

ANS: A
Insulin needs are reduced in the first trimester because of increased insulin production by the pancreas and increased peripheral sensitivity to insulin. “Insulin dosage will likely need to be increased during the second and third trimesters,” “Episodes of hypoglycemia are more likely to occur during the first 3 months,” and “Insulin needs should return to normal within 7 to 10 days after birth if I am bottle-feeding” are accurate statements and signify that the woman has understood the teachings regarding control of her diabetes during pregnancy.

How well did you know this?
1
Not at all
2
3
4
5
Perfectly
2
Q

Preconception counseling is critical to the outcome of diabetic pregnancies because poor glycemic control before and during early pregnancy is associated with:

a. frequent episodes of maternal hypoglycemia.
b. congenital anomalies in the fetus.
c. polyhydramnios.
d. hyperemesis gravidarum.

A

ANS: B
Preconception counseling is particularly important because strict metabolic control before conception and in the early weeks of gestation is instrumental in decreasing the risks of congenital anomalies. Frequent episodes of maternal hypoglycemia may occur during the first trimester (not before conception) as a result of hormone changes and the effects on insulin production and usage. Hydramnios occurs about 10 times more often in diabetic pregnancies than in nondiabetic pregnancies. Typically it is seen in the third trimester of pregnancy. Hyperemesis gravidarum may exacerbate hypoglycemic events because the decreased food intake by the mother and glucose transfer to the fetus contributes to hypoglycemia.

How well did you know this?
1
Not at all
2
3
4
5
Perfectly
3
Q

In planning for the care of a 30-year-old woman with pregestational diabetes, the nurse recognizes that the most important factor affecting pregnancy outcome is the:

a. mother’s age.
b. number of years since diabetes was diagnosed.
c. amount of insulin required prenatally.
d. degree of glycemic control during pregnancy.

A

ANS: D

Women with excellent glucose control and no blood vessel disease should have good pregnancy outcomes

How well did you know this?
1
Not at all
2
3
4
5
Perfectly
4
Q

Screening at 24 weeks of gestation reveals that a pregnant woman has gestational diabetes mellitus (GDM). In planning her care, the nurse and the woman mutually agree that an expected outcome is to prevent injury to the fetus as a result of GDM. The nurse identifies that the fetus is at greatest risk for:

a. macrosomia.
b. congenital anomalies of the central nervous system.
c. preterm birth.
d. low birth weight.

A

ANS: A
Poor glycemic control later in pregnancy increases the rate of fetal macrosomia. Poor glycemic control during the preconception time frame and into the early weeks of the pregnancy is associated with congenital anomalies. Preterm labor or birth is more likely to occur with severe diabetes and is the greatest risk in women with pregestational diabetes. Increased weight, or macrosomia, is the greatest risk factor for this woman.

How well did you know this?
1
Not at all
2
3
4
5
Perfectly
5
Q

A 26-year-old primigravida has come to the clinic for her regular prenatal visit at 12 weeks. She appears thin and somewhat nervous. She reports that she eats a well-balanced diet, although her weight is 5 lbs less than it was at her last visit. The results of laboratory studies confirm that she has a hyperthyroid condition. Based on the available data, the nurse formulates a plan of care. What nursing diagnosis is most appropriate for the woman at this time?

a. Deficient fluid volume
b. Imbalanced nutrition: less than body requirements
c. Imbalanced nutrition: more than body requirements
d. Disturbed sleep pattern

A

ANS: B
This patient’s clinical cues include weight loss, which would support the nursing diagnosis of Imbalanced nutrition: less than body requirements. No clinical signs or symptoms support the nursing diagnosis of Deficient fluid volume. This patient reports weight loss, not weight gain. Imbalanced nutrition: more than body requirements is not an appropriate nursing diagnosis. Although the patient reports nervousness based on the patient’s other clinical symptoms the most appropriate nursing diagnosis would be Imbalanced nutrition: less than body requirements.

How well did you know this?
1
Not at all
2
3
4
5
Perfectly
6
Q

Maternal phenylketonuria (PKU) is an important health concern during pregnancy because:

a. it is a recognized cause of preterm labor.
b. the fetus may develop neurologic problems.
c. a pregnant woman is more likely to die without dietary control.
d. women with PKU are usually retarded and should not reproduce.

A

ANS: B
Children born to women with untreated PKU are more likely to be born with mental retardation, microcephaly, congenital heart disease, and low birth weight. Maternal PKU has no effect on labor. Women without dietary control of PKU are more likely to miscarry or bear a child with congenital anomalies. Screening for undiagnosed maternal PKU at the first prenatal visit may be warranted, especially in individuals with a family history of the disorder, with low intelligence of uncertain etiology, or who have given birth to microcephalic infants.

How well did you know this?
1
Not at all
2
3
4
5
Perfectly
7
Q

In terms of the incidence and classification of diabetes, maternity nurses should know that:

a. type 1 diabetes is most common.
b. type 2 diabetes often goes undiagnosed.
c. gestational diabetes mellitus (GDM) means that the woman will be receiving insulin treatment until 6 weeks after birth.
d. type 1 diabetes may become type 2 during pregnancy.

A

ANS: B
Type 2 diabetes often goes undiagnosed because hyperglycemia develops gradually and often is not severe. Type 2 diabetes, sometimes called adult onset diabetes, is the most common. GDM refers to any degree of glucose intolerance first recognized during pregnancy. Insulin may or may not be needed. People do not go back and forth between type 1 and 2 diabetes.

How well did you know this?
1
Not at all
2
3
4
5
Perfectly
8
Q

Metabolic changes throughout pregnancy that affect glucose and insulin in the mother and the fetus are complicated but important to understand. Nurses should understand that:

a. insulin crosses the placenta to the fetus only in the first trimester, after which the fetus secretes its own.
b. women with insulin-dependent diabetes are prone to hyperglycemia during the first trimester because they are consuming more sugar.
c. during the second and third trimesters, pregnancy exerts a diabetogenic effect that ensures an abundant supply of glucose for the fetus.
d. maternal insulin requirements steadily decline during pregnancy.

A

ANS: C
Pregnant women develop increased insulin resistance during the second and third trimesters. Insulin never crosses the placenta; the fetus starts making its own insulin around the 10th week. As a result of normal metabolic changes during pregnancy, insulin-dependent women are prone to hypoglycemia (low levels). Maternal insulin requirements may double or quadruple by the end of pregnancy.

How well did you know this?
1
Not at all
2
3
4
5
Perfectly
9
Q

With regard to the association of maternal diabetes and other risk situations affecting mother and fetus, nurses should be aware that:

a. Diabetic ketoacidosis (DKA) can lead to fetal death at any time during pregnancy.
b. Hydramnios occurs approximately twice as often in diabetic pregnancies.
c. Infections occur about as often and are considered about as serious in diabetic and nondiabetic pregnancies.
d. Even mild to moderate hypoglycemic episodes can have significant effects on fetal well-being.

A

ANS: A
Prompt treatment of DKA is necessary to save the fetus and the mother. Hydramnios occurs 10 times more often in diabetic pregnancies. Infections are more common and more serious in pregnant women with diabetes. Mild-to-moderate hypoglycemic episodes do not appear to have significant effects on fetal well-being.

How well did you know this?
1
Not at all
2
3
4
5
Perfectly
10
Q

The nurse providing care for a woman with gestational diabetes understands that a laboratory test for glycosylated hemoglobin Alc:

a. is now done for all pregnant women, not just those with or likely to have diabetes.
b. is a snapshot of glucose control at the moment.
c. would be considered evidence of good diabetes control with a result of 5% to 6%.
d. is done on the patient’s urine, not her blood.

A

ANS: C
A score of 5% to 6% indicates good control. This is an extra test for diabetic women, not one done for all pregnant women. This test defines glycemic control over the previous 4 to 6 weeks. Glycosylated hemoglobin level tests are done on the blood.

How well did you know this?
1
Not at all
2
3
4
5
Perfectly
11
Q

A woman with gestational diabetes has had little or no experience reading and interpreting glucose levels. She shows the nurse her readings for the past few days. Which one should the nurse tell her indicates a need for adjustment (insulin or sugar)?

a. 75 mg/dL before lunch. This is low; better eat now.
b. 115 mg/dL 1 hour after lunch. This is a little high; maybe eat a little less next time.
c. 115 mg/dL 2 hours after lunch; This is too high; it is time for insulin.
d. 60 mg/dL just after waking up from a nap. This is too low; maybe eat a snack before going to sleep.

A

ANS: D
60 mg/dL after waking from a nap is too low. During hours of sleep glucose levels should not be less than 70 mg/dL. Snacks before sleeping can be helpful. The premeal acceptable range is 65 to 95 mg/dL. The readings 1 hour after a meal should be less than 140 mg/dL. Two hours after eating, the readings should be less than 120 mg/dL.

How well did you know this?
1
Not at all
2
3
4
5
Perfectly
12
Q

A new mother with which of these thyroid disorders would be strongly discouraged from breastfeeding?

a. Hyperthyroidism
b. Phenylketonuria (PKU)
c. Hypothyroidism
d. Thyroid storm

A

ANS: B
PKU is a cause of mental retardation in infants; mothers with PKU pass on phenylalanine. A woman with hyperthyroidism or hypothyroidism would have no particular reason not to breastfeed. A thyroid storm is a complication of hyperthyroidism

How well did you know this?
1
Not at all
2
3
4
5
Perfectly
13
Q

When caring for a pregnant woman with cardiac problems, the nurse must be alert for signs and symptoms of cardiac decompensation, which include:

a. a regular heart rate and hypertension.
b. an increased urinary output, tachycardia, and dry cough.
c. shortness of breath, bradycardia, and hypertension.
d. dyspnea; crackles; and an irregular, weak pulse.

A

ANS: D
Signs of cardiac decompensation include dyspnea; crackles; an irregular, weak, rapid pulse; rapid respirations; a moist, frequent cough; generalized edema; increasing fatigue; and cyanosis of the lips and nail beds. A regular heart rate and hypertension are not generally associated with cardiac decompensation. Tachycardia would indicate cardiac decompensation, but increased urinary output and a dry cough would not. Shortness of breath would indicate cardiac decompensation, but bradycardia and hypertension would not.

How well did you know this?
1
Not at all
2
3
4
5
Perfectly
14
Q

While providing care in an obstetric setting, the nurse should understand that after birth care of the woman with cardiac disease:

a. is the same as that for any pregnant woman.
b. includes rest, stool softeners, and monitoring of the effect of activity.
c. includes ambulating frequently, alternating with active range of motion.
d. includes limiting visits with the infant to once per day.

A

ANS: B
Bed rest may be ordered, with or without bathroom privileges. Bowel movements without stress or strain for the woman are promoted with stool softeners, diet, and fluid. Care of the woman with cardiac disease in the after birth period is tailored to the woman’s functional capacity. The woman will be on bed rest to conserve energy and reduce the strain on the heart. Although the woman may need help caring for the infant, breastfeeding and infant visits are not contraindicated.

How well did you know this?
1
Not at all
2
3
4
5
Perfectly
15
Q

A woman with asthma is experiencing a after birth hemorrhage. Which drug would not be used to treat her bleeding because it may exacerbate her asthma?

a. Pitocin
b. Nonsteroidal anti-inflammatory drugs (NSAIDs)
c. Hemabate
d. Fentanyl

A

ANS: C
Prostaglandin derivatives should not be used to treat women with asthma because they may exacerbate symptoms. Pitocin would be the drug of choice to treat this woman’s bleeding because it would not exacerbate her asthma. NSAIDs are not used to treat bleeding. Fentanyl is used to treat pain, not bleeding.

How well did you know this?
1
Not at all
2
3
4
5
Perfectly
16
Q

The use of methamphetamine (meth) has been described as a significant drug problem in the United States. In order to provide adequate nursing care to this patient population the nurse must be cognizant that methamphetamine:

a. is similar to opiates.
b. is a stimulant with vasoconstrictive characteristics.
c. should not be discontinued during pregnancy.
d. is associated with a low rate of relapse.

A

ANS: B
Methamphetamines are stimulants with vasoconstrictive characteristics similar to cocaine and are used similarly. As is the case with cocaine users, methamphetamine users are urged to immediately stop all use during pregnancy. Unfortunately, because methamphetamine users are extremely psychologically addicted, the rate of relapse is very high.

How well did you know this?
1
Not at all
2
3
4
5
Perfectly
17
Q

Since the gene for cystic fibrosis was identified in 1989, data can be collected for the purposes of genetic counseling for couples regarding carrier status. According to statistics, how often does cystic fibrosis occur in Caucasian live births?

a. 1 in 100
b. 1 in 1200
c. 1 in 2500
d. 1 in 3000

A

ANS: D

Cystic fibrosis occurs in about 1 in 3000 Caucasian live births.

How well did you know this?
1
Not at all
2
3
4
5
Perfectly
18
Q

Which heart condition is not a contraindication for pregnancy?

a. Peripartum cardiomyopathy
b. Eisenmenger syndrome
c. Heart transplant
d. All of these contraindicate pregnancy

A

ANS: C
Pregnancy is contraindicated for peripartum cardiomyopathy and Eisenmenger syndrome. Women who have had heart transplants are successfully having babies. However, conception should be postponed for at least 1 year after transplantation.

How well did you know this?
1
Not at all
2
3
4
5
Perfectly
19
Q

During a physical assessment of an at-risk patient, the nurse notes generalized edema, crackles at the base of the lungs, and some pulse irregularity. These are most likely signs of:

a. euglycemia.
b. rheumatic fever.
c. pneumonia.
d. cardiac decompensation.

A

ANS: D
Symptoms of cardiac decompensation may appear abruptly or gradually. Euglycemia is a condition of normal glucose levels. These symptoms indicate cardiac decompensation. Rheumatic fever can cause heart problems, but it does not manifest with these symptoms, which indicate cardiac decompensation. Pneumonia is an inflammation of the lungs and would not likely generate these symptoms, which indicate cardiac decompensation.

How well did you know this?
1
Not at all
2
3
4
5
Perfectly
20
Q

Nurses caring for antepartum women with cardiac conditions should be aware that:

a. stress on the heart is greatest in the first trimester and the last 2 weeks before labor.
b. women with Class II cardiac disease should avoid heavy exertion and any activity that causes even minor symptoms.
c. women with Class III cardiac disease should have 8 to 10 hours of sleep every day and limit housework, shopping, and exercise.
d. Women with Class I cardiac disease need bed rest through most of the pregnancy and face the possibility of hospitalization near term.

A

ANS: B
Class II cardiac disease is symptomatic with ordinary activity. Women in this category need to avoid heavy exertion and limit regular activities as symptoms dictate. Stress is greatest between weeks 28 and 32, when homodynamic changes reach their maximum. Class III cardiac disease is symptomatic with less than ordinary activity. These women need bed rest most of the day and face the possibility of hospitalization near term. Class I cardiac disease is asymptomatic at normal levels of activity. These women can carry on limited normal activities with discretion, although they still need a good amount of sleep.

How well did you know this?
1
Not at all
2
3
4
5
Perfectly
21
Q

As related to the care of the patient with anemia, the nurse should be aware that:

a. it is the most common medical disorder of pregnancy.
b. it can trigger reflex brachycardia.
c. the most common form of anemia is caused by folate deficiency.
d. thalassemia is a european version of sickle cell anemia

A

ANS: A
Combined with any other complication, anemia can result in congestive heart failure. Reflex bradycardia is a slowing of the heart in response to the blood flow increases immediately after birth. The most common form of anemia is iron deficiency anemia. Both thalassemia and sickle cell hemoglobinopathy are hereditary but not directly related or confined to geographic areas.

How well did you know this?
1
Not at all
2
3
4
5
Perfectly
22
Q

The most common neurologic disorder accompanying pregnancy is:

a. eclampsia.
b. Bell’s palsy.
c. epilepsy.
d. multiple sclerosis.

A

ANS: C
The effects of pregnancy on epilepsy are unpredictable. Eclampsia sometimes may be confused with epilepsy, which is the most common neurologic disorder accompanying pregnancy. Bell’s palsy is a form of facial paralysis. Multiple sclerosis is a patchy demyelinization of the spinal cord that does not affect the normal course of pregnancy or birth.

How well did you know this?
1
Not at all
2
3
4
5
Perfectly
23
Q

With one exception, the safest pregnancy is one in which the woman is drug and alcohol free. For women addicted to opioids, ________________________ treatment is the current standard of care during pregnancy.

a. methadone maintenance
b. detoxification
c. smoking cessation
d. 4 Ps Plus

A

ANS: A
Methadone maintenance treatment (MMT) is currently considered the standard of care for pregnant women who are dependent on heroin or other narcotics. Buprenorphine is another medication approved for opioid addiction treatment that is increasingly being used during pregnancy. Opioid replacement therapy has been shown to decrease opioid and other drug use, reduce criminal activity, improve individual functioning, and decrease rates of infections such as hepatitis B and C, HIV, and other sexually transmitted infections. Detoxification is the treatment used for alcohol addiction. Pregnant women requiring withdrawal from alcohol should be admitted for inpatient management. Women are more likely to stop smoking during pregnancy than at any other time in their lives. A smoking cessation program can assist in achieving this goal. The 4 Ps Plus is a screening tool designed specifically to identify pregnant women who need in-depth assessment related to substance abuse.

How well did you know this?
1
Not at all
2
3
4
5
Perfectly
24
Q

Which major neonatal complication is carefully monitored after the birth of the infant of a diabetic mother?

a. Hypoglycemia
b. Hypercalcemia
c. Hypobilirubinemia
d. Hypoinsulinemia

A

ANS: A
The neonate is at highest risk for hypoglycemia because fetal insulin production is accelerated during pregnancy to metabolize excessive glucose from the mother. At birth, the maternal glucose supply stops and the neonatal insulin exceeds the available glucose, thus leading to hypoglycemia. Hypocalcemia is associated with preterm birth, birth trauma, and asphyxia, all common problems of the infant of a diabetic mother. Excess erythrocytes are broken down after birth and release large amounts of bilirubin into the neonate’s circulation, with resulting hyperbilirubinemia. Because fetal insulin production is accelerated during pregnancy, the neonate presents with hyperinsulinemia.

How well did you know this?
1
Not at all
2
3
4
5
Perfectly
25
Q

Which factor is known to increase the risk of gestational diabetes mellitus?

a. Underweight before pregnancy
b. Maternal age younger than 25 years
c. Previous birth of large infant
d. Previous diagnosis of type 2 diabetes mellitus

A

ANS: C
Previous birth of a large infant suggests gestational diabetes mellitus. Obesity (BMI of 30 or greater) creates a higher risk for gestational diabetes. A woman younger than 25 years generally is not at risk for gestational diabetes mellitus. The person with type 2 diabetes mellitus already has diabetes and will continue to have it after pregnancy. Insulin may be required during pregnancy because oral hypoglycemia drugs are contraindicated during pregnancy.

How well did you know this?
1
Not at all
2
3
4
5
Perfectly
26
Q

Glucose metabolism is profoundly affected during pregnancy because:

a. pancreatic function in the islets of Langerhans is affected by pregnancy.
b. the pregnant woman uses glucose at a more rapid rate than the nonpregnant woman.
c. the pregnant woman increases her dietary intake significantly.
d. placental hormones are antagonistic to insulin, thus resulting in insulin resistance.

A

ANS: D
Placental hormones, estrogen, progesterone, and human placental lactogen (HPL) create insulin resistance. Insulin is also broken down more quickly by the enzyme placental insulinase. Pancreatic functioning is not affected by pregnancy. The glucose requirements differ because of the growing fetus. The pregnant woman should increase her intake by 200 calories a day.

How well did you know this?
1
Not at all
2
3
4
5
Perfectly
27
Q

To manage her diabetes appropriately and ensure a good fetal outcome, the pregnant woman with diabetes will need to alter her diet by:

a. eating six small equal meals per day.
b. reducing carbohydrates in her diet.
c. eating her meals and snacks on a fixed schedule.
d. increasing her consumption of protein.

A

ANS: C
Having a fixed meal schedule will provide the woman and the fetus with a steadier blood sugar level, provide better balance with insulin administration, and help prevent complications. It is more important to have a fixed meal schedule than equal division of food intake. Approximately 45% of the food eaten should be in the form of carbohydrates.

How well did you know this?
1
Not at all
2
3
4
5
Perfectly
28
Q

When the pregnant diabetic woman experiences hypoglycemia while hospitalized, the nurse should intervene by having the patient:

a. eat six saltine crackers.
b. drink 8 ounces of orange juice with 2 tsp of sugar added.
c. drink 4 ounces of orange juice followed by 8 ounces of milk.
d. eat hard candy or commercial glucose wafers.

A

ANS: A
Crackers provide carbohydrates in the form of polysaccharides. Orange juice and sugar will increase the blood sugar but not provide a slow-burning carbohydrate to sustain the blood sugar. Milk is a disaccharide and orange juice is a monosaccharide. They will provide an increase in blood sugar but will not sustain the level. Hard candy or commercial glucose wafers provide only monosaccharides.

How well did you know this?
1
Not at all
2
3
4
5
Perfectly
29
Q

Nursing intervention for the pregnant diabetic patient is based on the knowledge that the need for insulin:

a. increases throughout pregnancy and the after birth period.
b. decreases throughout pregnancy and the after birth period.
c. varies depending on the stage of gestation.
d. should not change because the fetus produces its own insulin.

A

ANS: C
Insulin needs decrease during the first trimester, when nausea, vomiting, and anorexia are a factor. They increase during the second and third trimesters, when the hormones of pregnancy create insulin resistance in maternal cells. Insulin needs increase during the second and third trimesters, when the hormones of pregnancy create insulin resistance in maternal cells. The insulin needs change throughout the different stages of pregnancy.

How well did you know this?
1
Not at all
2
3
4
5
Perfectly
30
Q

In caring for a pregnant woman with sickle cell anemia, the nurse is aware that signs and symptoms of sickle cell crisis include:

a. anemia.
b. endometritis.
c. fever and pain.
d. urinary tract infection.

A

ANS: C
Women with sickle cell anemia have recurrent attacks (crisis) of fever and pain, most often in the abdomen, joints, and extremities. These attacks are attributed to vascular occlusion when RBCs assume the characteristic sickled shape. Crises are usually triggered by dehydration, hypoxia, or acidosis. Women with sickle cell anemia are not iron deficient. Therefore, routine iron supplementation, even that found in prenatal vitamins, should be avoided in order to prevent iron overload. Women with sickle cell trait usually are at greater risk for after birth endometritis (uterine wall infection); however, this is not likely to occur in pregnancy and is not a sign of crisis. These women are at an increased risk for UTIs; however, this is not an indication of sickle cell crisis.

How well did you know this?
1
Not at all
2
3
4
5
Perfectly
31
Q

Congenital anomalies can occur with the use of antiepileptic drugs (AEDs), including: (Select all that apply.)

a. cleft lip.
b. congenital heart disease.
c. neural tube defects.
d. gastroschisis.
e. diaphragmatic hernia.

A

ANS: A, B, C
Congenital anomalies that can occur with AEDs include cleft lip or palate, congenital heart disease, urogenital defects, and neural tube defects. Gastroschisis and diaphragmatic hernia are not associated with the use of AEDs.

How well did you know this?
1
Not at all
2
3
4
5
Perfectly
32
Q

Diabetes refers to a group of metabolic diseases characterized by hyperglycemia resulting from defects in insulin action, insulin secretion, or both. Over time, diabetes causes significant changes in the microvascular and macrovascular circulations. These complications include: (Select all that apply.)

a. atherosclerosis.
b. retinopathy.
c. IUFD.
d. nephropathy.
e. neuropathy.

A

ANS: A, B, D, E

These structural changes are most likely to affect a variety of systems, including the heart, eyes, kidneys, and nerves. Intrauterine fetal death (stillbirth) remains a major complication of diabetes in pregnancy; however, this is a fetal complication.

How well did you know this?
1
Not at all
2
3
4
5
Perfectly
33
Q

Autoimmune disorders often occur during pregnancy because a large percentage of women with an autoimmune disorder are of childbearing age. Identify all disorders that fall into the category of collagen vascular disease.

a. Multiple sclerosis
b. Systemic lupus erythematosus
c. Antiphospholipid syndrome
d. Rheumatoid arthritis
e. Myasthenia gravis

A

ANS: B, C, D, E
Multiple sclerosis is not an autoimmune disorder. This patchy demyelinization of the spinal cord may be a viral disorder. Autoimmune disorders (collagen vascular disease) make up a large group of conditions that disrupt the function of the immune system of the body. They include those listed, as well as systemic sclerosis.

How well did you know this?
1
Not at all
2
3
4
5
Perfectly
34
Q

Achieving and maintaining euglycemia comprise the primary goals of medical therapy for the pregnant woman with diabetes. These goals are achieved through a combination of diet, insulin, exercise, and blood glucose monitoring. The target blood glucose levels 1 hour after a meal should be _________________.

A

ANS:
130 to 140 mg/dL

Target levels of blood glucose during pregnancy are lower than nonpregnant values. Accepted fasting levels are between 65 and 95 mg/dL, and 1-hour postmeal levels should be less than 130 to 140 mg/dL. Two-hour postmeal levels should be 120 mg/dL or less.

How well did you know this?
1
Not at all
2
3
4
5
Perfectly
35
Q

Women with hyperemesis gravidarum:

a. are a majority because 80% of all pregnant women suffer from it at some time.
b. have vomiting severe and persistent enough to cause weight loss, dehydration, and electrolyte imbalance.
c. need intravenous (IV) fluid and nutrition for most of their pregnancy.
d. often inspire similar, milder symptoms in their male partners and mothers.

A

ANS: B
Women with hyperemesis gravidarum have severe vomiting; however, treatment for several days sets things right in most cases. Although 80% of pregnant women experience nausea and vomiting, fewer than 1% (0.5%) proceed to this severe level. IV administration may be used at first to restore fluid levels, but it is seldom needed for very long. Women suffering from this condition want sympathy because some authorities believe that difficult relationships with mothers and/or partners may be the cause.

How well did you know this?
1
Not at all
2
3
4
5
Perfectly
36
Q

Because pregnant women may need surgery during pregnancy, nurses should be aware that:

a. the diagnosis of appendicitis may be difficult because the normal signs and symptoms mimic some normal changes in pregnancy.
b. rupture of the appendix is less likely in pregnant women because of the close monitoring.
c. surgery for intestinal obstructions should be delayed as long as possible because it usually affects the pregnancy.
d. when pregnancy takes over, a woman is less likely to have ovarian problems that require invasive responses.

A

ANS: A
Both appendicitis and pregnancy are linked with nausea, vomiting, and increased white blood cell count. Rupture of the appendix is two to three times more likely in pregnant women. Surgery to remove obstructions should be done right away. It usually does not affect the pregnancy. Pregnancy predisposes a woman to ovarian problems.

How well did you know this?
1
Not at all
2
3
4
5
Perfectly
37
Q

What laboratory marker is indicative of disseminated intravascular coagulation (DIC)?

a. Bleeding time of 10 minutes
b. Presence of fibrin split products
c. Thrombocytopenia
d. Hyperfibrinogenemia

A

ANS B
Degradation of fibrin leads to the accumulation of fibrin split products in the blood. Bleeding time in DIC is normal. Low platelets may occur with but are not indicative of DIC because they may result from other coagulopathies. Hypofibrinogenemia would occur with DIC.

How well did you know this?
1
Not at all
2
3
4
5
Perfectly
38
Q

In caring for an immediate after birth patient, you note petechiae and oozing from her IV site. You would monitor her closely for the clotting disorder:

a. disseminated intravascular coagulation (DIC).
b. amniotic fluid embolism (AFE).
c. hemorrhage.
d. HELLP syndrome.

A

ANS: A
The diagnosis of DIC is made according to clinical findings and laboratory markers. Physical examination reveals unusual bleeding. Petechiae may appear around a blood pressure cuff on the woman’s arm. Excessive bleeding may occur from the site of slight trauma such as venipuncture sites. These symptoms are not associated with AFE, nor is AFE a bleeding disorder. Hemorrhage occurs for a variety of reasons in the after birth patient. These symptoms are associated with DIC. Hemorrhage would be a finding associated with DIC and is not a clotting disorder in and of itself. HELLP is not a clotting disorder, but it may contribute to the clotting disorder DIC.

How well did you know this?
1
Not at all
2
3
4
5
Perfectly
39
Q

In caring for the woman with disseminated intravascular coagulation (DIC), what order should the nurse anticipate?

a. Administration of blood
b. Preparation of the patient for invasive hemodynamic monitoring
c. Restriction of intravascular fluids
d. Administration of steroids

A

ANS: A
Primary medical management in all cases of DIC involves correction of the underlying cause, volume replacement, blood component therapy, optimization of oxygenation and perfusion status, and continued reassessment of laboratory parameters. Central monitoring would not be ordered initially in a patient with DIC because this can contribute to more areas of bleeding. Management of DIC would include volume replacement, not volume restriction. Steroids are not indicated for the management of DIC.

How well did you know this?
1
Not at all
2
3
4
5
Perfectly
40
Q

A primigravida is being monitored in her prenatal clinic for preeclampsia. What finding should concern her nurse?

a. Blood pressure (BP) increase to 138/86 mm Hg.
b. Weight gain of 0.5 kg during the past 2 weeks.
c. A dipstick value of 3+ for protein in her urine.
d. Pitting pedal edema at the end of the day.

A

ANS: C
Proteinuria is defined as a concentration of 1+ or greater via dipstick measurement. A dipstick value of 3+ should alert the nurse that additional testing or assessment should be made. Generally, hypertension is defined as a BP of 140/90 or an increase in systolic pressure of 30 mm Hg or in diastolic pressure of 15 mm Hg. Preeclampsia may be manifested as a rapid weight gain of more than 2 kg in 1 week. Edema occurs in many normal pregnancies and in women with preeclampsia. Therefore, the presence of edema is no longer considered diagnostic of preeclampsia.

How well did you know this?
1
Not at all
2
3
4
5
Perfectly
41
Q

The labor of a pregnant woman with preeclampsia is going to be induced. Before initiating the Pitocin infusion, the nurse reviews the woman’s latest laboratory test findings, which reveal a platelet count of 90,000, an elevated aspartate transaminase (AST) level, and a falling hematocrit. The nurse notifies the physician because the laboratory results are indicative of:

a. eclampsia.
b. disseminated intravascular coagulation (DIC).
c. HELLP syndrome.
d. idiopathic thrombocytopenia.

A

ANS: C
HELLP syndrome is a laboratory diagnosis for a variant of severe preeclampsia that involves hepatic dysfunction characterized by hemolysis (H), elevated liver enzymes (EL), and low platelets (LP). Eclampsia is determined by the presence of seizures. DIC is a potential complication associated with HELLP syndrome. Idiopathic thrombocytopenia is the presence of low platelets of unknown cause and is not associated with preeclampsia.

How well did you know this?
1
Not at all
2
3
4
5
Perfectly
42
Q

A woman with preeclampsia has a seizure. The nurse’s primary duty during the seizure is to:

a. insert an oral airway.
b. suction the mouth to prevent aspiration.
c. administer oxygen by mask.
d. stay with the patient and call for help.

A

ANS: D
If a patient becomes eclamptic, the nurse should stay her and call for help. Insertion of an oral airway during seizure activity is no longer the standard of care. The nurse should attempt to keep the airway patent by turning the patient’s head to the side to prevent aspiration. Once the seizure has ended, it may be necessary to suction the patient’s mouth. Oxygen would be administered after the convulsion has ended.

How well did you know this?
1
Not at all
2
3
4
5
Perfectly
43
Q

A pregnant woman has been receiving a magnesium sulfate infusion for treatment of severe preeclampsia for 24 hours. On assessment the nurse finds the following vital signs: temperature of 37.3° C, pulse rate of 88 beats/min, respiratory rate of 10 breaths/min, blood pressure (BP) of 148/90 mm Hg, absent deep tendon reflexes, and no ankle clonus. The patient complains, “I’m so thirsty and warm.” The nurse:

a. calls for a stat magnesium sulfate level.
b. administers oxygen.
c. discontinues the magnesium sulfate infusion.
d. prepares to administer hydralazine.

A

ANS: C
The patient is displaying clinical signs and symptoms of magnesium toxicity. Magnesium should be discontinued immediately. In addition, calcium gluconate, the antidote for magnesium, may be administered. Hydralazine is an antihypertensive commonly used to treat hypertension in severe preeclampsia. Typically it is administered for a systolic BP greater than 160 mm Hg or a diastolic BP greater than 110 mm Hg.

How well did you know this?
1
Not at all
2
3
4
5
Perfectly
44
Q

A woman with severe preeclampsia has been receiving magnesium sulfate by intravenous infusion for 8 hours. The nurse assesses the woman and documents the following findings: temperature of 37.1° C, pulse rate of 96 beats/min, respiratory rate of 24 breaths/min, blood pressure (BP) of 155/112 mm Hg, 3+ deep tendon reflexes, and no ankle clonus. The nurse calls the physician, anticipating an order for:

a. hydralazine.
b. magnesium sulfate bolus.
c. diazepam.
d. calcium gluconate.

A

ANS: A
Hydralazine is an antihypertensive commonly used to treat hypertension in severe preeclampsia. Typically, it is administered for a systolic BP greater than 160 mm Hg or a diastolic BP greater than 110 mm Hg. An additional bolus of magnesium sulfate may be ordered for increasing signs of central nervous system irritability related to severe preeclampsia (e.g., clonus) or if eclampsia develops. Diazepam sometimes is used to stop or shorten eclamptic seizures. Calcium gluconate is used as the antidote for magnesium sulfate toxicity. The patient is not currently displaying any signs or symptoms of magnesium toxicity.

How well did you know this?
1
Not at all
2
3
4
5
Perfectly
45
Q

A woman at 39 weeks of gestation with a history of preeclampsia is admitted to the labor and birth unit. She suddenly experiences increased contraction frequency of every 1 to 2 minutes; dark red vaginal bleeding; and a tense, painful abdomen. The nurse suspects the onset of:

a. eclamptic seizure.
b. rupture of the uterus.
c. placenta previa.
d. placental abruption.

A

ANS: D

Uterine tenderness in the presence of increasing tone may be the earliest finding of premature separation of the placenta (abruptio placentae or placental abruption). Women with hypertension are at increased risk for an abruption. Eclamptic seizures are evidenced by the presence of generalized tonic-clonic convulsions. Uterine rupture manifests as hypotonic uterine activity, signs of hypovolemia, and in many cases the absence of pain. Placenta previa manifests with bright red, painless vaginal bleeding.

How well did you know this?
1
Not at all
2
3
4
5
Perfectly
46
Q

The patient that you are caring for has severe preeclampsia and is receiving a magnesium sulfate infusion. You become concerned after assessment when the woman exhibits:

a. a sleepy, sedated affect.
b. a respiratory rate of 10 breaths/min.
c. deep tendon reflexes of 2.
d. absent ankle clonus.

A

ANS: B
A respiratory rate of 10 breaths/min indicates that the patient is experiencing respiratory depression from magnesium toxicity. Because magnesium sulfate is a central nervous system depressant, the patient will most likely become sedated when the infusion is initiated. Deep tendon reflexes of two and absent ankle clonus are normal findings.

How well did you know this?
1
Not at all
2
3
4
5
Perfectly
47
Q

The nurse caring for pregnant women must be aware that the most common medical complication of pregnancy is:

a. hypertension.
b. hyperemesis gravidarum.
c. hemorrhagic complications.
d. infections.

A

ANS: A
Preeclampsia and eclampsia are two noted deadly forms of hypertension. A large percentage of pregnant women will have nausea and vomiting, but a relatively few have the severe form called hyperemesis gravidarum. Hemorrhagic complications are the second most common medical complication of pregnancy; hypertension is the most common.

How well did you know this?
1
Not at all
2
3
4
5
Perfectly
48
Q

Nurses should be aware that HELLP syndrome:

a. is a mild form of preeclampsia.
b. can be diagnosed by a nurse alert to its symptoms.
c. is characterized by hemolysis, elevated liver enzymes, and low platelets.
d. is associated with preterm labor but not perinatal mortality.

A

ANS C
The acronym HELLP stands for hemolysis (H), elevated liver enzymes (EL), and low platelets (LP). HELLP syndrome is a variant of severe preeclampsia. HELLP syndrome is difficult to identify because the symptoms often are not obvious. It must be diagnosed in the laboratory. Preterm labor is greatly increased, and so is perinatal mortality.

How well did you know this?
1
Not at all
2
3
4
5
Perfectly
49
Q

Nurses should be aware that chronic hypertension:

a. is defined as hypertension that begins during pregnancy and lasts for the duration of pregnancy.
b. is considered severe when the systolic blood pressure (BP) is greater than 140 mm Hg or the diastolic BP is greater than 90 mm Hg.
c. is general hypertension plus proteinuria.
d. can occur independently of or simultaneously with gestational hypertension.

A

ANS: D
Hypertension is present before pregnancy or diagnosed before 20 weeks of gestation and persists longer than 6 weeks after birth. The range for hypertension is systolic BP greater than 140 mm Hg or diastolic BP greater than 90 mm Hg. It becomes severe with a diastolic BP of 110 mm Hg or higher. Proteinuria is an excessive concentration of protein in the urine. It is a complication of hypertension, not a defining characteristic.

How well did you know this?
1
Not at all
2
3
4
5
Perfectly
50
Q

In planning care for women with preeclampsia, nurses should be aware that:

a. induction of labor is likely, as near term as possible.
b. if at home, the woman should be confined to her bed, even with mild preeclampsia.
c. a special diet low in protein and salt should be initiated.
d. vaginal birth is still an option, even in severe cases.

A

ANS: A
Induction of labor is likely, as near term as possible; however, at less than 37 weeks of gestation, immediate delivery may not be in the best interest of the fetus. Strict bed rest is becoming controversial for mild cases; some women in the hospital are even allowed to move around. Diet and fluid recommendations are much the same as for healthy pregnant women, although some authorities have suggested a diet high in protein. Women with severe preeclampsia should expect a cesarean delivery.

How well did you know this?
1
Not at all
2
3
4
5
Perfectly
51
Q

Magnesium sulfate is given to women with preeclampsia and eclampsia to:

a. improve patellar reflexes and increase respiratory efficiency.
b. shorten the duration of labor.
c. prevent and treat convulsions.
d. prevent a boggy uterus and lessen lochial flow.

A

ANS: C

Magnesium sulfate is the drug of choice to prevent convulsions, although it can generate other problems. Loss of patellar reflexes and respiratory depression are signs of magnesium toxicity. Magnesium sulfate can increase the duration of labor. Women are at risk for a boggy uterus and heavy lochial flow as a result of magnesium sulfate therapy

How well did you know this?
1
Not at all
2
3
4
5
Perfectly
52
Q

A woman presents to the emergency department with complaints of bleeding and cramping. The initial nursing history is significant for a last menstrual period 6 weeks ago. On sterile speculum examination, the primary care provider finds that the cervix is closed. The anticipated plan of care for this woman would be based on a probable diagnosis of which type of spontaneous abortion?

a. Incomplete
b. Inevitable
c. Threatened
d. Septic

A

ANS: C
A woman with a threatened abortion presents with spotting, mild cramps, and no cervical dilation. A woman with an incomplete abortion would present with heavy bleeding, mild to severe cramping, and cervical dilation. An inevitable abortion manifests with the same symptoms as an incomplete abortion: heavy bleeding, mild to severe cramping, and cervical dilation. A woman with a septic abortion presents with malodorous bleeding and typically a dilated cervix.

How well did you know this?
1
Not at all
2
3
4
5
Perfectly
53
Q

The perinatal nurse is giving discharge instructions to a woman after suction curettage secondary to a hydatidiform mole. The woman asks why she must take oral contraceptives for the next 12 months. The best response from the nurse would be:

a. “If you get pregnant within 1 year, the chance of a successful pregnancy is very small. Therefore, if you desire a future pregnancy, it would be better for you to use the most reliable method of contraception available.”
b. “The major risk to you after a molar pregnancy is a type of cancer that can be diagnosed only by measuring the same hormone that your body produces during pregnancy. If you were to get pregnant, it would make the diagnosis of this cancer more difficult.”
c. “If you can avoid a pregnancy for the next year, the chance of developing a second molar pregnancy is rare. Therefore, to improve your chance of a successful pregnancy, it is better not to get pregnant at this time.”
d. “Oral contraceptives are the only form of birth control that will prevent a recurrence of a molar pregnancy.”

A

ANS: B

This is an accurate statement. Beta-human chorionic gonadotropin (hCG) levels will be drawn for 1 year to ensure that the mole is completely gone. There is an increased chance of developing choriocarcinoma after the development of a hydatidiform mole. The goal is to achieve a “zero” hCG level. If the woman were to become pregnant, it could obscure the presence of the potentially carcinogenic cells. Women should be instructed to use birth control for 1 year after treatment for a hydatidiform mole. The rationale for avoiding pregnancy for 1 year is to ensure that carcinogenic cells are not present. Any contraceptive method except an intrauterine device is acceptable.

How well did you know this?
1
Not at all
2
3
4
5
Perfectly
54
Q

The most prevalent clinical manifestation of abruptio placentae (as opposed to placenta previa) is:

a. bleeding.
b. intense abdominal pain.
c. uterine activity.
d. cramping.

A

ANS: B
Pain is absent with placenta previa and may be agonizing with abruptio placentae. Bleeding may be present in varying degrees for both placental conditions. Uterine activity and cramping may be present with both placental conditions.

How well did you know this?
1
Not at all
2
3
4
5
Perfectly
55
Q

Methotrexate is recommended as part of the treatment plan for which obstetric complication?

a. Complete hydatidiform mole
b. Missed abortion
c. Unruptured ectopic pregnancy
d. Abruptio placentae

A

ANS: C
Methotrexate is an effective, nonsurgical treatment option for a hemodynamically stable woman whose ectopic pregnancy is unruptured and less than 4 cm in diameter. Methotrexate is not indicated or recommended as a treatment option for complete hydatidiform mole, missed abortion, and abruptio placentae.

How well did you know this?
1
Not at all
2
3
4
5
Perfectly
56
Q

A 26-year-old pregnant woman, gravida 2, para 1-0-0-1 is 28 weeks pregnant when she experiences bright red, painless vaginal bleeding. On her arrival at the hospital, what would be an expected diagnostic procedure?

a. Amniocentesis for fetal lung maturity
b. Ultrasound for placental location
c. Contraction stress test (CST)
d. Internal fetal monitoring

A

ANS B
The presence of painless bleeding should always alert the health care team to the possibility of placenta previa. This can be confirmed through ultrasonography. Amniocentesis would not be performed on a woman who is experiencing bleeding. In the event of an imminent delivery, the fetus would be presumed to have immature lungs at this gestational age, and the mother would be given corticosteroids to aid in fetal lung maturity. A CST would not be performed at a preterm gestational age. Furthermore, bleeding would be a contraindication to this test. Internal fetal monitoring would be contraindicated in the presence of bleeding.

How well did you know this?
1
Not at all
2
3
4
5
Perfectly
57
Q

A laboring woman with no known risk factors suddenly experiences spontaneous rupture of membranes (ROM). The fluid consists of bright red blood. Her contractions are consistent with her current stage of labor. There is no change in uterine resting tone. The fetal heart rate begins to decline rapidly after the ROM. The nurse should suspect the possibility of:

a. placenta previa.
b. vasa previa.
c. severe abruptio placentae.
d. disseminated intravascular coagulation (DIC).

A

ANS: B
Vasa previa is the result of a velamentous insertion of the umbilical cord. The umbilical vessels are not surrounded by Wharton jelly and have no supportive tissue. They are at risk for laceration at any time, but laceration occurs most frequently during ROM. The sudden appearance of bright red blood at the time of ROM and a sudden change in the fetal heart rate without other known risk factors should immediately alert the nurse to the possibility of vasa previa. The presence of placenta previa most likely would be ascertained before labor and would be considered a risk factor for this pregnancy. In addition, if the woman had a placenta previa, it is unlikely that she would be allowed to pursue labor and a vaginal birth. With the presence of severe abruptio placentae, the uterine tonicity would typically be tetanus (i.e., a board-like uterus). DIC is a pathologic form of diffuse clotting that consumes large amounts of clotting factors and causes widespread external bleeding, internal bleeding, or both. DIC is always a secondary diagnosis, often associated with obstetric risk factors such as HELLP syndrome. This woman did not have any prior risk factors.

How well did you know this?
1
Not at all
2
3
4
5
Perfectly
58
Q

A woman arrives for evaluation of her symptoms, which include a missed period, adnexal fullness, tenderness, and dark red vaginal bleeding. On examination the nurse notices an ecchymotic blueness around the woman’s umbilicus and recognizes this assessment finding as:

a. normal integumentary changes associated with pregnancy.
b. Turner’s sign associated with appendicitis.
c. Cullen’s sign associated with a ruptured ectopic pregnancy.
d. Chadwick’s sign associated with early pregnancy.

A

ANS: C

Cullen’s sign, the blue ecchymosis seen in the umbilical area, indicates hematoperitoneum associated with an undiagnosed ruptured intraabdominal ectopic pregnancy. Linea nigra on the abdomen is the normal integumentary change associated with pregnancy. It manifests as a brown, pigmented, vertical line on the lower abdomen. Turner’s sign is ecchymosis in the flank area, often associated with pancreatitis. Chadwick’s sign is the blue-purple color of the cervix that may be seen during or around the eighth week of pregnancy.

How well did you know this?
1
Not at all
2
3
4
5
Perfectly
59
Q

As related to the care of the patient with miscarriage, nurses should be aware that:

a. it is a natural pregnancy loss before labor begins.
b. it occurs in fewer than 5% of all clinically recognized pregnancies.
c. it often can be attributed to careless maternal behavior such as poor nutrition or excessive exercise.
d. if it occurs before the 12th week of pregnancy, it may manifest only as moderate discomfort and blood loss.

A

ANS: D
Before the sixth week the only evidence may be a heavy menstrual flow. After the 12th week more severe pain, similar to that of labor, is likely. Miscarriage is a natural pregnancy loss, but by definition it occurs before 20 weeks of gestation, before the fetus is viable. Miscarriages occur in approximately 10% to 15% of all clinically recognized pregnancies. Miscarriage can be caused by a number of disorders or illnesses outside of the mother’s control or knowledge.

How well did you know this?
1
Not at all
2
3
4
5
Perfectly
60
Q

Which condition would not be classified as a bleeding disorder in late pregnancy?

a. Placenta previa
b. Abruptio placentae
c. Spontaneous abortion
d. Cord insertion

A

ANS: C
Spontaneous abortion is another name for miscarriage; by definition it occurs early in pregnancy. Placenta previa is a cause of bleeding disorders in later pregnancy. Abruptio placentae is a cause of bleeding disorders in later pregnancy. Cord insertion is a cause of bleeding disorders in later pregnancy.

How well did you know this?
1
Not at all
2
3
4
5
Perfectly
61
Q

In providing nutritional counseling for the pregnant woman experiencing cholecystitis, the nurse would:

a. assess the woman’s dietary history for adequate calories and proteins.
b. instruct the woman that the bulk of calories should come from proteins.
c. instruct the woman to eat a low-fat diet and avoid fried foods.
d. instruct the woman to eat a low-cholesterol, low-salt diet.

A

ANS: C

Instructing the woman to eat a low-fat diet and avoid fried foods is appropriate nutritional counseling for this patient. Caloric and protein intake do not predispose a woman to the development of cholecystitis. The woman should be instructed to limit protein intake and choose foods that are high in carbohydrates. A low-cholesterol diet may be the result of limiting fats. However, a low-salt diet is not indicated.

How well did you know this?
1
Not at all
2
3
4
5
Perfectly
62
Q

Which maternal condition always necessitates delivery by cesarean section?

a. Partial abruptio placentae
b. Total placenta previa
c. Ectopic pregnancy
d. Eclampsia

A

ANS: B
In total placenta previa, the placenta completely covers the cervical os. The fetus would die if a vaginal delivery occurred. If the mother has stable vital signs and the fetus is alive, a vaginal delivery can be attempted in cases of partial abruptio placentae. If the fetus has died, a vaginal delivery is preferred. The most common ectopic pregnancy is a tubal pregnancy, which is usually detected and treated in the first trimester. Labor can be safely induced if the eclampsia is under control.

How well did you know this?
1
Not at all
2
3
4
5
Perfectly
63
Q

Spontaneous termination of a pregnancy is considered to be an abortion if:

a. the pregnancy is less than 20 weeks.
b. the fetus weighs less than 1000 g.
c. the products of conception are passed intact.
d. no evidence exists of intrauterine infection.

A

ANS: A
An abortion is the termination of pregnancy before the age of viability (20 weeks). The weight of the fetus is not considered because some older fetuses may have a low birth weight. A spontaneous abortion may be complete or incomplete. A spontaneous abortion may be caused by many problems, one being intrauterine infection.

How well did you know this?
1
Not at all
2
3
4
5
Perfectly
64
Q

An abortion in which the fetus dies but is retained within the uterus is called a(n):

a. inevitable abortion.
b. missed abortion.
c. incomplete abortion.
d. threatened abortion.

A

ANS: B
Missed abortion refers to retention of a dead fetus in the uterus. An inevitable abortion means that the cervix is dilating with the contractions. An incomplete abortion means that not all of the products of conception were expelled. With a threatened abortion the woman has cramping and bleeding but not cervical dilation.

How well did you know this?
1
Not at all
2
3
4
5
Perfectly
65
Q

A placenta previa in which the placental edge just reaches the internal os is more commonly known as:

a. total.
b. partial.
c. complete.
d. marginal.

A

ANS: D
A placenta previa that does not cover any part of the cervix is termed marginal. With a total placenta previa, the placenta completely covers the os. When the patient experiences a partial placenta previa, the lower border of the placenta is within 3 cm of the internal cervical os but does not completely cover the os. A complete placenta previa is termed total. The placenta completely covers the internal cervical os.

How well did you know this?
1
Not at all
2
3
4
5
Perfectly
66
Q

Which condition indicates concealed hemorrhage when the patient experiences an abruptio placentae?

a. Decrease in abdominal pain
b. Bradycardia
c. Hard, board-like abdomen
d. Decrease in fundal height

A

ANS: C
Concealed hemorrhage occurs when the edges of the placenta do not separate. The formation of a hematoma behind the placenta and subsequent infiltration of the blood into the uterine muscle results in a very firm, board-like abdomen. Abdominal pain may increase. The patient will have shock symptoms that include tachycardia. As bleeding occurs, the fundal height will increase.

How well did you know this?
1
Not at all
2
3
4
5
Perfectly
67
Q

The priority nursing intervention when admitting a pregnant woman who has experienced a bleeding episode in late pregnancy is to:

a. assess fetal heart rate (FHR) and maternal vital signs.
b. perform a venipuncture for hemoglobin and hematocrit levels.
c. place clean disposable pads to collect any drainage.
d. monitor uterine contractions.

A

ANS: A
Assessment of the FHR and maternal vital signs will assist the nurse in determining the degree of the blood loss and its effect on the mother and fetus. The most important assessment is to check mother/fetal well-being. The blood levels can be obtained later. It is important to assess future bleeding; however, the top priority remains mother/fetal well-being. Monitoring uterine contractions is important but not the top priority.

How well did you know this?
1
Not at all
2
3
4
5
Perfectly
68
Q

A patient with pregnancy-induced hypertension is admitted complaining of pounding headache, visual changes, and epigastric pain. Nursing care is based on the knowledge that these signs are an indication of:

a. anxiety due to hospitalization.
b. worsening disease and impending convulsion.
c. effects of magnesium sulfate.
d. gastrointestinal upset.

A

ANS: B
Headache and visual disturbances are caused by increased cerebral edema. Epigastric pain indicates distention of the hepatic capsules and often warns that a convulsion is imminent. These are danger signs showing increased cerebral edema and impending convulsion and should be treated immediately. The patient has not been started on magnesium sulfate treatment yet. Also, these are not anticipated effects of the medication.

How well did you know this?
1
Not at all
2
3
4
5
Perfectly
69
Q

Which order should the nurse expect for a patient admitted with a threatened abortion?

a. Bed rest
b. Ritodrine IV
c. NPO
d. Narcotic analgesia every 3 hours, prn

A

ANS: A
Decreasing the woman’s activity level may alleviate the bleeding and allow the pregnancy to continue. Ritodrine IV is not the first drug of choice for tocolytic medications. There is no reason for having the woman placed NPO. At times dehydration may produce contractions, so hydration is important. Narcotic analgesia will not decrease the contractions. It may mask the severity of the contractions.

How well did you know this?
1
Not at all
2
3
4
5
Perfectly
70
Q

A 32-year-old primigravida is admitted with a diagnosis of ectopic pregnancy. Nursing care is based on the knowledge that:

a. bed rest and analgesics are the recommended treatment.
b. she will be unable to conceive in the future.
c. a D&C will be performed to remove the products of conception.
d. hemorrhage is the major concern.

A

ANS: D
Severe bleeding occurs if the fallopian tube ruptures. The recommended treatment is to remove the pregnancy before rupture in order to prevent hemorrhaging. If the tube must be removed, the woman’s fertility will decrease; however, she will not be infertile.
D&C is performed on the inside of the uterine cavity. The ectopic pregnancy is located within the tubes.

How well did you know this?
1
Not at all
2
3
4
5
Perfectly
71
Q

Approximately 10% to 15% of all clinically recognized pregnancies end in miscarriage. Which is the most common cause of spontaneous abortion?

a. Chromosomal abnormalities
b. Infections
c. Endocrine imbalance
d. Immunologic factors

A

ANS: A
At least 50% of pregnancy losses result from chromosomal abnormalities that are incompatible with life. Maternal infection may be a cause of early miscarriage. Endocrine imbalances such as hypothyroidism or diabetes are possible causes for early pregnancy loss. Women who have repeated early pregnancy losses appear to have immunologic factors that play a role in spontaneous abortion incidents.

How well did you know this?
1
Not at all
2
3
4
5
Perfectly
72
Q

The nurse caring for a woman hospitalized for hyperemesis gravidarum should expect that initial treatment to involve:

a. corticosteroids to reduce inflammation.
b. IV therapy to correct fluid and electrolyte imbalances.
c. an antiemetic, such as pyridoxine, to control nausea and vomiting.
d. enteral nutrition to correct nutritional deficits.

A

ANS: B
Initially, the woman who is unable to keep down clear liquids by mouth requires IV therapy for correction of fluid and electrolyte imbalances. Corticosteroids have been used successfully to treat refractory hyperemesis gravidarum; however, they are not the expected initial treatment for this disorder. Pyridoxine is vitamin B6, not an antiemetic. Promethazine, a common antiemetic, may be prescribed. In severe cases of hyperemesis gravidarum, enteral nutrition via a feeding tube may be necessary to correct maternal nutritional deprivation. This is not an initial treatment for this patient.

How well did you know this?
1
Not at all
2
3
4
5
Perfectly
73
Q

A patient who has undergone a dilation and curettage for early pregnancy loss is likely to be discharged the same day. The nurse must ensure that vital signs are stable, bleeding has been controlled, and the woman has adequately recovered from the administration of anesthesia. To promote an optimal recovery, discharge teaching should include: (Select all that apply.)

a. iron supplementation.
b. resumption of intercourse at 6 weeks following the procedure.
c. referral to a support group if necessary.
d. expectation of heavy bleeding for at least 2 weeks.
e. emphasizing the need for rest.

A

ANS: A, C, E

The woman should be advised to consume a diet high in iron and protein. For many women iron supplementation is also necessary. Acknowledge that the patient has experienced a loss, albeit early. She can be taught to expect mood swings and possibly depression. Referral to a support group, clergy, or professional counseling may be necessary. Discharge teaching should emphasize the need for rest. Nothing should be placed in the vagina for 2 weeks after the procedure. This includes tampons and vaginal intercourse. The purpose of this recommendation is to prevent infection. Should infection occur, antibiotics may be prescribed. The patient should expect a scant, dark discharge for 1 to 2 weeks. Should heavy, profuse, or bright bleeding occur, she should be instructed to contact her provider.

How well did you know this?
1
Not at all
2
3
4
5
Perfectly
74
Q

The reported incidence of ectopic pregnancy in the United States has risen steadily over the past two decades. Causes include the increase in STDs accompanied by tubal infection and damage. The popularity of contraceptive devices such as the IUD has also increased the risk for ectopic pregnancy. The nurse who suspects that a patient has early signs of ectopic pregnancy should be observing her for symptoms such as: (Select all that apply.)

a. pelvic pain.
b. abdominal pain.
c. unanticipated heavy bleeding.
d. vaginal spotting or light bleeding.
e. missed period.

A

ANS: A, B, D, E
A missed period or spotting can easily be mistaken by the patient as early signs of pregnancy. More subtle signs depend on exactly where the implantation occurs. The nurse must be thorough in her assessment because pain is not a normal symptom of early pregnancy. As the fallopian tube tears open and the embryo is expelled, the patient often exhibits severe pain accompanied by intra-abdominal hemorrhage. This may progress to hypovolemic shock with minimal or even no external bleeding. In about half of women, shoulder and neck pain results from irritation of the diaphragm from the hemorrhage.

How well did you know this?
1
Not at all
2
3
4
5
Perfectly
75
Q

In planning for home care of a woman with preterm labor, which concern must the nurse address?

a. Nursing assessments will be different from those done in the hospital setting.
b. Restricted activity and medications will be necessary to prevent recurrence of preterm labor.
c. Prolonged bed rest may cause negative physiologic effects.
d. Home health care providers will be necessary.

A

ANS: C
Prolonged bed rest may cause adverse effects such as weight loss, loss of appetite, muscle wasting, weakness, bone demineralization, decreased cardiac output, risk for thrombophlebitis, alteration in bowel functions, sleep disturbance, and prolonged after birth recovery. Nursing assessments will differ somewhat from those performed in the acute care setting, but this is not the concern that needs to be addressed. Restricted activity and medication may prevent preterm labor, but not in all women. In addition, the plan of care is individualized to meet the needs of each woman. Many women will receive home health nurse visits, but care is individualized for each woman.

How well did you know this?
1
Not at all
2
3
4
5
Perfectly
76
Q

The nurse providing care for a woman with preterm labor who is receiving terbutaline would include which intervention to identify side effects of the drug?

a. Assessing deep tendon reflexes (DTRs)
b. Assessing for chest discomfort and palpitations
c. Assessing for bradycardia
d. Assessing for hypoglycemia

A

ANS: B
Terbutaline is a 2-adrenergic agonist that affects the cardiopulmonary and metabolic systems of the mother. Signs of cardiopulmonary decompensation would include chest pain and palpitations. Assessing DTRs would not address these concerns. 2-Adrenergic agonist drugs cause tachycardia, not bradycardia. The metabolic effect leads to hyperglycemia, not hypoglycemia.

How well did you know this?
1
Not at all
2
3
4
5
Perfectly
77
Q

In evaluating the effectiveness of magnesium sulfate for the treatment of preterm labor, what finding would alert the nurse to possible side effects?

a. Urine output of 160 mL in 4 hours
b. Deep tendon reflexes 2+ and no clonus
c. Respiratory rate of 16 breaths/min
d. Serum magnesium level of 10 mg/dL

A

ANS: D
The therapeutic range for magnesium sulfate management is 5 to 8 mg/dL. A serum magnesium level of 10 mg/dL could lead to signs and symptoms of magnesium toxicity, including oliguria and respiratory distress. Urine output of 160 mL in 4 hours, deep tendon reflexes 2+ with no clonus, and respiratory rate of 16 breaths/min are normal findings.

How well did you know this?
1
Not at all
2
3
4
5
Perfectly
78
Q

A woman in preterm labor at 30 weeks of gestation receives two 12-mg doses of betamethasone intramuscularly. The purpose of this pharmacologic treatment is to:

a. stimulate fetal surfactant production.
b. reduce maternal and fetal tachycardia associated with ritodrine administration.
c. suppress uterine contractions.
d. maintain adequate maternal respiratory effort and ventilation during magnesium sulfate therapy.

A

ANS: A
Antenatal glucocorticoids given as intramuscular injections to the mother accelerate fetal lung maturity. Inderal would be given to reduce the effects of ritodrine administration. Betamethasone has no effect on uterine contractions. Calcium gluconate would be given to reverse the respiratory depressive effects of magnesium sulfate therapy.

How well did you know this?
1
Not at all
2
3
4
5
Perfectly
79
Q

A woman at 26 weeks of gestation is being assessed to determine whether she is experiencing preterm labor. What finding indicates that preterm labor is occurring?

a. Estriol is not found in maternal saliva.
b. Irregular, mild uterine contractions are occurring every 12 to 15 minutes.
c. Fetal fibronectin is present in vaginal secretions.
d. The cervix is effacing and dilated to 2 cm.

A

ANS: D
Cervical changes such as shortened endocervical length, effacement, and dilation are predictors of imminent preterm labor. Changes in the cervix accompanied by regular contractions indicate labor at any gestation. Estriol is a form of estrogen produced by the fetus that is present in plasma at 9 weeks of gestation. Levels of salivary estriol have been shown to increase before preterm birth. Irregular, mild contractions that do not cause cervical change are not considered a threat. The presence of fetal fibronectin in vaginal secretions between 24 and 36 weeks of gestation could predict preterm labor, but it has only a 20% to 40% positive predictive value. Of more importance are other physiologic clues of preterm labor such as cervical changes.

How well did you know this?
1
Not at all
2
3
4
5
Perfectly
80
Q

A primigravida at 40 weeks of gestation is having uterine contractions every 1.5 to 2 minutes and says that they are very painful. Her cervix is dilated 2 cm and has not changed in 3 hours. The woman is crying and wants an epidural. What is the likely status of this woman’s labor?

a. She is exhibiting hypotonic uterine dysfunction.
b. She is experiencing a normal latent stage.
c. She is exhibiting hypertonic uterine dysfunction.
d. She is experiencing pelvic dystocia.

A

ANS: C
Women who experience hypertonic uterine dysfunction, or primary dysfunctional labor, often are anxious first-time mothers who are having painful and frequent contractions that are ineffective at causing cervical dilation or effacement to progress. With hypotonic uterine dysfunction, the woman initially makes normal progress into the active stage of labor; then the contractions become weak and inefficient or stop altogether. The contraction pattern seen in this woman signifies hypertonic uterine activity. Typically uterine activity in this phase occurs at 4- to 5-minute intervals lasting 30 to 45 seconds. Pelvic dystocia can occur whenever contractures of the pelvic diameters reduce the capacity of the bony pelvis, including the inlet, midpelvis, outlet, or any combination of these planes

How well did you know this?
1
Not at all
2
3
4
5
Perfectly
81
Q

Which assessment is least likely to be associated with a breech presentation?

a. Meconium-stained amniotic fluid
b. Fetal heart tones heard at or above the maternal umbilicus
c. Preterm labor and birth
d. Postterm gestation

A

ANS: D
Postterm gestation is not likely to be seen with a breech presentation. The presence of meconium in a breech presentation may result from pressure on the fetal wall as it traverses the birth canal. Fetal heart tones heard at the level of the umbilical level of the mother are a typical finding in a breech presentation because the fetal back would be located in the upper abdominal area. Breech presentations often occur in preterm births.

How well did you know this?
1
Not at all
2
3
4
5
Perfectly
82
Q

A woman is having her first child. She has been in labor for 15 hours. Two hours ago her vaginal examination revealed the cervix to be dilated to 5 cm and 100% effaced, and the presenting part was at station 0. Five minutes ago her vaginal examination indicated that there had been no change. What abnormal labor pattern is associated with this description?

a. Prolonged latent phase
b. Protracted active phase
c. Arrest of active phase
d. Protracted descent

A

ANS: C
With an arrest of the active phase, the progress of labor has stopped. This patient has not had any anticipated cervical change, thus indicating an arrest of labor. In the nulliparous woman a prolonged latent phase typically would last more than 20 hours. A protracted active phase, the first or second stage of labor, would be prolonged (slow dilation). With protracted descent, the fetus would fail to descend at an anticipated rate during the deceleration phase and second stage of labor.

How well did you know this?
1
Not at all
2
3
4
5
Perfectly
83
Q

In evaluating the effectiveness of oxytocin induction, the nurse would expect:

a. contractions lasting 80 to 90 seconds, 2 to 3 minutes apart.
b. the intensity of contractions to be at least 110 to 130 mm Hg.
c. labor to progress at least 2 cm/hr dilation.
d. At least 30 mU/min of oxytocin will be needed to achieve cervical dilation.

A

ANS: A
The goal of induction of labor would be to produce contractions that occur every 2 to 3 minutes and last 60 to 90 seconds. The intensity of the contractions should be 80 to 90 mm Hg by intrauterine pressure catheter. Cervical dilation of 1 cm/hr in the active phase of labor would be the goal in an oxytocin induction. The dose is increased by 1 to 2 mU/min at intervals of 30 to 60 minutes until the desired contraction pattern is achieved. Doses are increased up to a maximum of 20 to 40 mU/min.

How well did you know this?
1
Not at all
2
3
4
5
Perfectly
84
Q

A pregnant woman’s amniotic membranes rupture. Prolapsed umbilical cord is suspected. What intervention would be the top priority?

a. Placing the woman in the knee-chest position.
b. Covering the cord in sterile gauze soaked in saline.
c. Preparing the woman for a cesarean birth.
d. Starting oxygen by face mask.

A

ANS: A
The woman is assisted into a position (e.g., modified Sims position, Trendelenburg position, or the knee-chest position) in which gravity keeps the pressure of the presenting part off the cord. Although covering the cord in sterile gauze soaked saline, preparing the woman for a cesarean, and starting oxygen by face mark are appropriate nursing interventions in the event of a prolapsed cord, the intervention of top priority would be positioning the mother to relieve cord compression.

How well did you know this?
1
Not at all
2
3
4
5
Perfectly
85
Q

Prepidil (prostaglandin gel) has been ordered for a pregnant woman at 43 weeks of gestation. The nurse recognizes that this medication will be administered to:

a. enhance uteroplacental perfusion in an aging placenta.
b. increase amniotic fluid volume.
c. ripen the cervix in preparation for labor induction.
d. stimulate the amniotic membranes to rupture.

A

ANS: C
It is accurate to state that Prepidil will be administered to ripen the cervix in preparation for labor induction. It is not administered to enhance uteroplacental perfusion in an aging placenta, increase amniotic fluid volume, or stimulate the amniotic membranes to rupture.

How well did you know this?
1
Not at all
2
3
4
5
Perfectly
86
Q

The nurse, caring for a patient whose labor is being augmented with oxytocin, recognizes that the oxytocin should be discontinued immediately if there is evidence of:

a. uterine contractions occurring every 8 to 10 minutes.
b. a fetal heart rate (FHR) of 180 with absence of variability.
c. the patient’s needing to void.
d. rupture of the patient’s amniotic membranes.

A

ANS: B
This FHR is nonreassuring. The oxytocin should be discontinued immediately, and the physician should be notified. The oxytocin should be discontinued if uterine hyperstimulation occurs. Uterine contractions that are occurring every 8 to 10 minutes do not qualify as hyperstimulation. The patient’s needing to void is not an indication to discontinue the oxytocin induction immediately or to call the physician. Unless a change occurs in the FHR pattern that is nonreassuring or the patient experiences uterine hyperstimulation, the oxytocin does not need to be discontinued. The physician should be notified that the patient’s membranes have ruptured.

How well did you know this?
1
Not at all
2
3
4
5
Perfectly
87
Q

Nurses should know some basic definitions concerning preterm birth, preterm labor, and low birth weight. For instance:

a. the terms preterm birth and low birth weight can be used interchangeably.
b. preterm labor is defined as cervical changes and uterine contractions occurring between 20 and 37 weeks of pregnancy.
c. low birth weight is anything below 3.7 lbs.
d. in the United States early in this century, preterm birth accounted for 18% to 20% of all births.

A

ANS: B
Before 20 weeks, it is not viable (miscarriage); after 37 weeks, it can be considered term. Although these terms are used interchangeably, they have different meanings: preterm birth describes the length of gestation (37 weeks) regardless of weight; low birth weight describes weight only (2500 g or less) at the time of birth, whenever it occurs. Low birth weight is anything less than 2500 g, or about 5.5 lbs. In 2003 the preterm birth rate in the United States was 12.3%, but it is increasing in frequency.

How well did you know this?
1
Not at all
2
3
4
5
Perfectly
88
Q

With regard to the care management of preterm labor, nurses should be aware that:

a. all women must be considered at risk for preterm labor and prediction is so hit-and-miss, teaching pregnant women the symptoms probably causes more harm through false alarms.
b. Braxton Hicks contractions often signal the onset of preterm labor.
c. preterm labor is likely to be the start of an extended labor, a woman with symptoms can wait several hours before contacting the primary caregiver.
d. the diagnosis of preterm labor is based on gestational age, uterine activity, and progressive cervical change.

A

ANS: D
Gestational age of 20 to 37 weeks, uterine contractions, and a cervix that is 80% effaced or dilated 2 cm indicates preterm labor. It is essential that nurses teach women how to detect the early symptoms of preterm labor. Braxton Hicks contractions resemble preterm labor contractions, but they are not true labor. Waiting too long to see a health care provider could result in not administering essential medications. Preterm labor is not necessarily long-term labor.

How well did you know this?
1
Not at all
2
3
4
5
Perfectly
89
Q

As relates to the use of tocolytic therapy to suppress uterine activity, nurses should be aware that:

a. the drugs can be given efficaciously up to the designated beginning of term at 37 weeks.
b. there are no important maternal (as opposed to fetal) contraindications.
c. its most important function is to afford the opportunity to administer antenatal glucocorticoids.
d. if the patient develops pulmonary edema while receiving tocolytics, intravenous (IV) fluids should be given.

A

ANS: C
Buying time for antenatal glucocorticoids to accelerate fetal lung development may be the best reason to use tocolytics. Once the pregnancy has reached 34 weeks, the risks of tocolytic therapy outweigh the benefits. There are important maternal contraindications to tocolytic therapy. Tocolytic-induced edema can be caused by IV fluids.

How well did you know this?
1
Not at all
2
3
4
5
Perfectly
90
Q

With regard to dysfunctional labor, nurses should be aware that:

a. women who are underweight are more at risk.
b. women experiencing precipitous labor are about the only “dysfunctionals” not to be exhausted.
c. hypertonic uterine dysfunction is more common than hypotonic dysfunction.
d. abnormal labor patterns are most common in older women.

A

ANS: B
Precipitous labor lasts less than 3 hours. Short women more than 30 lbs overweight are more at risk for dysfunctional labor. Hypotonic uterine dysfunction, in which the contractions become weaker, is more common. Abnormal labor patterns are more common in women less than 20 years of age.

How well did you know this?
1
Not at all
2
3
4
5
Perfectly
91
Q

The least common cause of long, difficult, or abnormal labor (dystocia) is:

a. midplane contracture of the pelvis.
b. compromised bearing-down efforts as a result of pain medication.
c. disproportion of the pelvis.
d. low-lying placenta.

A

ANS: C

The least common cause of dystocia is disproportion of the pelvis.

How well did you know this?
1
Not at all
2
3
4
5
Perfectly
92
Q

Nurses should be aware that the induction of labor:

a. can be achieved by external and internal version techniques.
b. is also known as a trial of labor (TOL).
c. is almost always done for medical reasons.
d. is rated for viability by a Bishop score.

A

ANS: D
Induction of labor is likely to be more successful with a Bishop score of 9 or higher for first-time mothers and 5 or higher for veterans. Version is turning of the fetus to a better position by a physician for an easier or safer birth. A trial of labor is the observance of a woman and her fetus for several hours of active labor to assess the safety of vaginal birth. Two thirds of cases of induced labor are elective and are not done for medical reasons.

How well did you know this?
1
Not at all
2
3
4
5
Perfectly
93
Q

While caring for the patient who requires an induction of labor, the nurse should be cognizant that:

a. ripening the cervix usually results in a decreased success rate for induction.
b. labor sometimes can be induced with balloon catheters or laminaria tents.
c. oxytocin is less expensive than prostaglandins and more effective but creates greater health risks.
d. amniotomy can be used to make the cervix more favorable for labor.

A

ANS: B
Balloon catheters or laminaria tents are mechanical means of ripening the cervix. Ripening the cervix, making it softer and thinner, increases the success rate of induced labor. Prostaglandin E1 is less expensive and more effective than oxytocin but carries a greater risk. Amniotomy is the artificial rupture of membranes, which is used to induce labor only when the cervix is already ripe.

How well did you know this?
1
Not at all
2
3
4
5
Perfectly
94
Q

With regard to the process of augmentation of labor, the nurse should be aware that it:

a. is part of the active management of labor that is instituted when the labor process is unsatisfactory.
b. relies on more invasive methods when oxytocin and amniotomy have failed.
c. is a modern management term to cover up the negative connotations of forceps-assisted birth.
d. uses vacuum cups.

A

ANS: A
Augmentation is part of the active management of labor that stimulates uterine contractions after labor has started but is not progressing satisfactorily. Augmentation uses amniotomy and oxytocin infusion, as well as some gentler, noninvasive methods. Forceps-assisted births and vacuum-assisted births are appropriately used at the end of labor and are not part of augmentation.

How well did you know this?
1
Not at all
2
3
4
5
Perfectly
95
Q

The exact cause of preterm labor is unknown and believed to be multifactorial. Infection is thought to be a major factor in many preterm labors. Select the type of infection that has not been linked to preterm births.

a. Viral
b. Periodontal
c. Cervical
d. Urinary tract

A

ANS: A
The infections that increase the risk of preterm labor and birth are all bacterial. They include cervical, urinary tract, periodontal, and other bacterial infections. Therefore, it is important for the patient to participate in early, continual, and comprehensive prenatal care. Evidence has shown a link between periodontal infections and preterm labor. Researchers recommend regular dental care before and during pregnancy, oral assessment as a routine part of prenatal care, and scrupulous oral hygiene to prevent infection. Cervical infections of a bacterial nature have been linked to preterm labor and birth. The presence of urinary tract infections increases the risk of preterm labor and birth.

How well did you know this?
1
Not at all
2
3
4
5
Perfectly
96
Q

The standard of care for obstetrics dictates that an internal version may be used to manipulate the:

a. fetus from a breech to a cephalic presentation before labor begins.
b. fetus from a transverse lie to a longitudinal lie before cesarean birth.
c. second twin from an oblique lie to a transverse lie before labor begins.
d. second twin from a transverse lie to a breech presentation during vaginal birth.

A

ANS: D
Internal version is used only during vaginal birth to manipulate the second twin into a presentation that allows it to be born vaginally. For internal version to occur, the cervix needs to be completely dilated.

How well did you know this?
1
Not at all
2
3
4
5
Perfectly
97
Q

The nurse practicing in a labor setting knows that the woman most at risk for uterine rupture is:

a. a gravida 3 who has had two low-segment transverse cesarean births.
b. a gravida 2 who had a low-segment vertical incision for delivery of a 10-lb infant.
c. a gravida 5 who had two vaginal births and two cesarean births.
d. a gravida 4 who has had all cesarean births.

A

ANS: D
The risk of uterine rupture increases for the patient who has had multiple prior births with no vaginal births. As the number of prior uterine incisions increases, so does the risk for uterine rupture. Low-segment transverse cesarean scars do not predispose the patient to uterine rupture.

How well did you know this?
1
Not at all
2
3
4
5
Perfectly
98
Q

Before the physician performs an external version, the nurse should expect an order for a:

a. tocolytic drug.
b. contraction stress test (CST).
c. local anesthetic.
d. Foley catheter.

A

ANS: A
A tocolytic drug will relax the uterus before and during version, thus making manipulation easier. CST is used to determine the fetal response to stress. A local anesthetic is not used with external version. The bladder should be emptied; however, catheterization is not necessary.

How well did you know this?
1
Not at all
2
3
4
5
Perfectly
99
Q

A maternal indication for the use of forceps is:

a. a wide pelvic outlet.
b. maternal exhaustion.
c. a history of rapid deliveries.
d. failure to progress past 0 station.

A

ANS: B
A mother who is exhausted may be unable to assist with the expulsion of the fetus.
The patient with a wide pelvic outlet will likely not require vacuum extraction. With a rapid delivery, vacuum extraction is not necessary. A station of 0 is too high for a vacuum extraction.

How well did you know this?
1
Not at all
2
3
4
5
Perfectly
100
Q

The priority nursing intervention after an amniotomy should be to:

a. assess the color of the amniotic fluid.
b. change the patient’s gown.
c. estimate the amount of amniotic fluid.
d. assess the fetal heart rate.

A

ANS: D
The fetal heart rate must be assessed immediately after the rupture of the membranes to determine whether cord prolapse or compression has occurred. Secondary to FHR assessment, amniotic fluid amount, color, odor, and consistency is assessed. Dry clothing is important for patient comfort; however, it is not the top priority.

How well did you know this?
1
Not at all
2
3
4
5
Perfectly
101
Q

The priority nursing care associated with an oxytocin (Pitocin) infusion is:

a. measuring urinary output.
b. increasing infusion rate every 30 minutes.
c. monitoring uterine response.
d. evaluating cervical dilation.

A

ANS: C
Because of the risk of hyperstimulation, which could result in decreased placental perfusion and uterine rupture, the nurse’s priority intervention is monitoring uterine response. Monitoring urinary output is also important; however, it is not the top priority during the administration of Pitocin. The infusion rate may be increased after proper assessment that it is an appropriate interval to do so. Monitoring labor progression is the standard of care for all labor patients.

How well did you know this?
1
Not at all
2
3
4
5
Perfectly
102
Q

Immediately after the forceps-assisted birth of an infant, the nurse should:

a. assess the infant for signs of trauma.
b. give the infant prophylactic antibiotics.
c. apply a cold pack to the infant’s scalp.
d. measure the circumference of the infant’s head.

A

ANS: A
The infant should be assessed for bruising or abrasions at the site of application, facial palsy, and subdural hematoma. Prophylactic antibiotics are not necessary with a forceps delivery. A cold pack would put the infant at risk for cold stress and is contraindicated. Measuring the circumference of the head is part of the initial nursing assessment.

How well did you know this?
1
Not at all
2
3
4
5
Perfectly
103
Q

Surgical, medical, or mechanical methods may be used for labor induction. Which technique is considered a mechanical method of induction?

a. Amniotomy
b. Intravenous Pitocin
c. Transcervical catheter
d. Vaginal insertion of prostaglandins

A

ANS: C
Placement of a balloon-tipped Foley catheter into the cervix is a mechanical method of induction. Other methods to expand and gradually dilate the cervix include hydroscopic dilators such as laminaria tents (made from desiccated seaweed), or Lamicel (contains magnesium sulfate). Amniotomy is a surgical method of augmentation and induction.
Intravenous Pitocin and insertion of prostaglandins are medical methods of induction.

How well did you know this?
1
Not at all
2
3
4
5
Perfectly
104
Q

Complications and risks associated with cesarean births include: (Select all that apply.)

a. placental abruption.
b. wound dehiscence.
c. hemorrhage.
d. urinary tract infections.
e. fetal injuries.

A

ANS: B, C, D, E
Placental abruption and placenta previa are both indications for cesarean birth and are not complications thereof. Wound dehiscence, hemorrhage, urinary tract infection, and fetal injuries are all possible complications and risks associated with delivery by cesarean section.

How well did you know this?
1
Not at all
2
3
4
5
Perfectly
105
Q

Induction of labor is considered an acceptable obstetric procedure if it is in the best interest to deliver the fetus. The charge nurse in the labor and delivery unit is often asked to schedule patients for this procedure and therefore must be cognizant of the specific conditions appropriate for labor induction. These include: (Select all that apply.)

a. rupture of membranes at or near term.
b. convenience of the woman or her physician.
c. chorioamnionitis (inflammation of the amniotic sac).
d. postterm pregnancy.
e. fetal death.

A

ANS: A, C, D, E
These are all acceptable indications for induction. Other conditions include intrauterine growth retardation (IUGR), maternal-fetal blood incompatibility, hypertension, and placental abruption. Elective inductions for the convenience of the woman or her provider are not recommended; however, they have become commonplace. Factors such as rapid labors and living a long distance from a health care facility may be valid reasons in such a circumstance. Elective delivery should not occur before 39 weeks’ completed gestation.

How well did you know this?
1
Not at all
2
3
4
5
Perfectly
106
Q

A woman gave birth to an infant boy 10 hours ago. Where would the nurse expect to locate this woman’s fundus?

a. One centimeter above the umbilicus
b. Two centimeters below the umbilicus
c. Midway between the umbilicus and the symphysis pubis
d. Nonpalpable abdominally

A

ANS: A
Within 12 hours after delivery the fundus may be approximately 1 cm above the umbilicus. The fundus descends about 1 to 2 cm every 24 hours. Within 12 hours after delivery the fundus may be approximately 1 cm above the umbilicus. By the sixth after birth week the fundus normally is halfway between the symphysis pubis and the umbilicus. The fundus should be easily palpated using the maternal umbilicus as a reference point.

How well did you know this?
1
Not at all
2
3
4
5
Perfectly
107
Q

Which woman is most likely to experience strong afterpains?

a. A woman who experienced oligohydramnios
b. A woman who is a gravida 4, para 4-0-0-4
c. A woman who is bottle-feeding her infant
d. A woman whose infant weighed 5 lbs, 3 ounces

A

ANS: B
Afterpains are more common in multiparous women. Afterpains are more noticeable with births in which the uterus was greatly distended, as in a woman who experienced polyhydramnios or a woman who delivered a large infant. Breastfeeding may cause afterpains to intensify.

How well did you know this?
1
Not at all
2
3
4
5
Perfectly
108
Q

A woman gave birth to a healthy infant boy 5 days ago. What type of lochia would the nurse expect to find when assessing this woman?

a. Lochia rubra
b. Lochia sangra
c. Lochia alba
d. Lochia serosa

A

ANS: D
Lochia serosa, which consists of blood, serum, leukocytes, and tissue debris, generally occurs around day 3 or 4 after childbirth. Lochia rubra consists of blood and decidual and trophoblastic debris. The flow generally lasts 3 to 4 days and pales, becoming pink or brown. There is no such term as lochia sangra. Lochia alba occurs in most women after day 10 and can continue up to 6 weeks after childbirth.

How well did you know this?
1
Not at all
2
3
4
5
Perfectly
109
Q

Which hormone remains elevated in the immediate after birth period of the breastfeeding woman?

a. Estrogen
b. Progesterone
c. Prolactin
d. Human placental lactogen

A

ANS: C
Prolactin levels in the blood increase progressively throughout pregnancy. In women who breastfeed, prolactin levels remain elevated into the sixth week after birth. Estrogen and progesterone levels decrease markedly after expulsion of the placenta and reach their lowest levels 1 week into the after birth period. Human placental lactogen levels decrease dramatically after expulsion of the placenta.

How well did you know this?
1
Not at all
2
3
4
5
Perfectly
110
Q

Two days ago a woman gave birth to a full-term infant. Last night she awakened several times to urinate and noted that her gown and bedding were wet from profuse diaphoresis. One mechanism for the diaphoresis and diuresis that this woman is experiencing during the early after birth period is:

a. elevated temperature caused by after birth infection.
b. increased basal metabolic rate after giving birth.
c. loss of increased blood volume associated with pregnancy.
d. increased venous pressure in the lower extremities.

A

ANS: C
Within 12 hours of birth women begin to lose the excess tissue fluid that has accumulated during pregnancy. One mechanism for reducing these retained fluids is the profuse diaphoresis that often occurs, especially at night, for the first 2 or 3 days after childbirth. Postpartal diuresis is another mechanism by which the body rids itself of excess fluid.
An elevated temperature would cause chills and may cause dehydration, not diaphoresis and diuresis. Diaphoresis and diuresis sometimes are referred to as reversal of the water metabolism of pregnancy, not as the basal metabolic rate. Postpartal diuresis may be caused by the removal of increased venous pressure in the lower extremities.

How well did you know this?
1
Not at all
2
3
4
5
Perfectly
111
Q

A woman gave birth to a 7-lb, 3-ounce infant boy 2 hours ago. The nurse determines that the woman’s bladder is distended because her fundus is now 3 cm above the umbilicus and to the right of the midline. In the immediate after birth period, the most serious consequence likely to occur from bladder distention is:

a. urinary tract infection.
b. excessive uterine bleeding.
c. a ruptured bladder.
d. bladder wall atony.

A

ANS: B
Excessive bleeding can occur immediately after birth if the bladder becomes distended because it pushes the uterus up and to the side and prevents it from contracting firmly. A urinary tract infection may result from overdistention of the bladder, but it is not the most serious consequence. A ruptured bladder may result from a severely overdistended bladder. However, vaginal bleeding most likely would occur before the bladder reaches this level of overdistention. Bladder distention may result from bladder wall atony. The most serious concern associated with bladder distention is excessive uterine bleeding.

How well did you know this?
1
Not at all
2
3
4
5
Perfectly
112
Q

The nurse caring for the after birth woman understands that breast engorgement is caused by:

a. overproduction of colostrum.
b. accumulation of milk in the lactiferous ducts.
c. hyperplasia of mammary tissue.
d. congestion of veins and lymphatics.

A

ANS: D
Breast engorgement is caused by the temporary congestion of veins and lymphatics, not by overproduction of colostrum, overproduction of milk, or hyperplasia of mammary tissue.

How well did you know this?
1
Not at all
2
3
4
5
Perfectly
113
Q

A woman gave birth to a 7-lb, 6-ounce infant girl 1 hour ago. The birth was vaginal, and the estimated blood loss (EBL) was approximately 1500 mL. When assessing the woman’s vital signs, the nurse would be concerned to see:

a. temperature 37.9° C, heart rate 120, respirations 20, blood pressure (BP) 90/50.
b. temperature 37.4° C, heart rate 88, respirations 36, BP 126/68.
c. temperature 38° C, heart rate 80, respirations 16, BP 110/80.
d. temperature 36.8° C, heart rate 60, respirations 18, BP 140/90.

A

ANS: A
An EBL of 1500 mL with tachycardia and hypotension suggests hypovolemia caused by excessive blood loss. An increased respiratory rate of 36 may be secondary to pain from the birth. Temperature may increase to 38° C during the first 24 hours as a result of the dehydrating effects of labor. A BP of 140/90 is slightly elevated, which may be caused by the use of oxytocic medications.

How well did you know this?
1
Not at all
2
3
4
5
Perfectly
114
Q

Which statement by a newly delivered woman indicates that she knows what to expect about her menstrual activity after childbirth?

a. “My first menstrual cycle will be lighter than normal and then will get heavier every month thereafter.”
b. “My first menstrual cycle will be heavier than normal and will return to my prepregnant volume within three or four cycles.”
c. “I will not have a menstrual cycle for 6 months after childbirth.”
d. “My first menstrual cycle will be heavier than normal and then will be light for several months after.”

A

ANS: B
“My first menstrual cycle will be heavier than normal and will return to my prepregnant volume within three or four cycles” is an accurate statement and indicates her understanding of her expected menstrual activity. She can expect her first menstrual cycle to be heavier than normal (which occurs by 3 months after childbirth), and the volume of her subsequent cycles will return to prepregnant levels within three or four cycles.

How well did you know this?
1
Not at all
2
3
4
5
Perfectly
115
Q

The interval between the birth of the newborn and the return of the reproductive organs to their normal nonpregnant state is called the:

a. involutionary period because of what happens to the uterus.
b. lochia period because of the nature of the vaginal discharge.
c. mini-tri period because it lasts only 3 to 6 weeks.
d. puerperium, or fourth trimester of pregnancy.

A

ANS: D
The puerperium, also called the fourth trimester or the after birth period of pregnancy, lasts about 3 to 6 weeks. Involution marks the end of the puerperium, or the fourth trimester of pregnancy. Lochia refers to the various vaginal discharges during the puerperium, or fourth trimester of pregnancy.

How well did you know this?
1
Not at all
2
3
4
5
Perfectly
116
Q

The self-destruction of excess hypertrophied tissue in the uterus is called:

a. autolysis.
b. subinvolution.
c. afterpain.
d. diastasis.

A

ANS: A
Autolysis is caused by a decrease in hormone levels. Subinvolution is failure of the uterus to return to a nonpregnant state. Afterpain is caused by uterine cramps 2 to 3 days after birth. Diastasis refers to the separation of muscles.

How well did you know this?
1
Not at all
2
3
4
5
Perfectly
117
Q

With regard to the after birth uterus, nurses should be aware that:

a. at the end of the third stage of labor it weighs approximately 500 g.
b. after 2 weeks after birth it should not be palpable abdominally.
c. after 2 weeks after birth it weighs 100 g.
d. it returns to its original (prepregnancy) size by 6 weeks after birth.

A

ANS: B
After 2 weeks after birth, the uterus should not be palpable abdominally; however, it has not yet returned to its original size. At the end of the third stage of labor, the uterus weighs approximately 1000 g. It takes 6 full weeks for the uterus to return to its original size. After 2 weeks after birth the uterus weighs about 350 g, not its original size. The normal self-destruction of excess hypertrophied tissue accounts for the slight increase in uterine size after each pregnancy.

How well did you know this?
1
Not at all
2
3
4
5
Perfectly
118
Q

With regard to after birth pains, nurses should be aware that these pains are:

a. caused by mild, continuous contractions for the duration of the after birth period.
b. more common in first-time mothers.
c. more noticeable in births in which the uterus was overdistended.
d. alleviated somewhat when the mother breastfeeds.

A

ANS: C
A large baby or multiple babies overdistend the uterus. The cramping that causes after birth pains arises from periodic, vigorous contractions and relaxations, which persist through the first part of the after birth period. After birth pains are more common in multiparous women because first-time mothers have better uterine tone. Breastfeeding intensifies after birth pain because it stimulates contractions.

How well did you know this?
1
Not at all
2
3
4
5
Perfectly
119
Q

Post birth uterine/vaginal discharge, called lochia:

a. is similar to a light menstrual period for the first 6 to 12 hours.
b. is usually greater after cesarean births.
c. will usually decrease with ambulation and breastfeeding.
d. should smell like normal menstrual flow unless an infection is present.

A

ANS: D
An offensive odor usually indicates an infection. Lochia flow should approximate a heavy menstrual period for the first 2 hours and then steadily decrease. Less lochia usually is seen after cesarean births and usually increases with ambulation and breastfeeding.

How well did you know this?
1
Not at all
2
3
4
5
Perfectly
120
Q

With regard to after birth ovarian function, nurses should be aware that:

a. almost 75% of women who do not breastfeed resume menstruating within a month after birth.
b. ovulation occurs slightly earlier for breastfeeding women.
c. because of menstruation/ovulation schedules, contraception considerations can be postponed until after the puerperium.
d. the first menstrual flow after childbirth usually is heavier than normal.

A

ANS: D
The first flow is heavier, but within three or four cycles, it is back to normal. Ovulation can occur within the first month, but for 70% of nonlactating women, it returns within 12 weeks after birth. Breastfeeding women take longer to resume ovulation. Because many women ovulate before their first after birth menstrual period, contraceptive options need to be discussed early in the puerperium.

How well did you know this?
1
Not at all
2
3
4
5
Perfectly
121
Q

As relates to the condition and reconditioning of the urinary system after childbirth, nurses should be aware that:

a. kidney function returns to normal a few days after birth.
b. diastasis recti abdominis is a common condition that alters the voiding reflex.
c. fluid loss through perspiration and increased urinary output accounts for a weight loss of more than 2 kg during the puerperium.
d. with adequate emptying of the bladder, bladder tone usually is restored 2 to 3 weeks after childbirth.

A

ANS: C
Excess fluid loss through other means occurs as well. Kidney function usually returns to normal in about a month. Diastasis recti abdominis is the separation of muscles in the abdominal wall; it has no effect on the voiding reflex. Bladder tone usually is restored 5 to 7 days after childbirth.

How well did you know this?
1
Not at all
2
3
4
5
Perfectly
122
Q

Knowing that the condition of the new mother’s breasts will be affected by whether she is breastfeeding, nurses should be able to tell their patients all the following statements except:

a. breast tenderness is likely to persist for about a week after the start of lactation.
b. as lactation is established, a mass may form that can be distinguished from cancer by its position shift from day to day.
c. in nonlactating mothers colostrum is present for the first few days after childbirth.
d. if suckling is never begun (or is discontinued), lactation ceases within a few days to a week.

A

ANS: A
Breast tenderness should persist for 24 to 48 hours after lactation begins. That movable, noncancerous mass is a filled milk sac. Colostrum is present for a few days whether the mother breastfeeds or not. A mother who does not want to breastfeed should also avoid stimulating her nipples.

How well did you know this?
1
Not at all
2
3
4
5
Perfectly
123
Q

With regard to the after birth changes and developments in a woman’s cardiovascular system, nurses should be aware that:

a. cardiac output, the pulse rate, and stroke volume all return to prepregnancy normal values within a few hours of childbirth.
b. respiratory function returns to nonpregnant levels by 6 to 8 weeks after birth.
c. the lowered white blood cell count after pregnancy can lead to false-positive results on tests for infections.
d. a hypercoagulable state protects the new mother from thromboembolism, especially after a cesarean birth.

A

ANS: B
Respirations should decrease to within the woman’s normal prepregnancy range by 6 to 8 weeks after birth. Stroke volume increases, and cardiac output remains high for a couple of days. However, the heart rate and blood pressure return to normal quickly. Leukocytosis increases 10 to 12 days after childbirth and can obscure the diagnosis of acute infections (false-negative results). The hypercoagulable state increases the risk of thromboembolism, especially after a cesarean birth.

How well did you know this?
1
Not at all
2
3
4
5
Perfectly
124
Q

Which condition, not uncommon in pregnancy, is likely to require careful medical assessment during the puerperium?

a. Varicosities of the legs
b. Carpal tunnel syndrome
c. Periodic numbness and tingling of the fingers
d. Headaches

A

ANS: D
Headaches in the after birth period can have a number of causes, some of which deserve medical attention. Total or nearly total regression of varicosities is expected after childbirth. Carpal tunnel syndrome is relieved in childbirth when the compression on the median nerve is lessened. Periodic numbness of the fingers usually disappears after birth unless carrying the baby aggravates the condition.

How well did you know this?
1
Not at all
2
3
4
5
Perfectly
125
Q

Several changes in the integumentary system that appear during pregnancy disappear after birth, although not always completely. What change is almost certain to be completely reversed?

a. Nail brittleness
b. Darker pigmentation of the areolae and linea nigra
c. Striae gravidarum on the breasts, abdomen, and thighs
d. Spider nevi

A

ANS: A
The nails return to their prepregnancy consistency and strength. Some women have permanent darker pigmentation of the areolae and linea nigra. Striae gravidarum (stretch marks) usually do not completely disappear. For some women spider nevi persist indefinitely.

How well did you know this?
1
Not at all
2
3
4
5
Perfectly
126
Q

Childbirth may result in injuries to the vagina and uterus. Pelvic floor exercises also known as Kegel exercises will help to strengthen the perineal muscles and encourage healing. The nurse knows that the patient understands the correct process for completing these conditioning exercises when she reports:

a. “I contract my thighs, buttocks, and abdomen.”
b. “I do 10 of these exercises every day.”
c. “I stand while practicing this new exercise routine.”
d. “I pretend that I am trying to stop the flow of urine midstream.”

A

ANS: D
The woman can pretend that she is attempting to stop the passing of gas or the flow of urine midstream. This will replicate the sensation of the muscles drawing upward and inward. Each contraction should be as intense as possible without contracting the abdomen, buttocks, or thighs. Guidelines suggest that these exercises should be done 24 to 100 times per day. Positive results are shown with a minimum of 24 to 45 repetitions per day. The best position to learn Kegel exercises is to lie supine with knees bent. A secondary position is on the hands and knees.

How well did you know this?
1
Not at all
2
3
4
5
Perfectly
127
Q

Which maternal event is abnormal in the early after birth period?

a. Diuresis and diaphoresis
b. Flatulence and constipation
c. Extreme hunger and thirst
d. Lochial color changes from rubra to alba

A

ANS: D
For the first 3 days after childbirth, lochia is termed rubra. Lochia serosa follows, and then at about 11 days, the discharge becomes clear, colorless, or white. Diuresis and diaphoresis are the methods by which the body rids itself of increased plasma volume. Urine output of 3000 mL/day is common for the first few days after delivery and is facilitated by hormonal changes in the mother. Bowel tone remains sluggish for days. Many women anticipate pain during defecation and are unwilling to exert pressure on the perineum. The new mother is hungry because of energy used in labor and thirsty because of fluid restrictions during labor.

How well did you know this?
1
Not at all
2
3
4
5
Perfectly
128
Q

Which finding 12 hours after birth requires further assessment?

a. The fundus is palpable two fingerbreadths above the umbilicus.
b. The fundus is palpable at the level of the umbilicus.
c. The fundus is palpable one fingerbreadth below the umbilicus.
d. The fundus is palpable two fingerbreadths below the umbilicus.

A

ANS: A
The fundus rises to the umbilicus after delivery and remains there for about 24 hours. A fundus that is above the umbilicus may indicate uterine atony or urinary retention. A fundus that is palpable at or below the level of the umbilicus is a normal finding for a patient who is 12 hours after birth. Palpation of the fundus 2 fingerbreadths below the umbilicus is an unusual finding for 12 hours after birth; however, it is still appropriate.

How well did you know this?
1
Not at all
2
3
4
5
Perfectly
129
Q

If the patient’s white blood cell (WBC) count is 25,000/mm on her second after birth day, the nurse should:

a. tell the physician immediately.
b. have the laboratory draw blood for reanalysis.
c. recognize that this is an acceptable range at this point after birth.
d. begin antibiotic therapy immediately.

A

ANS: C
During the first 10 to 12 days after childbirth, values between 20,000 and 25,000/mm are common. Because this is a normal finding there is no reason to alert the physician. There is no need for reassessment or antibiotics because it is expected for the WBCs to be elevated.

How well did you know this?
1
Not at all
2
3
4
5
Perfectly
130
Q

Which documentation on a woman’s chart on after birth day 14 indicates a normal involution process?

a. Moderate bright red lochial flow
b. Breasts firm and tender
c. Fundus below the symphysis and not palpable
d. Episiotomy slightly red and puffy

A

ANS: C
The fundus descends 1 cm/day, so by after birth day 14 it is no longer palpable. The lochia should be changed by this day to serosa. Breasts are not part of the involution process. The episiotomy should not be red or puffy at this stage.

How well did you know this?
1
Not at all
2
3
4
5
Perfectly
131
Q

Changes in blood volume after childbirth depend on several factors such as blood loss during childbirth and the amount of extravascular water (physiologic edema) mobilized and excreted. A after birth nurse anticipates blood loss of: (Select all that apply.)

a. 100 mL.
b. 250 mL or less.
c. 300 to 500 mL.
d. 500 to 1000 mL.
e. 1500 mL or greater.

A

ANS: C, D
The average blood loss for a vaginal birth of a single fetus ranges from 300 to 500 mL (10% of blood volume). The typical blood loss for women who gave birth by cesarean is 500 to 1000 mL (15% to 30% of blood volume). During the first few days after birth the plasma volume decreases further as a result diuresis. Pregnancy-induced hypervolemia (an increase in blood volume of at least 35%) allows most women to tolerate considerable blood loss during childbirth.

How well did you know this?
1
Not at all
2
3
4
5
Perfectly
132
Q

A 25-year-old gravida 2, para 2-0-0-2 gave birth 4 hours ago to a 9-lb, 7-ounce boy after augmentation of labor with Pitocin. She puts on her call light and asks for her nurse right away, stating, “I’m bleeding a lot.” The most likely cause of after birth hemorrhage in this woman is:

a. retained placental fragments.
b. unrepaired vaginal lacerations.
c. uterine atony.
d. puerperal infection.

A

ANS: C
This woman gave birth to a macrosomic boy after Pitocin augmentation. The most likely cause of bleeding 4 hours after delivery, combined with these risk factors, is uterine atony. Although retained placental fragments may cause after birth hemorrhage, this typically would be detected in the first hour after delivery of the placenta and is not the most likely cause of hemorrhage in this woman. Although unrepaired vaginal lacerations may cause bleeding, they typically would occur in the period immediately after birth. Puerperal infection can cause subinvolution and subsequent bleeding; however, this typically would be detected 24 hours after delivery.

How well did you know this?
1
Not at all
2
3
4
5
Perfectly
133
Q

On examining a woman who gave birth 5 hours ago, the nurse finds that the woman has completely saturated a perineal pad within 15 minutes. The nurse’s first action is to:

a. begin an intravenous (IV) infusion of Ringer’s lactate solution.
b. assess the woman’s vital signs.
c. call the woman’s primary health care provider.
d. massage the woman’s fundus.

A

ANS: D
The nurse should assess the uterus for atony. Uterine tone must be established to prevent excessive blood loss. The nurse may begin an IV infusion to restore circulatory volume, but this would not be the first action. Blood pressure is not a reliable indicator of impending shock from impending hemorrhage; assessing vital signs should not be the nurse’s first action. The physician would be notified after the nurse completes the assessment of the woman.

How well did you know this?
1
Not at all
2
3
4
5
Perfectly
134
Q

A woman gave birth vaginally to a 9-lb, 12-ounce girl yesterday. Her primary health care provider has written orders for perineal ice packs, use of a sitz bath tid, and a stool softener. What information is most closely correlated with these orders?

a. The woman is a gravida 2, para 2.
b. The woman had a vacuum-assisted birth.
c. The woman received epidural anesthesia.
d. The woman has an episiotomy.

A

ANS: D
These orders are typical interventions for a woman who has had an episiotomy, lacerations, and hemorrhoids. A multiparous classification is not an indication for these orders. A vacuum-assisted birth may be used in conjunction with an episiotomy, which would indicate these interventions. Use of epidural anesthesia has no correlation with these orders.

How well did you know this?
1
Not at all
2
3
4
5
Perfectly
135
Q

The laboratory results for a after birth woman are as follows: blood type, A; Rh status, positive; rubella titer, 1:8 (EIA 0.8); hematocrit, 30%. How would the nurse best interpret these data?

a. Rubella vaccine should be given.
b. A blood transfusion is necessary.
c. Rh immune globulin is necessary within 72 hours of birth.
d. A Kleihauer-Betke test should be performed.

A

ANS: A
This patient’s rubella titer indicates that she is not immune and that she needs to receive a vaccine. These data do not indicate that the patient needs a blood transfusion. Rh immune globulin is indicated only if the patient has a negative Rh status and the infant has a positive Rh status. A Kleihauer-Betke test should be performed if a large fetomaternal transfusion is suspected, especially if the mother is Rh negative. The data do not provide any indication for performing this test

How well did you know this?
1
Not at all
2
3
4
5
Perfectly
136
Q

A woman gave birth 48 hours ago to a healthy infant girl. She has decided to bottle-feed. During your assessment you notice that both of her breasts are swollen, warm, and tender on palpation. The woman should be advised that this condition can best be treated by:

a. running warm water on her breasts during a shower.
b. applying ice to the breasts for comfort.
c. expressing small amounts of milk from the breasts to relieve pressure.
d. wearing a loose-fitting bra to prevent nipple irritation.

A

ANS: B
Applying ice to the breasts for comfort is appropriate for treating engorgement in a mother who is bottle-feeding. This woman is experiencing engorgement, which can be treated by using ice packs (because she is not breastfeeding) and cabbage leaves. A bottle-feeding mother should avoid any breast stimulation, including pumping or expressing milk. A bottle-feeding mother should wear a well-fitted support bra or breast binder continuously for at least the first 72 hours after giving birth. A loose-fitting bra will not aid lactation suppression. Furthermore, the shifting of the bra against the breasts may stimulate the nipples and thereby stimulate lactation.

How well did you know this?
1
Not at all
2
3
4
5
Perfectly
137
Q

A 25-year-old multiparous woman gave birth to an infant boy 1 day ago. Today her husband brings a large container of brown seaweed soup to the hospital. When the nurse enters the room, the husband asks for help with warming the soup so that his wife can eat it. The nurse’s most appropriate response is to ask the woman:

a. “Didn’t you like your lunch?”
b. “Does your doctor know that you are planning to eat that?”
c. “What is that anyway?”
d. “I’ll warm the soup in the microwave for you.”

A

ANS: D
“I’ll warm the soup in the microwave for you” shows cultural sensitivity to the dietary preferences of the woman and is the most appropriate response. Cultural dietary preferences must be respected. Women may request that family members bring favorite or culturally appropriate foods to the hospital. “What is that anyway?” does not show cultural sensitivity.

How well did you know this?
1
Not at all
2
3
4
5
Perfectly
138
Q

In many hospitals new mothers are routinely presented with gift bags containing samples of infant formula. This practice:

a. is inconsistent with the Baby-Friendly Hospital Initiative.
b. promotes longer periods of breastfeeding.
c. is perceived as supportive to both bottle-feeding and breastfeeding mothers.
d. is associated with earlier cessation of breastfeeding.

A

ANS: A
Infant formula should not be given to mothers who are breastfeeding. Such gifts are associated with earlier cessation of breastfeeding. Baby-Friendly USA prohibits the distribution of any gift bags or formula to new mothers.

How well did you know this?
1
Not at all
2
3
4
5
Perfectly
139
Q

A after birth woman overhears the nurse tell the obstetrics clinician that she has a positive Homans’ sign and asks what it means. The nurse’s best response is:

a. “You have pitting edema in your ankles.”
b. “You have deep tendon reflexes rated 2+.”
c. “You have calf pain when the nurse flexes your foot.”
d. “You have a ‘fleshy’ odor to your vaginal drainage.”

A

ANS: C
Discomfort in the calf with sharp dorsiflexion of the foot may indicate deep vein thrombosis. Edema is within normal limits for the first few days until the excess interstitial fluid is remobilized and excreted. Deep tendon reflexes should be 1+ to 2+. A “fleshy” odor, not a foul odor, is within normal limits.

How well did you know this?
1
Not at all
2
3
4
5
Perfectly
140
Q

In the recovery room, if a woman is asked either to raise her legs (knees extended) off the bed or to flex her knees, place her feet flat on the bed, and raise her buttocks well off the bed, most likely she is being tested to see whether she:

a. has recovered from epidural or spinal anesthesia.
b. has hidden bleeding underneath her.
c. has regained some flexibility.
d. is a candidate to go home after 6 hours.

A

ANS: A
If the numb or prickly sensations are gone from her legs after these movements, she has likely recovered from the epidural or spinal anesthesia.

How well did you know this?
1
Not at all
2
3
4
5
Perfectly
141
Q

Under the Newborns’ and Mothers’ Health Protection Act, all health plans are required to allow new mothers and newborns to remain in the hospital for a minimum of _____ hours after a normal vaginal birth and for _____ hours after a cesarean birth.

a. 24, 73
b. 24, 96
c. 48, 96
d. 48, 120

A

ANS: C
The specified stays are 48 hours (2 days) for a vaginal birth and 96 hours (4 days) for a cesarean birth. The attending provider and the mother together can decide on an earlier discharge.

How well did you know this?
1
Not at all
2
3
4
5
Perfectly
142
Q

In a variation of rooming-in, called couplet care, the mother and infant share a room, and the mother shares the care of the infant with:

a. the father of the infant.
b. her mother (the infant’s grandmother).
c. her eldest daughter (the infant’s sister).
d. the nurse.

A

ANS: D
In couplet care the mother shares a room with the newborn and shares infant care with a nurse educated in maternity and infant care.

How well did you know this?
1
Not at all
2
3
4
5
Perfectly
143
Q

Nursing care in the fourth trimester includes an important intervention sometimes referred to as taking the time to mother the mother. Specifically this expression refers to:

a. formally initializing individualized care by confirming the woman’s and infant’s identification (ID) numbers on their respective wrist bands. (“This is your baby.”)
b. teaching the mother to check the identity of any person who comes to remove the baby from the room. (“It’s a dangerous world out there.”)
c. including other family members in the teaching of self-care and child care. (“We’re all in this together.”)
d. nurturing the woman by providing encouragement and support as she takes on the many tasks of motherhood.

A

ANS: D
Many professionals believe that the nurse’s nurturing and support function is more important than providing physical care and teaching. Matching ID wrist bands is more of a formality, but it is also a get-acquainted procedure. “Mothering the mother” is more a process of encouraging and supporting the woman in her new role. Having the mother check IDs is a security measure for protecting the baby from abduction. Teaching the whole family is just good nursing practice.

How well did you know this?
1
Not at all
2
3
4
5
Perfectly
144
Q

Excessive blood loss after childbirth can have several causes; the most common is:

a. vaginal or vulvar hematomas.
b. unrepaired lacerations of the vagina or cervix.
c. failure of the uterine muscle to contract firmly.
d. retained placental fragments.

A

ANS: C
Uterine atony can best be thwarted by maintaining good uterine tone and preventing bladder distention. Although vaginal or vulvar hematomas, unpaired lacerations of the vagina or cervix, and retained placental fragments are possible causes of excessive blood loss, uterine muscle failure (uterine atony) is the most common cause.

How well did you know this?
1
Not at all
2
3
4
5
Perfectly
145
Q

A hospital has a number of different perineal pads available for use. A nurse is observed soaking several of them and writing down what she sees. This activity indicates that the nurse is trying to:

a. improve the accuracy of blood loss estimation, which usually is a subjective assessment.
b. determine which pad is best.
c. demonstrate that other nurses usually underestimate blood loss.
d. reveal to the nurse supervisor that one of them needs some time off.

A

ANS: A
Saturation of perineal pads is a critical indicator of excessive blood loss, and anything done to aid in assessment is valuable. The nurse is noting the saturation volumes and soaking appearances. It is possible that the nurse is trying to determine which pad is best, but it is more likely that the nurse is noting saturation volumes and soaking appearances to improve the accuracy of blood loss estimation. Nurses usually overestimate blood loss, if anything.

How well did you know this?
1
Not at all
2
3
4
5
Perfectly
146
Q

Because a full bladder prevents the uterus from contracting normally, nurses intervene to help the woman empty her bladder spontaneously as soon as possible. If all else fails, the last thing the nurse could try is:

a. pouring water from a squeeze bottle over the woman’s perineum.
b. placing oil of peppermint in a bedpan under the woman.
c. asking the physician to prescribe analgesics.
d. inserting a sterile catheter.

A

ANS: D
Invasive procedures usually are the last to be tried, especially with so many other simple and easy methods available (e.g., water, peppermint vapors, pain medication). Pouring water over the perineum may stimulate voiding. It is easy, noninvasive, and should be tried early. The oil of peppermint releases vapors that may relax the necessary muscles. If the woman is anticipating pain from voiding, pain medications may be helpful. Other nonmedical means and pain medication should be tried before insertion of a catheter.

How well did you know this?
1
Not at all
2
3
4
5
Perfectly
147
Q

If a woman is at risk for thrombus and is not ready to ambulate, nurses may intervene by performing a number of interventions. Which intervention should the nurse avoid?

a. Putting the patient in antiembolic stockings (TED hose) and/or sequential compression device (SCD) boots.
b. Having the patient flex, extend, and rotate her feet, ankles, and legs.
c. Having the patient sit in a chair.
d. Notifying the physician immediately if a positive Homans’ sign occurs.

A

ANS: C
Sitting immobile in a chair will not help. Bed exercise and prophylactic footwear may. TED hose and SCD boots are recommended. Bed exercises, such as flexing, extending, and rotating her feet, ankles, and legs, are useful. A positive Homans’ sign (calf muscle pain or warmth, redness, or tenderness) requires the physician’s immediate attention.

How well did you know this?
1
Not at all
2
3
4
5
Perfectly
148
Q

As relates to rubella and Rh issues, nurses should be aware that:

a. breastfeeding mothers cannot be vaccinated with the live attenuated rubella virus.
b. women should be warned that the rubella vaccination is teratogenic, and that they must avoid pregnancy for 1 month after vaccination.
c. Rh immune globulin is safely administered intravenously because it cannot harm a nursing infant.
d. Rh immune globulin boosts the immune system and thereby enhances the effectiveness of vaccinations.

A

ANS: B
Women should understand they must practice contraception for 1 month after being vaccinated. Because the live attenuated rubella virus is not communicable in breast milk, breastfeeding mothers can be vaccinated. Rh immune globulin is administered intramuscularly; it should never be given to an infant. Rh immune globulin suppresses the immune system and therefore could thwart the rubella vaccination.

How well did you know this?
1
Not at all
2
3
4
5
Perfectly
149
Q

Discharge instruction, or teaching the woman what she needs to know to care for herself and her newborn, officially begins:

a. at the time of admission to the nurse’s unit.
b. when the infant is presented to the mother at birth.
c. during the first visit with the physician in the unit.
d. when the take-home information packet is given to the couple.

A

ANS: A
Discharge planning, the teaching of maternal and newborn care, begins on the woman’s admission to the unit, continues throughout her stay, and actually never ends as long as she has contact with medical personnel.

How well did you know this?
1
Not at all
2
3
4
5
Perfectly
150
Q

A recently delivered mother and her baby are at the clinic for a 6-week after birth checkup. The nurse should be concerned that psychosocial outcomes are not being met if the woman:

a. discusses her labor and birth experience excessively.
b. believes that her baby is more attractive and clever than any others.
c. has not given the baby a name.
d. has a partner or family members who react very positively about the baby.

A

ANS: C
If the mother is having difficulty naming her new infant, it may be a signal that she is not adapting well to parenthood. Other red flags include refusal to hold or feed the baby, lack of interaction with the infant, and becoming upset when the baby vomits or needs a diaper change. A new mother who is having difficulty would be unwilling to discuss her labor and birth experience. An appropriate nursing diagnosis could be Impaired parenting related to a long, difficult labor, or unmet expectations of birth. A mother who is willing to discuss her birth experience is making a healthy personal adjustment. The mother who is not coping well would find her baby unattractive and messy. She may also be overly disappointed in the baby’s sex. The patient may voice concern that the baby reminds her of a family member whom she does not like. Having a partner and/or other family members react positively is an indication that this new mother has a good support system in place. This support system will help reduce anxiety related to her new role as a mother.

How well did you know this?
1
Not at all
2
3
4
5
Perfectly
151
Q

Postpartal overdistention of the bladder and urinary retention can lead to which complications?

a. After birth hemorrhage and eclampsia
b. Fever and increased blood pressure
c. After birth hemorrhage and urinary tract infection
d. Urinary tract infection and uterine rupture

A

ANS: C
Incomplete emptying and overdistention of the bladder can lead to urinary tract infection. Overdistention of the bladder displaces the uterus and prevents contraction of the uterine muscle, thus leading to after birth hemorrhage. There is no correlation between bladder distention and high blood pressure or eclampsia. The risk of uterine rupture decreases after the birth of the infant.

How well did you know this?
1
Not at all
2
3
4
5
Perfectly
152
Q

Rho immune globulin will be ordered after birth if which situation occurs?

a. Mother Rh–, baby Rh+
b. Mother Rh–, baby Rh–
c. Mother Rh+, baby Rh+
d. Mother Rh+, baby Rh–

A

ANS: A
An Rh– mother delivering an Rh+ baby may develop antibodies to fetal cells that entered her bloodstream when the placenta separated. The Rho immune globulin works to destroy the fetal cells in the maternal circulation before sensitization occurs. If mother and baby are both Rh+ or Rh– the blood types are alike, so no antibody formation would be anticipated. If the Rh+ blood of the mother comes in contact with the Rh– blood of the infant, no antibodies would develop because the antigens are in the mother’s blood, not the infant’s.

How well did you know this?
1
Not at all
2
3
4
5
Perfectly
153
Q

Which nursing action is most appropriate to correct a boggy uterus that is displaced above and to the right of the umbilicus?

a. Notify the physician of an impending hemorrhage.
b. Assess the blood pressure and pulse.
c. Evaluate the lochia.
d. Assist the patient in emptying her bladder.

A

ANS: D
Urinary retention may cause overdistention of the urinary bladder, which lifts and displaces the uterus. Nursing actions need to be implemented before notifying the physician. It is important to evaluate blood pressure, pulse, and lochia if the bleeding continues; however, the focus at this point in time is to assist the patient in emptying her bladder.

154
Q

When caring for a newly delivered woman, the nurse is aware that the best measure to prevent abdominal distention after a cesarean birth is:

a. rectal suppositories.
b. early and frequent ambulation.
c. tightening and relaxing abdominal muscles.
d. carbonated beverages.

A

ANS: B
Activity will aid the movement of accumulated gas in the gastrointestinal tract. Rectal suppositories can be helpful after distention occurs; however, they do not prevent it. Ambulation is the best prevention. Carbonated beverages may increase distention.

155
Q

The nurse caring for the after birth woman understands that breast engorgement is caused by:

a. overproduction of colostrum.
b. accumulation of milk in the lactiferous ducts and glands.
c. hyperplasia of mammary tissue.
d. congestion of veins and lymphatics.

A

ANS: D
Breast engorgement is caused by the temporary congestion of veins and lymphatics. Breast engorgement is not the result of overproduction of colostrum. Accumulation of milk in the lactiferous ducts and glands does not cause breast engorgement. Hyperplasia of mammary tissue does not cause breast engorgement.

156
Q

After giving birth to a healthy infant boy, a primiparous woman, 16 years old, is admitted to the after birth unit. An appropriate nursing diagnosis for her at this time is risk for impaired parenting related to deficient knowledge of newborn care. In planning for the woman’s discharge, what should the nurse be certain to include in the plan of care?

a. Instruct the patient how to feed and bathe her infant.
b. Give the patient written information on bathing her infant.
c. Advise the patient that all mothers instinctively know how to care for their infants.
d. Provide time for the patient to bathe her infant after she views an infant bath demonstration.

A

ANS: D
Having the mother demonstrate infant care is a valuable method of assessing the patient’s understanding of her newly acquired knowledge, especially in this age group, because she may inadvertently neglect her child. Although verbalizing how to care for the infant is a form of patient education, it is not the most developmentally appropriate teaching for a teenage mother. Advising the patient that all mothers instinctively know how to care for their infants is an inappropriate statement; it is belittling and false.

157
Q

The nurse observes several interactions between a after birth woman and her new son. What behavior, if exhibited by this woman, would the nurse identify as a possible maladaptive behavior regarding parent-infant attachment?

a. Talks and coos to her son
b. Seldom makes eye contact with her son
c. Cuddles her son close to her
d. Tells visitors how well her son is feeding

A

ANS: B
The woman should be encouraged to hold her infant in the en face position and make eye contact with the infant. Normal infant-parent interactions include talking and cooing to her son, cuddling her son close to her, and telling visitors how well her son is feeding.

158
Q

The nurse observes that a 15-year-old mother seems to ignore her newborn. A strategy that the nurse can use to facilitate mother-infant attachment in this mother is to:

a. tell the mother she must pay attention to her infant.
b. show the mother how the infant initiates interaction and pays attention to her.
c. demonstrate for the mother different positions for holding her infant while feeding.
d. arrange for the mother to watch a video on parent-infant interaction.

A

ANS: B
Pointing out the responsiveness of the infant is a positive strategy for facilitating parent-infant attachment. Telling the mother that she must pay attention to her infant may be perceived as derogatory and is not appropriate. Educating the young mother in infant care is important; however, pointing out the responsiveness of her baby is a better tool for facilitating mother-infant attachment. Videos are an educational tool that can demonstrate parent-infant attachment, but encouraging the mother to recognize the infant’s responsiveness is more appropriate.

159
Q

The nurse hears a primiparous woman talking to her son and telling him that his chin is just like his dad’s chin. This woman’s statement reflects:

a. mutuality.
b. synchrony.
c. claiming.
d. reciprocity.

A

ANS: C
Claiming refers to the process by which the child is identified in terms of likeness to other family members. Mutuality occurs when the infant’s behaviors and characteristics call forth a corresponding set of maternal behaviors and characteristics. Synchrony refers to the “fit” between the infant’s cues and the parent’s responses. Reciprocity is a type of body movement or behavior that provides the observer with cues.

160
Q

New parents express concern that, because of the mother’s emergency cesarean birth under general anesthesia, they did not have the opportunity to hold and bond with their daughter immediately after her birth. The nurse’s response should convey to the parents that:

a. attachment, or bonding, is a process that occurs over time and does not require early contact.
b. the time immediately after birth is a critical period for people.
c. early contact is essential for optimum parent-infant relationships.
d. they should just be happy that the infant is healthy.

A

ANS: A
Attachment, or bonding, is a process that occurs over time and does not require early contact. The formerly accepted definition of bonding held that the period immediately after birth was a critical time for bonding to occur. Research since has indicated that parent-infant attachment occurs over time. A delay does not inhibit the process. Parent-infant attachment involves activities such as touching, holding, and gazing; it is not exclusively eye contact. A response that conveys that the parents should just be happy that the infant is healthy is inappropriate because it is derogatory and belittling.

161
Q

During a phone follow-up conversation with a woman who is 4 days’ after birth, the woman tells the nurse, “I don’t know what’s wrong. I love my son, but I feel so let down. I seem to cry for no reason!” The nurse would recognize that the woman is experiencing:

a. taking-in.
b. postpartum depression (PPD).
c. postpartum (PP) blues.
d. attachment difficulty.

A

ANS: C
During the PP blues women are emotionally labile, often crying easily and for no apparent reason. This lability seems to peak around the fifth PP day. The taking-in phase is the period after birth when the mother focuses on her own psychologic needs. Typically this period lasts 24 hours. PPD is an intense, pervasive sadness marked by severe, labile mood swings; it is more serious and persistent than the PP blues. Crying is not a maladaptive attachment response; it indicates PP blues.

162
Q

The nurse can help a father in his transition to parenthood by:

a. pointing out that the infant turned at the sound of his voice.
b. encouraging him to go home to get some sleep.
c. telling him to tape the infant’s diaper a different way.
d. suggesting that he let the infant sleep in the bassinet.

A

ANS: A
Infants respond to the sound of voices. Because attachment involves a reciprocal interchange, observing the interaction between parent and infant is very important. Separation of the parent and infant does not encourage parent-infant attachment. Educating the parent in infant care techniques is important; however, the manner in which a diaper is taped is not relevant and does not enhance parent-infant interactions. Parent-infant attachment involves touching, holding, and cuddling. It is appropriate for a father to want to hold the infant as the baby sleeps.

163
Q

The nurse notes that a Vietnamese woman does not cuddle or interact with her newborn other than to feed him, change his diapers or soiled clothes, and put him to bed. In evaluating the woman’s behavior with her infant, the nurse realizes that:

a. what appears to be a lack of interest in the newborn is in fact the Vietnamese way of demonstrating intense love by attempting to ward off evil spirits.
b. the woman is inexperienced in caring for newborns.
c. the woman needs a referral to a social worker for further evaluation of her parenting behaviors once she goes home with the newborn.
d. extra time needs to be planned for assisting the woman in bonding with her newborn.

A

ANS: A
The nurse may observe a Vietnamese woman who gives minimal care to her infant and refuses to cuddle or interact with her infant. The apparent lack of interest in the newborn is this cultural group’s attempt to ward off evil spirits and actually reflects an intense love and concern for the infant. It is important to educate the woman in infant care, but it is equally important to acknowledge her cultural beliefs and practices.

164
Q

Many first-time parents do not plan on their parents’ help immediately after the newborn arrives. What statement by the nurse is the most appropriate when counseling new parents about the involvement of grandparents?

a. “You should tell your parents to leave you alone.”
b. “Grandparents can help you with parenting skills and also help preserve family traditions.”
c. “Grandparent involvement can be very disruptive to the family.”
d. “They are getting old. You should let them be involved while they can.”

A

ANS: B
“Grandparents can help you with parenting skills and also help preserve family traditions” is the most appropriate response. Intergenerational help may be perceived as interference; however, a statement of this sort is not therapeutic to the adaptation of the family. Not only is “Grandparent involvement can be very disruptive to the family” invalid, it also is not an appropriate nursing response. Regardless of age, grandparents can help with parenting skills and preserve family traditions. Talking about the age of the grandparents is not the most appropriate statement, and it does not demonstrate sensitivity on the part of the nurse.

165
Q

When the infant’s behaviors and characteristics call forth a corresponding set of maternal behaviors and characteristics, this is called:

a. mutuality.
b. bonding.
c. claiming.
d. acquaintance.

A

ANS: A
Mutuality extends the concept of attachment to include this shared set of behaviors. Bonding is the process over time of parents forming an emotional attachment to their infant. Mutuality refers to a shared set of behaviors that is a part of the bonding process. Claiming is the process by which parents identify their new baby in terms of likeness to other family members and their differences and uniqueness. Like mutuality, acquaintance is part of attachment. It describes how parents get to know their baby during the immediate after birth period through eye contact, touching, and talking.

166
Q

In follow-up appointments or visits with parents and their new baby, it may be useful if the nurse can identify parental behaviors that can either facilitate or inhibit attachment. Which one is a facilitating behavior?

a. The parents have difficulty naming the infant.
b. The parents hover around the infant, directing attention to and pointing at the infant.
c. The parents make no effort to interpret the actions or needs of the infant.
d. The parents do not move from fingertip touch to palmar contact and holding.

A

ANS: B
Hovering over the infant and obviously paying attention to the baby are facilitating behaviors. Inhibiting behaviors include difficulty naming the infant, making no effort to interpret the actions or needs of the infant, and not moving from fingertip touch to palmar contact and holding.

167
Q

With regard to parents’ early and extended contact with their infant and the relationships built, nurses should be aware that:

a. immediate contact is essential for the parent-child relationship.
b. skin-to-skin contact is preferable to contact with the body totally wrapped in a blanket.
c. extended contact is especially important for adolescents and low-income women because they are at risk for parenting inadequacies.
d. mothers need to take precedence over their partners and other family matters.

A

ANS: C
Nurses should encourage any activity that optimizes family extended contact. Immediate contact facilitates the attachment process but is not essential; otherwise, adopted infants would not establish the affectionate ties they do. The mode of infant-mother contact does not appear to have any important effect. Mothers and their partners are considered equally important.

168
Q

In the United States the en face position is preferred immediately after birth. Nurses can facilitate this process by all of these actions except:

a. washing both the infant’s face and the mother’s face.
b. placing the infant on the mother’s abdomen or breast with their heads on the same plane.
c. dimming the lights.
d. delaying the instillation of prophylactic antibiotic ointment in the infant’s eyes.

A

ANS: A
To facilitate the position in which the parent’s and infant’s faces are approximately 8 inches apart on the same plane, allowing them to make eye contact, the nurse can place the infant at the proper height on the mother’s body, dim the light so that the infant’s eyes open, and delay putting ointment in the infant’s eyes.

169
Q

Other early sensual contacts between infant and mother involve sound and smell. Nurses should be aware that, despite what folk wisdom may say:

a. high-pitched voices irritate newborns.
b. infants can learn to distinguish their mother’s voice from others soon after birth.
c. all babies in the hospital smell alike.
d. a mother’s breast milk has no distinctive odor.

A

ANS: B
Infants know the sound of their mother’s voice early. Infants respond positively to high-pitched voices. Each infant has a unique odor. Infants quickly learn to distinguish the odor of their mother’s breast milk.

170
Q

After they are born, a crying infant may be soothed by being held in a position in which the newborn can hear the mother’s heartbeat. This phenomenon is known as:

a. entrainment.
b. reciprocity.
c. synchrony.
d. biorhythmicity.

A

ANS: D
The newborn is in rhythm with the mother. The infant develops a personal biorhythm with the parents’ help over time. Entrainment is the movement of newborns in time to the structure of adult speech. Reciprocity is body movement or behavior that gives cues to the person’s desires. These take several weeks to develop with a new baby. Synchrony is the fit between the infant’s behavioral cues and the parent’s responses.

171
Q

Of the many factors that influence parental responses, nurses should be conscious of negative stereotypes that apply to specific patient populations. Which response could be an inappropriate stereotype of adolescent mothers?

a. An adolescent mother’s egocentricity and unmet developmental needs interfere with her ability to parent effectively.
b. An adolescent mother is likely to use less verbal instruction, be less responsive, and interact less positively than other mothers.
c. Adolescent mothers have a higher documented incidence of child abuse.
d. Mothers older than 35 often deal with more stress related to work and career issues and decreasing libido.

A

ANS: C
Adolescent mothers are more inclined to have a number of parenting difficulties that benefit from counseling; however, a higher incidence of child abuse is not one of them. Midlife mothers have many competencies, but they are more likely to have to deal with career issues and the accompanying stress.

172
Q

When working with parents who have some form of sensory impairment, nurses should understand that ________ is an inaccurate statement.

a. “One of the major difficulties visually impaired parents experience is the skepticism of health care professionals.”
b. “Visually impaired mothers cannot overcome the infant’s need for eye-to-eye contact.”
c. “The best approach for the nurse is to assess the parents’ capabilities rather than focusing on their disabilities.”
d. “Technologic advances, including the Internet, can provide deaf parents with a full range of parenting activities and information.”

A

ANS: B
Other sensory output can be provided by the parent, other people can participate, and other coping devices can be used. The skepticism, open or hidden, of health care professionals places an additional and unneeded hurdle for the parents. After the parents’ capabilities have been assessed (including some the nurse may not have expected), the nurse can help find ways to assist the parents that play to their strengths. The Internet affords an extra teaching tool for the deaf, as do videos with subtitles or nurses signing. A number of electronic devices can turn sound into light flashes to help pick up a child’s cry. Sign language is readily acquired by young children.

173
Q

With regard to the adaptation of other family members, mainly siblings and grandparents, to the newborn, nurses should be aware that:

a. sibling rivalry cannot be dismissed as overblown psychobabble; negative feelings and behaviors can take a long time to blow over.
b. participation in preparation classes helps both siblings and grandparents.
c. in the United States paternal and maternal grandparents consider themselves of equal importance and status.
d. in the past few decades the number of grandparents providing permanent care to their grandchildren has been declining.

A

ANS: B
Preparing older siblings and grandparents helps everyone to adapt. Sibling rivalry should be expected initially, but the negative behaviors associated with it have been overemphasized and stop in a comparatively short time. In the United States, in contrast to other cultures, paternal grandparents frequently consider themselves secondary to maternal grandparents. The number of grandparents providing permanent child care has been on the increase.

174
Q

Nursing activities that promote parent-infant attachment are many and varied. One activity that should not be overlooked is management of the environment. While providing routine mother-baby care, the nurse should ensure that:

a. the baby is able to return to the nursery at night so that the new mother can sleep.
b. routine times for care are established to reassure the parents.
c. the father should be encouraged to go home at night to prepare for mother-baby discharge.
d. an environment that fosters as much privacy as possible should be created.

A

ANS: D
Care providers need to knock before gaining entry. Nursing care activities should be grouped. Once the baby has demonstrated adjustment to extrauterine life (either in the mother’s room or the transitional nursery), all care should be provided in one location. This important principle of family-centered maternity care fosters attachment by giving parents the opportunity to learn about their infant 24 hours a day. One nurse should provide care to both mother and baby in this couplet care or rooming-in model. It is not necessary for the baby to return to the nursery at night. In fact, the mother will sleep better with the infant close by. Care should be individualized to meet the parents’ needs, not the routines of the staff. Teaching goals should be developed in collaboration with the parents. The father, or other significant other, should be permitted to sleep in the room with the mother. The maternity unit should develop policies that allow for the presence of significant others as much as the new mother desires.

175
Q

The early after birth period is a time of emotional and physical vulnerability. Many mothers can easily become psychologically overwhelmed by the reality of their new parental responsibilities. Fatigue compounds these issues. Although the baby blues are a common occurrence in the after birth period, about one-half million women in America experience a more severe syndrome known as postpartum depression (PPD). Which statement regarding PPD is essential for the nurse to be aware of when attempting to formulate a nursing diagnosis?

a. PPD symptoms are consistently severe.
b. This syndrome affects only new mothers.
c. PPD can easily go undetected.
d. Only mental health professionals should teach new parents about this condition.

A

ANS: C
PPD can go undetected because parents do not voluntarily admit to this type of emotional distress out of embarrassment, fear, or guilt. PPD symptoms range from mild to severe, with women having both good and bad days. Both mothers and fathers should be screened. PPD may also affect new fathers. The nurse should include information on PPD and how to differentiate this from the baby blues for all patients on discharge. Nurses also can urge new parents to report symptoms and seek follow-up care promptly if symptoms occur.

176
Q

The mother-baby nurse is able to recognize reciprocal attachment behavior. This refers to:

a. the positive feedback an infant exhibits toward parents during the attachment process.
b. behavior during the sensitive period when the infant is in the quiet alert stage.
c. unidirectional behavior exhibited by the infant, initiated and enhanced by eye contact.
d. behavior by the infant during the sensitive period to elicit feelings of “falling in love” from the parents.

ANS: A
In this definition, “reciprocal” refers to the feedback from the infant during the attachment process. This is a good time for bonding; however, it does not define reciprocal attachment. Reciprocal attachment applies to feedback behavior and is not unidirectional.

A

ANS: A
In this definition, “reciprocal” refers to the feedback from the infant during the attachment process. This is a good time for bonding; however, it does not define reciprocal attachment. Reciprocal attachment applies to feedback behavior and is not unidirectional.

177
Q

The after birth woman who continually repeats the story of her labor, delivery, and recovery experience is:

a. providing others with her knowledge of events.
b. making the birth experience “real.”
c. taking hold of the events leading to her labor and delivery.
d. accepting her response to labor and delivery.

A

ANS: B
Reliving the birth experience makes the event real and helps the mother realize that the pregnancy is over and that the infant is born and is now a separate individual. The retelling of the story is to satisfy her needs, not the needs of others. This new mother is in the taking-in phase, trying to make the birth experience seem real and separate the infant from herself.

178
Q

On observing a woman on her first after birth day sitting in bed while her newborn lies awake in the bassinet, the nurse should:

a. realize that this situation is perfectly acceptable.
b. offer to hand the baby to the woman.
c. hand the baby to the woman.
d. explain “taking in” to the woman.

A

ANS: C
During the “taking-in” phase of maternal adaptation (the mother may be passive and dependent), the nurse should encourage bonding when the infant is in the quiet alert stage. This is done best by simply giving the baby to the mother. The patient is exhibiting expected behavior during the taking-in phase; however, interventions by the nurse can facilitate infant bonding. The patient will learn best during the taking-hold phase.

179
Q

A nurse is observing a family. The mother is holding the baby she delivered less than 24 hours ago. Her husband is watching his wife and asking questions about newborn care. The 4-year-old brother is punching his mother on the back. The nurse should:

a. report the incident to the social services department.
b. advise the parents that the toddler needs to be reprimanded.
c. report to oncoming staff that the mother is probably not a good disciplinarian.
d. realize that this is a normal family adjusting to family change.

A

ANS: D
The observed behaviors are normal variations of families adjusting to change. There is no need to report this one incident. Giving advice at this point would make the parents feel inadequate.

180
Q

The best way for the nurse to promote and support the maternal-infant bonding process is to:

a. help the mother identify her positive feelings toward the newborn.
b. encourage the mother to provide all newborn care.
c. assist the family with rooming-in.
d. return the newborn to the nursery during sleep periods.

A

ANS: C
Close and frequent interaction between mother and infant, which is facilitated by rooming-in, is important in the bonding process. This is often referred to as the mother-baby care or couplet care. Having the mother express her feelings is important; however, it is not the best way to promote bonding. The mother needs time to rest and recuperate; she should not be expected to do all of the care. The patient needs to observe the infant during all stages so she will be aware of what to anticipate when they go home.

181
Q

During which phase of maternal adjustment will the mother relinquish the baby of her fantasies and accept the real baby?

a. Letting go
b. Taking hold
c. Taking in
d. Taking on

A

ANS: A
Accepting the real infant and relinquishing the fantasy infant occurs during the letting-go phase of maternal adjustment. During the taking-hold phase the mother assumes responsibility for her own care and shifts her attention to the infant. In the taking-in phase the mother is primarily focused on her own needs. There is no taking-on phase of maternal adjustment.

182
Q

A 25-year-old gravida 1 para 1 who had an emergency cesarean birth 3 days ago is scheduled for discharge. As you prepare her for discharge, she begins to cry. Your initial action should be to:

a. assess her for pain.
b. point out how lucky she is to have a healthy baby.
c. explain that she is experiencing after birth blues.
d. allow her time to express her feelings.

A

ANS: D
Although many women experience transient after birth blues, they need assistance in expressing their feelings. This condition affects 50% to 80% of new mothers. There should be no assumption that the patient is in pain, when in fact she may have no pain whatsoever. This is “blocking” communication and inappropriate in this situation. The patient needs the opportunity to express her feelings first; patient teaching can occur later.

183
Q

A man calls the nurse’s station and states that his wife, who delivered 2 days ago, is happy one minute and crying the next. The man says, “She was never like this before the baby was born.” The nurse’s initial response could be to:

a. tell him to ignore the mood swings, as they will go away.
b. reassure him that this behavior is normal.
c. advise him to get immediate psychological help for her.
d. instruct him in the signs, symptoms, and duration of after birth blues.

A

ANS: B
Before providing further instructions, inform family members of the fact that after birth blues are a normal process. Telling her partner to “ignore the mood swings” does not encourage further communication and may belittle the husband’s concerns. After birth blues are usually short-lived; no medical intervention is needed. Patient teaching is important; however, the new father’s anxieties need to be allayed before he will be receptive to teaching.

184
Q

To promote bonding and attachment immediately after delivery, the most important nursing intervention is to:

a. allow the mother quiet time with her infant.
b. assist the mother in assuming an en face position with her newborn.
c. teach the mother about the concepts of bonding and attachment.
d. assist the mother in feeding her baby.

A

ANS: B
Assisting the mother in assuming an en face position with her newborn will support the bonding process. The mother should be given as much privacy as possible; however, nursing assessments must still be continued during this critical time. The mother has just delivered and is more focused on the infant; she will not be receptive to teaching at this time. This is a good time to initiate breastfeeding; however, the mother first needs time to explore the new infant and begin the bonding process.

185
Q

A new father states, “I know nothing about babies,” but he seems to be interested in learning. This is an ideal opportunity for the nurse to:

a. continue to observe his interaction with the newborn.
b. tell him when he does something wrong.
c. show no concern, as he will learn on his own.
d. include him in teaching sessions.

A

ANS: D
The nurse must be sensitive to the father’s needs and include him whenever possible. As fathers take on their new role, the nurse should praise every attempt, even if his early care is awkward. It is important to note the bonding process of the mother and the father; however, that does not satisfy the expressed needs of the father. The new father should be encouraged in caring for his baby by pointing out the things that he does right. Criticizing him will discourage him.

186
Q

Which concerns about parenthood are often expressed by visually impaired mothers? (Select all that apply.)

a. Infant safety
b. Transportation
c. The ability to care for the infant
d. Missing out visually
e. Needing extra time for parenting activities to accommodate the visual limitations

A

ANS: A, B, D, E
Concerns expressed by visually impaired mothers include infant safety, extra time needed for parenting activities, transportation, handling other people’s reactions, providing proper discipline, and missing out visually. Blind people sense reluctance on the part of others to acknowledge that they have a right to be parents; however, blind parents are fully capable of caring for their infants.

187
Q

A parent who has a hearing impairment is presented with a number of challenges in parenting. Which nursing approaches are appropriate for working with hearing-impaired new parents? (Select all that apply.)

a. Use devices that transform sound into light.
b. Assume that the patient knows sign language.
c. Speak quickly and loudly.
d. Ascertain whether the patient can read lips before teaching.
e. Written messages aid in communication.

A

ANS: A, D, E
Section 504 of the Rehabilitation Act of 1973 requires that hospitals use various communication techniques and resources with the deaf and hard of hearing patient. This includes devices such as door alarms, cry alarms, and amplifiers. Before initiating communication, the nurse needs to be aware of the parents’ preferences for communication. Not all hearing-impaired patients know sign language. Do they wear a hearing aid? Do they read lips? Do they wish to have a sign language interpreter? If the parent relies on lipreading, the nurse should sit close enough so that the parent can visualize lip movements. The nurse should speak clearly in a regular voice volume, in short, simple sentences. Written messages such as on a black or white erasable board can be useful. Written materials should be reviewed with the parents before discharge.

188
Q

The perinatal nurse is caring for a woman in the immediate postbirth period. Assessment reveals that the woman is experiencing profuse bleeding. The most likely etiology for the bleeding is:

a. uterine atony.
b. uterine inversion.
c. vaginal hematoma.
d. vaginal laceration.

A

ANS: A
Uterine atony is marked hypotonia of the uterus. It is the leading cause of after birth hemorrhage. Uterine inversion may lead to hemorrhage, but it is not the most likely source of this patient’s bleeding. Furthermore, if the woman is experiencing a uterine inversion, it would be evidenced by the presence of a large, red, rounded mass protruding from the introitus. A vaginal hematoma may be associated with hemorrhage. However, the most likely clinical finding would be pain, not the presence of profuse bleeding. A vaginal laceration may cause hemorrhage, but it is more likely that profuse bleeding would result from uterine atony. A vaginal laceration should be suspected if vaginal bleeding continues in the presence of a firm, contracted uterine fundus.

189
Q

A primary nursing responsibility when caring for a woman experiencing an obstetric hemorrhage associated with uterine atony is to:

a. establish venous access.
b. perform fundal massage.
c. prepare the woman for surgical intervention.
d. catheterize the bladder.

A

ANS: B
The initial management of excessive after birth bleeding is firm massage of the uterine fundus. Although establishing venous access may be a necessary intervention, the initial intervention would be fundal massage. The woman may need surgical intervention to treat her after birth hemorrhage, but the initial nursing intervention would be to assess the uterus. After uterine massage the nurse may want to catheterize the patient to eliminate any bladder distention that may be preventing the uterus from contracting properly.

190
Q

The perinatal nurse caring for the after birth woman understands that late postpartum hemorrhage (PPH) is most likely caused by:

a. subinvolution of the placental site.
b. defective vascularity of the decidua.
c. cervical lacerations.
d. coagulation disorders.

A

ANS: A
Late PPH may be the result of subinvolution of the uterus, pelvic infection, or retained placental fragments. Late PPH is not typically a result of defective vascularity of the decidua, cervical lacerations, or coagulation disorders.

191
Q

Which woman is at greatest risk for early postpartum hemorrhage (PPH)?

a. A primiparous woman (G 2 P 1 0 0 1) being prepared for an emergency cesarean birth for fetal distress.
b. A woman with severe preeclampsia who is receiving magnesium sulfate and whose labor is being induced.
c. A multiparous woman (G 3 P 2 0 0 2) with an 8-hour labor.
d. A primigravida in spontaneous labor with preterm twins.

A

ANS: B
Magnesium sulfate administration during labor poses a risk for PPH. Magnesium acts as a smooth muscle relaxant, thereby contributing to uterine relaxation and atony. Although many causes and risk factors are associated with PPH, the primiparous woman being prepared for an emergency C-section, the multiparous woman with 8-hour labor, and the primigravida in spontaneous labor do not pose risk factors or causes of early PPH.

192
Q

The first and most important nursing intervention when a nurse observes profuse after birth bleeding is to:

a. call the woman’s primary health care provider.
b. administer the standing order for an oxytocic.
c. palpate the uterus and massage it if it is boggy.
d. assess maternal blood pressure and pulse for signs of hypovolemic shock.

A

ANS: C
The initial management of excessive after birth bleeding is firm massage of the uterine fundus. Although calling the health care provider, administering an oxytocic, and assessing maternal BP are appropriate interventions, the primary intervention should be to assess the uterus. Uterine atony is the leading cause of postpartum hemorrhage (PPH).

193
Q

When caring for a after birth woman experiencing hemorrhagic shock, the nurse recognizes that the most objective and least invasive assessment of adequate organ perfusion and oxygenation is:

a. absence of cyanosis in the buccal mucosa.
b. cool, dry skin.
c. diminished restlessness.
d. urinary output of at least 30 mL/hr.

A

ANS: D
Hemorrhage may result in hemorrhagic shock. Shock is an emergency situation in which the perfusion of body organs may become severely compromised and death may occur. The presence of adequate urinary output indicates adequate tissue perfusion. The assessment of the buccal mucosa for cyanosis can be subjective. The presence of cool, pale, clammy skin would be an indicative finding associated with hemorrhagic shock. Hemorrhagic shock is associated with lethargy, not restlessness.

194
Q

One of the first symptoms of puerperal infection to assess for in the after birth woman is:

a. fatigue continuing for longer than 1 week.
b. pain with voiding.
c. profuse vaginal bleeding with ambulation.
d. temperature of 38° C (100.4° F) or higher on two successive days starting 24 hours after birth.

A

ANS: D
After birth or puerperal infection is any clinical infection of the genital canal that occurs within 28 days after miscarriage, induced abortion, or childbirth. The definition used in the United States continues to be the presence of a fever of 38° C (100.4° F) or higher on two successive days of the first 10 after birth days, starting 24 hours after birth. Fatigue would be a late finding associated with infection. Pain with voiding may indicate a urinary tract infection, but it is not typically one of the earlier symptoms of infection. Profuse lochia may be associated with endometritis, but it is not the first symptom associated with infection.

195
Q

Nurses need to know the basic definitions and incidence data about postpartum hemorrhage (PPH). For instance:

a. PPH is easy to recognize early; after all, the woman is bleeding.
b. traditionally it takes more than 1000 mL of blood after vaginal birth and 2500 mL after cesarean birth to define the condition as PPH.
c. if anything, nurses and doctors tend to overestimate the amount of blood loss.
d. traditionally PPH has been classified as early or late with respect to birth.

A

ANS: D
Early PPH is also known as primary, or acute, PPH; late PPH is known as secondary PPH. Unfortunately PPH can occur with little warning and often is recognized only after the mother has profound symptoms. Traditionally a 500-mL blood loss after a vaginal birth and a 1000-mL blood loss after a cesarean birth constitute PPH. Medical personnel tend to underestimate blood loss by as much as 50% in their subjective observations.

196
Q

A woman who has recently given birth complains of pain and tenderness in her leg. On physical examination the nurse notices warmth and redness over an enlarged, hardened area. The nurse should suspect __________ and should confirm the diagnosis by ___________.

a. disseminated intravascular coagulation; asking for laboratory tests
b. von Willebrand disease; noting whether bleeding times have been extended
c. thrombophlebitis; using real-time and color Doppler ultrasound
d. coagulopathies; drawing blood for laboratory analysis

A

ANS: C
Pain and tenderness in the extremities, which show warmth, redness, and hardness, likely indicate thrombophlebitis. Doppler ultrasound is a common noninvasive way to confirm diagnosis.

197
Q

What PPH conditions are considered medical emergencies that require immediate treatment?

a. Inversion of the uterus and hypovolemic shock
b. Hypotonic uterus and coagulopathies
c. Subinvolution of the uterus and idiopathic thrombocytopenic purpura
d. Uterine atony and disseminated intravascular coagulation

A

ANS: A
Inversion of the uterus and hypovolemic shock are considered medical emergencies. Although hypotonic uterus and coagulopathies, subinvolution of the uterus and idiopathic thrombocytopenic purpura, and uterine atony and disseminated intravascular coagulation are serious conditions, they are not necessarily medical emergencies that require immediate treatment.

198
Q

What infection is contracted mostly by first-time mothers who are breastfeeding?

a. Endometritis
b. Wound infections
c. Mastitis
d. Urinary tract infections

A

ANS: C
Mastitis is infection in a breast, usually confined to a milk duct. Most women who suffer this are primiparas who are breastfeeding.

199
Q

Despite popular belief, there is a rare type of hemophilia that affects women of childbearing age. von Willebrand disease is the most common of the hereditary bleeding disorders and can affect males and females alike. It results from a factor VIII deficiency and platelet dysfunction. Although factor VIII levels increase naturally during pregnancy, there is an increased risk for after birth hemorrhage from birth until 4 weeks after delivery as levels of von Willebrand factor (vWf) and factor VIII decrease. The treatment that should be considered first for the patient with von Willebrand disease who experiences a after birth hemorrhage is:

a. cryoprecipitate.
b. factor VIII and vWf.
c. desmopressin.
d. hemabate.

A

ANS: C
Desmopressin is the primary treatment of choice. This hormone can be administered orally, nasally, and intravenously. This medication promotes the release of factor VIII and vWf from storage. Cryoprecipitate may be used; however, because of the risk of possible donor viruses, other modalities are considered safer. Treatment with plasma products such as factor VIII and vWf is an acceptable option for this patient. Because of the repeated exposure to donor blood products and possible viruses, this is not the initial treatment of choice. Although the administration of this prostaglandin is known to promote contraction of the uterus during after birth hemorrhage, it is not effective for the patient who presents with a bleeding disorder.

200
Q

When a woman is diagnosed with postpartum depression (PPD) with psychotic features, one of the main concerns is that she may:

a. have outbursts of anger.
b. neglect her hygiene.
c. harm her infant.
d. lose interest in her husband.

A

ANS: C
Thoughts of harm to oneself’ or the infant are among the most serious symptoms of PPD and require immediate assessment and intervention. Although outbursts of anger, hygiene neglect, and loss of interest in her husband are attributable to PPD, the major concern would be the potential to harm herself or her infant.

201
Q

To provide adequate after birth care, the nurse should be aware that postpartum depression (PPD) without psychotic features:

a. means that the woman is experiencing the baby blues. In addition she has a visit with a counselor or psychologist.
b. is more common among older, Caucasian women because they have higher expectations.
c. is distinguished by irritability, severe anxiety, and panic attacks.
d. will disappear on its own without outside help.

A

ANS: C
PPD is also characterized by spontaneous crying long after the usual duration of the baby blues. PPD, even without psychotic features, is more serious and persistent than after birth baby blues. It is more common among younger mothers and African-American mothers. Most women need professional help to get through PPD, including pharmacologic intervention.

202
Q

To provide adequate after birth care, the nurse should be aware that postpartum depression (PPD) with psychotic features:

a. is more likely to occur in women with more than two children.
b. is rarely delusional and then is usually about someone trying to harm her (the mother).
c. although serious, is not likely to need psychiatric hospitalization.
d. may include bipolar disorder (formerly called “manic depression”).

A

ANS: D
Manic mood swings are possible. PPD is more likely to occur in first-time mothers. Delusions may be present in 50% of women with PPD, usually about something being wrong with the infant. PPD with psychosis is a psychiatric emergency that requires hospitalization.

203
Q

With shortened hospital stays, new mothers are often discharged before they begin to experience symptoms of the baby blues or after birth depression. As part of the discharge teaching, the nurse can prepare the mother for this adjustment to her new role by instructing her regarding self-care activities to help prevent after birth depression. The most accurate statement as related to these activities is to:

a. stay home and avoid outside activities to ensure adequate rest.
b. be certain that you are the only caregiver for your baby to facilitate infant attachment.
c. keep feelings of sadness and adjustment to your new role to yourself.
d. realize that this is a common occurrence that affects many women.

A

ANS: D
Should the new mother experience symptoms of the baby blues, it is important that she be aware that this is nothing to be ashamed of. Up to 80% of women experience this type of mild depression after the birth of their infant. Although it is important for the mother to obtain enough rest, she should not distance herself from family and friends. Her spouse or partner can communicate the best visiting times so the new mother can obtain adequate rest. It is also important that she does not isolate herself at home during this time of role adjustment. Even if breastfeeding, other family members can participate in the infant’s care. If depression occurs, the symptoms can often interfere with mothering functions, and this support will be essential. The new mother should share her feelings with someone else. It is also important that she not overcommit herself or think she has to be “superwoman.” A telephone call to the hospital warm line may provide reassurance with lactation issues and other infant care questions. Should symptoms continue, a referral to a professional therapist may be necessary.

204
Q

Complicated bereavement:

a. occurs when, in multiple births, one child dies, and the other or others live.
b. is a state in which the parents are ambivalent, as with an abortion.
c. is an extremely intense grief reaction that persists for a long time.
d. is felt by the family of adolescent mothers who lose their babies.

A

ANS: C
Parents showing signs of complicated grief should be referred for counseling. Multiple births in which not all the babies survive creates a complicated parenting situation, but this is not complicated bereavement. Abortion can generate complicated emotional responses, but they do not constitute complicated bereavement. Families of lost adolescent pregnancies may have to deal with complicated issues, but this is not complicated bereavement.

205
Q

Early after birth hemorrhage is defined as a blood loss greater than:

a. 500 mL in the first 24 hours after vaginal delivery.
b. 750 mL in the first 24 hours after vaginal delivery.
c. 1000 mL in the first 48 hours after cesarean delivery.
d. 1500 mL in the first 48 hours after cesarean delivery.

A

ANS: A
The average amount of bleeding after a vaginal birth is 500 mL. Blood loss after a cesarean birth averages 1000 mL. Early after birth hemorrhage occurs in the first 24 hours, not 48 hours. Late after birth hemorrhage is 48 hours and later.

206
Q

A woman delivered a 9-lb, 10-ounce baby 1 hour ago. When you arrive to perform her 15-minute assessment, she tells you that she “feels all wet underneath.” You discover that both pads are completely saturated and that she is lying in a 6-inch-diameter puddle of blood. What is your first action?

a. Call for help.
b. Assess the fundus for firmness.
c. Take her blood pressure.
d. Check the perineum for lacerations.

A

ANS: B
Firmness of the uterus is necessary to control bleeding from the placental site. The nurse should first assess for firmness and massage the fundus as indicated. Assessing blood pressure is an important assessment with a bleeding patient; however, the top priority is to control the bleeding. If bleeding continues in the presence of a firm fundus, lacerations may be the cause.

207
Q

A steady trickle of bright red blood from the vagina in the presence of a firm fundus suggests:

a. uterine atony.
b. lacerations of the genital tract.
c. perineal hematoma.
d. infection of the uterus.

A

ANS: B
Undetected lacerations will bleed slowly and continuously. Bleeding from lacerations is uncontrolled by uterine contraction. The fundus is not firm in the presence of uterine atony. A hematoma would develop internally. Swelling and discoloration would be noticeable; however, bright bleeding would not be. With an infection of the uterus there would be an odor to the lochia and systemic symptoms such as fever and malaise.

208
Q

Which instruction should be included in the discharge teaching plan to assist the patient in recognizing early signs of complications?

a. Palpate the fundus daily to ensure that it is soft.
b. Notify the physician of any increase in the amount of lochia or a return to bright red bleeding.
c. Report any decrease in the amount of brownish red lochia.
d. The passage of clots as large as an orange can be expected.

A

ANS: B
An increase in lochia or a return to bright red bleeding after the lochia has become pink indicates a complication. The fundus should stay firm. The lochia should decrease in amount over time. Large clots after discharge are a sign of complications and should be reported.

209
Q

If nonsurgical treatment for late after birth hemorrhage is ineffective, which surgical procedure is appropriate to correct the cause of this condition?

a. Hysterectomy
b. Laparoscopy
c. Laparotomy
d. D&C

A

ANS: D
D&C allows examination of the uterine contents and removal of any retained placental fragments or blood clots. Hysterectomy is the removal of the uterus and is not indicated for this condition. A laparoscopy is the insertion of an endoscope through the abdominal wall to examine the peritoneal cavity. It is not the appropriate treatment for this condition. A laparotomy is also not indicated for this condition. A laparotomy is a surgical incision into the peritoneal cavity to explore it.

210
Q

Which condition is a transient, self-limiting mood disorder that affects new mothers after childbirth?

a. After birth depression
b. After birth psychosis
c. After birth bipolar disorder
d. After birth blues

A

ANS: D
After birth blues or “baby blues” is a transient self-limiting disease that is believed to be related to hormonal fluctuations after childbirth. After birth depression is not the normal worries (blues) that many new mothers experience. Many caregivers believe that after birth depression is underdiagnosed and underreported. After birth psychosis is a rare condition that usually surfaces within 3 weeks of delivery. Hospitalization of the woman is usually necessary for treatment of this disorder. Bipolar disorder is one of the two categories of after birth psychosis, characterized by both manic and depressive episodes.

211
Q

Anxiety disorders are the most common mental disorders that affect women. While providing care to the maternity patient, the nurse should be aware that one of these disorders is likely to be triggered by the process of labor and birth. This disorder is:

a. phobias.
b. panic disorder.
c. posttraumatic stress disorder (PTSD).
d. obsessive-compulsive disorder (OCD).

A

ANS: C
In PTSD, women perceive childbirth as a traumatic event. They have nightmares and flashbacks about the event, anxiety, and avoidance of reminders of the traumatic event. Phobias are irrational fears that may lead a person to avoid certain objects, events, or situations. Panic disorders include episodes of intense apprehension, fear, and terror. Symptoms may manifest themselves as palpitations, chest pain, choking, or smothering. OCD symptoms include recurrent, persistent, and intrusive thoughts. The mother may repeatedly check and recheck her infant once he or she is born, even though she realizes that this is irrational. OCD is best treated with medications.

212
Q

Medications used to manage postpartum hemorrhage (PPH) include: (Select all that apply.)

a. Pitocin.
b. Methergine.
c. Terbutaline.
d. Hemabate.
e. magnesium sulfate.

A

ANS: A, B, D
Pitocin, Methergine, and Hemabate are all used to manage PPH. Terbutaline and magnesium sulfate are tocolytics; relaxation of the uterus causes or worsens PPH.

213
Q

What is the primary result of anemia?

a. Increased blood viscosity.
b. Depressed hematopoietic system.
c. Presence of abnormal hemoglobin.
d. Decreased oxygen-carrying capacity of blood.

A

ANS: D
Anemia is a condition in which the number of red blood cells or hemoglobin concentration is reduced below the normal values for age. This results in a decreased oxygen-carrying capacity of blood. Increased blood viscosity is usually a function of too many cells or of dehydration, not of anemia. A depressed hematopoietic system or abnormal hemoglobin can contribute to anemia, but the definition depends on the deceased oxygen-carrying capacity of the blood.

214
Q

Several blood tests are ordered for a preschool child with severe anemia. She is crying and upset because she remembers the venipuncture done at the clinic 2 days ago. The nurse should explain that:

a. venipuncture discomfort is very brief.
b. only one venipuncture will be needed.
c. topical application of local anesthetic can eliminate venipuncture pain.
d. most blood tests on children require only a finger puncture because a small amount of blood is needed.

A

ANS: C
Preschool children are very concerned about both pain and the loss of blood. When preparing the child for venipuncture, a topical anesthetic will be used to eliminate any pain. This is a very traumatic experience for preschool children. They are concerned about their bodily integrity. A local anesthetic should be used, and a bandage should be applied to maintain bodily integrity. A promise that only one venipuncture will be needed should not be made in case multiple attempts are required. Both finger punctures and venipunctures are traumatic for children. Both require preparation.

215
Q

What is the most appropriate nursing diagnosis for a child diagnosed with moderate anemia?

a. Activity intolerance related to generalized weakness
b. Decreased cardiac output related to abnormal hemoglobin
c. Risk for injury related to depressed sensorium
d. Risk for Injury related to dehydration and abnormal hemoglobin

A

ANS: A
The basic pathology in anemia is the decreased oxygen-carrying capacity of the blood. The nurse must assess the child’s activity level (response to the physiologic state). The nursing diagnosis would reflect the activity intolerance. In generalized anemia no abnormal hemoglobin may be present. Only at a level of very severe anemia does cardiac output become altered. No decreased sensorium exists until profound anemia occurs. Dehydration and abnormal hemoglobin are not usually part of anemia.

216
Q

Which statement best explains why iron deficiency anemia is common during toddlerhood?

a. Milk is a poor source of iron.
b. Iron cannot be stored during fetal development.
c. Fetal iron stores are depleted by age 1 month.
d. Dietary iron cannot be started until age 12 months.

A

ANS: A
Children between the ages of 12 and 36 months are at risk for anemia because cow’s milk is a major component of their diet, and it is a poor source of iron. Iron is stored during fetal development, but the amount stored depends on maternal iron stores. Fetal iron stores are usually depleted by age 5 to 6 months. Dietary iron can be introduced by breastfeeding, iron-fortified formula, and cereals during the first 12 months of life.

217
Q

When teaching the mother of a 9-month-old infant about administering liquid iron preparations, the nurse should include that information?

a. They should be given with meals.
b. They should be stopped immediately if nausea and vomiting occur.
c. Adequate dosage will turn the stools a tarry green color.
d. Preparation should be allowed to mix with saliva and bathe the teeth before swallowing.

A

ANS: C
The nurse should prepare the mother for the anticipated change in the child’s stools. If the iron dose is adequate, the stools will become a tarry green color. The lack of the color change may indicate insufficient iron. The iron should be given in two divided doses between meals, when the presence of free hydrochloric acid is greatest. Iron is absorbed best in an acidic environment. Vomiting and diarrhea may occur with iron administration. If these occur, the iron should be given with meals, and the dosage reduced and gradually increased as the child develops tolerance. Liquid preparations of iron stain the teeth. They should be administered through a straw, and the mouth rinsed after administration.

218
Q

Iron dextran is ordered for a young child with severe iron deficiency anemia. What nursing consideration should be considered?

a. Administering with meals
b. Administering between meals
c. Injecting deeply into a large muscle
d. Massaging injection site for 5 minutes after administration of drug

A

ANS: C
Iron dextran is a parenteral form of iron. When administered intramuscularly, it must be injected into a large muscle using the Z-track method. Iron dextran is for intramuscular or intravenous administration; it is not taken orally. The site should not be massaged to prevent leakage, potential irritation, and staining of the skin. The administration has no relationship to food since it is not being given orally.

219
Q

What intervention should the nurse share with parents on how to prevent iron deficiency anemia in a healthy, term, breastfed infant?

a. Iron (ferrous sulfate) drops after age 1 month
b. Iron-fortified commercial formula can be used by ages 4 to 6 months
c. Iron-fortified solid foods are introduced at 3 months
d. Iron-fortified infant cereal can be introduced at approximately 6 months of age

A

ANS: D
Breast milk supplies inadequate iron for growth and development after age 5 months. Supplementation is necessary at this time. Iron supplementation or the introduction of solid foods in a breastfed baby is not indicated. Introducing iron-fortified infant cereal at 2 months should be done only if the mother is choosing to discontinue breastfeeding.

220
Q

What term is used to identify the condition in which the normal adult hemoglobin is partly or completely replaced by abnormal hemoglobin?

a. Aplastic anemia
b. Sickle cell anemia
c. Thalassemia major
d. Iron deficiency anemia

A

ANS: B
Sickle cell anemia is one of a group of diseases collectively called hemoglobinopathies, in which normal adult hemoglobin is replaced by abnormal hemoglobin. Aplastic anemia is a lack of cellular elements being produced. Hemophilia refers to a group of bleeding disorders in which there is deficiency of one of the factors necessary for coagulation. Iron deficiency anemia affects size and depth of color of hemoglobin and does not involve abnormal hemoglobin.

221
Q

Which statement most accurately describes the pathologic changes of sickle cell anemia?

a. Sickle-shaped cells carry excess oxygen
b. Sickle-shaped cells decrease blood viscosity
c. Increased red blood cell destruction occurs
d. Decreased red blood cell destruction occurs

A

ANS: C
The clinical features of sickle cell anemia are primarily the result of increased red blood cell destruction and obstruction caused by the sickle-shaped red blood cells. Sickled red cells have decreased oxygen-carrying capacity and transform into the sickle shape in conditions of low oxygen tension. When the sickle cells change shape, they increase the viscosity in the area where they are involved in the microcirculation.

222
Q

Which clinical manifestation should the nurse expect when a child diagnosed with sickle cell anemia experiences an acute vaso-occlusive crisis?

a. Circulatory collapse
b. Cardiomegaly, systolic murmurs
c. Hepatomegaly, intrahepatic cholestasis
d. Painful swelling of hands and feet, painful joints

A

ANS: D
A vaso-occlusive crisis is characterized by severe pain in the area of involvement. If in the extremities, painful swelling of the hands and feet is seen; if in the abdomen, severe pain resembles that of acute surgical abdomen; and if in the head, stroke and visual disturbances occur. Circulatory collapse results from sequestration crises. Cardiomegaly, systolic murmurs, hepatomegaly, and intrahepatic cholestasis result from chronic vaso-occlusive phenomena.

223
Q

The parents of a child hospitalized with sickle cell anemia tell the nurse that they are concerned about narcotic analgesics causing addiction. The nurse should provide what explanation in response to the parent’s concern?

a. Narcotics are often ordered but not usually needed.
b. Narcotics rarely cause addiction when they are medically indicated.
c. Narcotics are given as a last resort because of the threat of addiction.
d. Narcotics are used only if other measures such as ice packs are ineffective.

A

ANS: B
The pain of sickle cell anemia is best treated by a multidisciplinary approach. Mild-to-moderate pain can be controlled by ibuprofen and acetaminophen. When narcotics are indicated, they are titrated to effect and given around the clock. Patient-controlled analgesia reinforces the patient’s role and responsibility in managing the pain and provides flexibility in dealing with pain. Few if any patients who receive opioids for severe pain become behaviorally addicted to the drug. Narcotics are often used because of the severe nature of the pain of vaso-occlusive crisis. Ice is contraindicated because of its vasoconstrictive effects.

224
Q

Which statement correctly describes -thalassemia major (Cooley’s anemia)?

a. All formed elements of the blood are depressed.
b. Inadequate numbers of red blood cells are present.
c. Increased incidence occurs in families of Mediterranean extraction.
d. Increased incidence occurs in persons of West African descent.

A

ANS: C
Individuals who live near the Mediterranean Sea and their descendants have the highest incidence of thalassemia. An overproduction of red cells occurs. Although numerous, the red cells are relatively unstable. Sickle cell disease is common in blacks of West African descent.

225
Q

Chelation therapy is begun on a child with -thalassemia major with what expected result?

a. Treatment of the disease.
b. Elimination of excess iron.
c. Decreasing the risk of hypoxia.
d. Managing nausea and vomiting.

A

ANS: B
A complication of the frequent blood transfusions in thalassemia is iron overload. Chelation therapy with deferoxamine (an iron-chelating agent) is given with oral supplements of vitamin C to increase iron excretion. Chelation therapy treats the side effects of disease management. Decreasing the risk of hypoxia and managing nausea and vomiting are not the purposes of chelation therapy.

226
Q

In which condition are all the formed elements of the blood simultaneously depressed?

a. Aplastic anemia
b. Sickle cell anemia
c. Thalassemia major
d. Iron deficiency anemia

A

ANS: A
Aplastic anemia refers to a bone marrow-failure condition in which the formed elements of the blood are simultaneously depressed. Sickle cell anemia is a hemoglobinopathy in which normal adult hemoglobin is partly or completely replaced by abnormal sickle hemoglobin. Thalassemia major is a group of blood disorders characterized by deficiency in the production rate of specific hemoglobin globin chains. Iron deficiency anemia results in a decreased amount of circulating red cells.

227
Q

As related to inherited disorders, which statement is descriptive of most cases of hemophilia?

a. Autosomal dominant disorder causing deficiency in a factor involved in the blood-clotting reaction
b. X-linked recessive inherited disorder causing deficiency of platelets and prolonged bleeding
c. X-linked recessive inherited disorder in which a blood-clotting factor is deficient
d. Y-linked recessive inherited disorder in which the red blood cells become moon shaped

A

ANS: C
The inheritance pattern in 80% of all of the cases of hemophilia is X-linked recessive. The two most common forms of the disorder are factor VIII deficiency (hemophilia A or classic hemophilia), and factor IX deficiency (hemophilia B or Christmas disease). The disorder involves coagulation factors, not platelets. The disorder does not involve red cells or the Y chromosome.

228
Q

What name is given to identify an acquired hemorrhagic disorder that is characterized by excessive destruction of platelets?

a. Aplastic anemia
b. Thalassemia major
c. Disseminated intravascular coagulation
d. Immune thrombocytopenia

A

ANS: D
Immune thrombocytopenia is an acquired hemorrhagic disorder characterized by an excessive destruction of platelets, discolorations caused by petechiae beneath the skin, and a normal bone marrow. Aplastic anemia refers to a bone marrow failure condition in which the formed elements of the blood are simultaneously depressed. Thalassemia major is a group of blood disorders characterized by deficiency in the production rate of specific hemoglobin globin chains. Disseminated intravascular coagulation is characterized by diffuse fibrin deposition in the microvasculature, consumption of coagulation factors, and endogenous generation of thrombin and plasma.

229
Q

Which condition is caused by a virus that primarily infects a specific subset of T lymphocytes, the CD4+ T-cells?

a. Wiskott-Aldrich syndrome
b. Idiopathic thrombocytopenic purpura (ITP)
c. Acquired immunodeficiency syndrome (AIDS)
d. Severe combined immunodeficiency disease

A

ANS: C
AIDS is caused by the human immunodeficiency virus, which primarily attacks the CD4+ T-cells. Wiskott-Aldrich syndrome, ITP, and severe combined immunodeficiency disease are not viral illnesses.

230
Q

A young child with human immunodeficiency virus is receiving several antiretroviral drugs. What is the expected outcome of these drug therapies?

a. Cure the disease
b. Delay disease progression
c. Prevent spread of disease
d. Treat Pneumocystis jiroveci pneumonia

A

ANS: B
Although not a cure, these antiviral drugs can suppress viral replication, preventing further deterioration of the immune system, and delay disease progression. At this time cure is not possible. These drugs do not prevent the spread of the disease. Pneumocystis jiroveci prophylaxis is accomplished with antibiotics.

231
Q

Which immunization should be given with caution to children infected with human immunodeficiency virus?

a. Influenza
b. Varicella
c. Pneumococcus
d. Inactivated poliovirus

A

ANS: B
The children should be carefully evaluated before giving live viral vaccines such as varicella, measles, mumps, and rubella. The child must be immunocompetent and not have contact with other severely immunocompromised individuals. Influenza, pneumococcus, and inactivated poliovirus are not live vaccines.

232
Q

An 8-year-old girl is receiving a blood transfusion when the nurse notes that she has developed precordial pain, dyspnea, distended neck veins, slight cyanosis, and a dry cough. These manifestations are most suggestive of what associated adverse reaction?

a. Air embolism
b. Allergic reaction
c. Hemolytic reaction
d. Circulatory overload

A

ANS: D
The signs of circulatory overload include distended neck veins, hypertension, crackles, dry cough, cyanosis, and precordial pain. Signs of air embolism are sudden difficulty breathing, sharp pain in the chest, and apprehension. Allergic reactions are manifested by urticaria, pruritus, flushing, asthmatic wheezing, and laryngeal edema. Hemolytic reactions are characterized by chills, shaking, fever, pain at infusion site, nausea, vomiting, tightness in chest, flank pain, red or black urine, and progressive signs of shock and renal failure.

233
Q

The parents of a child diagnosed with aplastic anemia tell the nurse that a bone marrow transplant (BMT) may be necessary. What should the nurse recognize as important when discussing this with the family?

a. BMT should be done at time of diagnosis.
b. Parents and siblings of child have a 25% chance of being a suitable donor.
c. Finding a suitable donor involves matching antigens from the human leukocyte antigen (HLA) system.
d. If BMT fails, chemotherapy or radiotherapy must be continued.

A

ANS: C
The most successful BMTs come from suitable HLA-matched donors. The timing of a BMT depends on the disease process involved. It usually follows intensive high-dose chemotherapy and/or radiation therapy. Usually parents only share approximately 50% of the genetic material with their children. A one-in-four chance exists that two siblings will have two identical haplotypes and will be identically matched at the HLA loci. Discussing the continuation of chemotherapy or radiotherapy is not appropriate when planning the BMT. That decision will be made later.

234
Q

What is the priority nursing intervention for a child hospitalized with hemarthrosis resulting from hemophilia?

a. Immobilization and elevation of the affected joint
b. Administration of acetaminophen for pain relief
c. Assessment of the child’s response to hospitalization
d. Assessment of the impact of hospitalization on the family system

A

ANS: A
Immobilization and elevation of the joint will prevent further injury until bleeding is resolved. Although acetaminophen may help with pain associated with the treatment of hemarthrosis, it is not the priority nursing intervention. Assessment of a child’s response to hospitalization is relevant to all hospitalized children; however, in this situation, psychosocial concerns are secondary to physiologic concerns. A priority nursing concern for this child is the management of hemarthrosis. Assessing the impact of hospitalization on the family system is relevant to all hospitalized children; however, it is not the priority in this situation.

235
Q

What is the most common mode of transmission of human immunodeficiency virus (HIV) in the pediatric population?

a. Perinatal transmission
b. Sexual abuse
c. Blood transfusions
d. Poor hand washing

A

ANS: A
Infected women can transmit the virus to their infants across the placenta during pregnancy, at delivery, and through breastfeeding. Cases of HIV infection from sexual abuse have been reported; however, perinatal transmission accounts for most pediatric HIV infections. In the past some children became infected with HIV through blood transfusions; however, improved laboratory screening has significantly reduced the probability of contracting HIV from blood products. Poor hand washing is not an etiology of HIV infection.

236
Q

The nurse is planning activity for a 4-year-old child with anemia. Which activity should the nurse plan for this child?

a. Game of “hide and seek” in the children’s outdoor play area
b. Participation in dance activities in the playroom
c. Puppet play in the child’s room
d. A walk down to the hospital lobby

A

ANS: C
Because the basic pathologic process in anemia is a decrease in oxygen-carrying capacity, an important nursing responsibility is to assess the child’s energy level and minimize excess demands. The child’s level of tolerance for activities of daily living and play is assessed, and adjustments are made to allow as much self-care as possible without undue exertion. Puppet play in the child’s room would not be overly tiring. Hide and seek, dancing, and walking to the lobby would not conserve the anemic child’s energy.

237
Q

A school-age child is admitted in vaso-occlusive sickle cell crisis. The child’s care should include which intervention? (Select all that apply.)

a. Correction of acidosis
b. Adequate hydration
c. Pain management
d. Administration of heparin
e. Replacement of factor VIII

A

ANS: B, C
The management of crises includes adequate hydration, minimizing energy expenditures, pain management, electrolyte replacement, and blood component therapy if indicated. The acidosis will be corrected as the crisis is treated. Heparin and factor VIII are not indicated in the treatment of vaso-occlusive sickle cell crisis.

238
Q

The nurse is caring for a child with aplastic anemia. Which nursing diagnoses are appropriate? (Select all that apply.)

a. Acute Pain related to vaso-occlusion
b. Risk for Infection related to inadequate secondary defenses or immunosuppression
c. Ineffective Protection related to thrombocytopenia
d. Ineffective Tissue Perfusion related to anemia
e. Ineffective Protection related to abnormal clotting

A

ANS: B, C, D
These are appropriate nursing diagnosis for the nurse planning care for a child with aplastic anemia. Aplastic anemia is a condition in which the bone marrow ceases production of the cells it normally manufactures, resulting in pancytopenia. The child will have varying degrees of the disease depending on how low the values are for absolute neutrophil count (affecting the body’s response to infection), platelet count (putting the child at risk for bleeding), and absolute reticulocyte count (causing the child to have anemia). Acute Pain related to vaso-occlusion is an appropriate nursing diagnosis for sickle cell anemia for the child in vaso-occlusive crisis, but it is not applicable to a child with aplastic anemia. Ineffective Protection related to abnormal clotting is an appropriate diagnosis for a child with hemophilia.

239
Q

The nurse is planning care for a school-age child admitted to the hospital with hemophilia. Which interventions should the nurse plan to implement for this child? (Select all that apply.)

a. Fingersticks for blood work instead of venipunctures
b. Avoidance of intramuscular (IM) injections
c. Acetaminophen for mild pain control
d. Soft toothbrush for dental hygiene
e. Administration of packed red blood cells

A

ANS: B, C, D
Nurses should take special precautions when caring for a child with hemophilia to prevent the use of procedures that may cause bleeding, such as IM injections. The subcutaneous route is substituted for IM injections whenever possible. Venipunctures for blood samples are usually preferred for these children. There is usually less bleeding after the venipuncture than after finger or heel punctures. Neither aspirin nor any aspirin-containing compound should be used. Acetaminophen is a suitable aspirin substitute, especially for controlling mild pain. A soft toothbrush is recommended for dental hygiene to prevent bleeding from the gums. Packed red blood cells are not administered. The primary therapy for hemophilia is replacement of the missing clotting factor. The products available are factor VIII concentrates.

240
Q

Parents of a school-age child with hemophilia ask the nurse, “Which sports are recommended for children with hemophilia?” Which sports should the nurse recommend? (Select all that apply.)

a. Soccer
b. Swimming
c. Basketball
d. Golf
e. Bowling

A

ANS: B, D, E
Because almost all persons with hemophilia are boys, the physical limitations in regard to active sports may be a difficult adjustment, and activity restrictions must be tempered with sensitivity to the child’s emotional and physical needs. Use of protective equipment, such as padding and helmets, is particularly important, and noncontact sports, especially swimming, walking, jogging, tennis, golf, fishing, and bowling, are encouraged. Contact sports such as soccer and basketball are not recommended.

241
Q

Which should the nurse teach about prevention of sickle cell crises to parents of a preschool child with sickle cell disease? (Select all that apply.)

a. Limit fluids at bedtime.
b. Notify the health care provider if a fever of 38.5° C (101.3° F) or greater occurs.
c. Give penicillin as prescribed.
d. Use ice packs to decrease the discomfort of vaso-occlusive pain in the legs.
e. Notify the health care provider if your child begins to develop symptoms of cold.

A

ANS: B, C, E
The most important issues to teach the family of a child with sickle cell anemia are to (1) seek early intervention for problems, such as a fever of 38.5° C (101.3° F) or greater; (2) give penicillin as ordered; (3) recognize signs and symptoms of splenic sequestration, as well as respiratory problems that can lead to hypoxia; and (4) treat the child normally. The nurse emphasizes the importance of adequate hydration to prevent sickling and to delay the adhesion-stasis-thrombosis-ischemia cycle. It is not sufficient to advise parents to “force fluids” or “encourage drinking.” They need specific instructions on how many daily glasses or bottles of fluid are required. Many foods are also a source of fluid, particularly soups, flavored ice pops, ice cream, sherbet, gelatin, and puddings. Increased fluids combined with impaired kidney function result in the problem of enuresis. Parents who are unaware of this fact frequently use the usual measures to discourage bed-wetting, such as limiting fluids at night. Enuresis is treated as a complication of the disease, such as joint pain or some other symptom, to alleviate parental pressure on the child. Ice should not be used during a vaso-occlusive pain crisis because it vasoconstricts and impairs circulation even more.

242
Q

A student nurse is assessing the peripheral vascular system of an older adult. What action by the student
would cause the faculty member to intervene?
a. Assessing blood pressure in both upper extremities
b. Auscultating the carotid arteries for any bruits
c. Classifying capillary refill of 4 seconds as normal
d. Palpating both carotid arteries at the same time

A

ANS: D
The student should not compress both carotid arteries at the same time to avoid brain ischemia. Blood pressure
should be taken and compared in both arms. Prolonged capillary refill is considered to be greater than 5
seconds in an older adult, so classifying refill of 4 seconds as normal would not require intervention. Bruits
should be auscultated.

243
Q

The nurse is reviewing the lipid panel of a male client who has atherosclerosis. Which finding is most
concerning?
a. Cholesterol: 126 mg/dL
b. High-density lipoprotein cholesterol (HDL-C): 48 mg/dL
c. Low-density lipoprotein cholesterol (LDL-C): 122 mg/dL
d. Triglycerides: 198 mg/dL

A

ANS: D

Triglycerides in men should be below 160 mg/dL. The other values are appropriate for adult males.

244
Q

The nurse is evaluating a 3-day diet history with a client who has an elevated lipid panel. What meal
selection indicates the client is managing this condition well with diet?
a. A 4-ounce steak, French fries, iceberg lettuce
b. Baked chicken breast, broccoli, tomatoes
c. Fried catfish, cornbread, peas
d. Spaghetti with meat sauce, garlic bread

A

ANS: B
The diet recommended for this client would be low in saturated fats and red meat, high in vegetables and
whole grains (fiber), low in salt, and low in trans fat. The best choice is the chicken with broccoli and
tomatoes. The French fries have too much fat and the iceberg lettuce has little fiber. The catfish is fried. The
spaghetti dinner has too much red meat and no vegetables.

245
Q

A nurse is working with a client who takes atorvastatin (Lipitor). The clients recent laboratory results
include a blood urea nitrogen (BUN) of 33 mg/dL and creatinine of 2.8 mg/dL. What action by the nurse is
best?
a. Ask if the client eats grapefruit.
b. Assess the client for dehydration. c. Facilitate admission to the hospital. d. Obtain a random urinalysis.

A

ANS: A
There is a drug-food interaction between statins and grapefruit that can lead to acute kidney failure. This client
has elevated renal laboratory results, indicating some degree of kidney involvement. The nurse should assess if
the client eats grapefruit or drinks grapefruit juice. Dehydration can cause the BUN to be elevated, but the
elevation in creatinine is more specific for a kidney injury. The client does not necessarily need to be admitted. A urinalysis may or may not be ordered.

246
Q

A client has been diagnosed with hypertension but does not take the antihypertensive medications because
of a lack of symptoms. What response by the nurse is best?
a. Do you have trouble affording your medications?
b. Most people with hypertension do not have symptoms. c. You are lucky; most people get severe morning headaches. d. You need to take your medicine or you will get kidney failure.

A

ANS: B
Most people with hypertension are asymptomatic, although a small percentage do have symptoms such as
headache. The nurse should explain this to the client. Asking about paying for medications is not related
because the client has already admitted nonadherence. Threatening the client with possible complications will
not increase compliance.

247
Q

. A student nurse asks what essential hypertension is. What response by the registered nurse is best?
a. It means it is caused by another disease. b. It means it is essential that it be treated. c. It is hypertension with no specific cause. d. It refers to severe and life-threatening hypertension.

A

ANS: C
Essential hypertension is the most common type of hypertension and has no specific cause such as an
underlying disease process. Hypertension that is due to another disease process is called secondary
hypertension. A severe, life-threatening form of hypertension is malignant hypertension.

248
Q

A nurse is interested in providing community education and screening on hypertension. In order to reach a
priority population, to what target audience should the nurse provide this service?
a. African-American churches
b. Asian-American groceries
c. High school sports camps
d. Womens health clinics

A

ANS: A
African Americans in the United States have one of the highest rates of hypertension in the world. The nurse has the potential to reach this priority population by providing services at African-American churches. Although hypertension education and screening are important for all groups, African Americans are the
priority population for this intervention.

249
Q

has the potential to reach this priority population by providing services at African-American churches. Although hypertension education and screening are important for all groups, African Americans are the
priority population for this intervention.

A

ANS: B
All options are appropriate when assessing stress and responses to stress. However, this client feels
overwhelmed by the suggested lifestyle changes. Instead of looking at all the needed changes, the nurse should
assist the client in choosing one the client feels optimistic about controlling. Once the client has mastered that
change, he or she can move forward with another change. Determining support systems, daily stressors, and
delegation opportunities does not directly impact the clients feelings of control.

250
Q

The nurse is caring for four hypertensive clients. Which druglaboratory value combination should the nurse
report immediately to the health care provider?
a. Furosemide (Lasix)/potassium: 2.1 mEq/L
b. Hydrochlorothiazide (Hydrodiuril)/potassium: 4.2 mEq/L
c. Spironolactone (Aldactone)/potassium: 5.1 mEq/L
d. Torsemide (Demadex)/sodium: 142 mEq/L

A

ANS: A
Lasix is a loop diuretic and can cause hypokalemia. A potassium level of 2.1 mEq/L is quite low and should be
reported immediately. Spironolactone is a potassium-sparing diuretic that can cause hyperkalemia. A
potassium level of 5.1 mEq/L is on the high side, but it is not as critical as the low potassium with furosemide. The other two laboratory values are normal.

251
Q

A nurse is assessing a client with peripheral artery disease (PAD). The client states walking five blocks is
possible without pain. What question asked next by the nurse will give the best information?
a. Could you walk further than that a few months ago?
b. Do you walk mostly uphill, downhill, or on flat surfaces?
c. Have you ever considered swimming instead of walking?
d. How much pain medication do you take each day?

A

ANS: A
As PAD progresses, it takes less oxygen demand to cause pain. Needing to cut down on activity to be pain free
indicates the clients disease is worsening. The other questions are useful, but not as important.

252
Q

An older client with peripheral vascular disease (PVD) is explaining the daily foot care regimen to the
family practice clinic nurse. What statement by the client may indicate a barrier to proper foot care?
a. I nearly always wear comfy sweatpants and house shoes. b. Im glad I get energy assistance so my house isnt so cold. c. My daughter makes sure I have plenty of lotion for my feet. d. My hands shake when I try to do things requiring coordination.

A

ANS: D
Clients with PVD need to pay special attention to their feet. Toenails need to be kept short and cut straight
across. The client whose hands shake may cause injury when trimming toenails. The nurse should refer this
client to a podiatrist. Comfy sweatpants and house shoes are generally loose and not restrictive, which is
important for clients with PVD. Keeping the house at a comfortable temperature makes it less likely the client
will use alternative heat sources, such as heating pads, to stay warm. The client should keep the feet moist and
soft with lotion.

253
Q

A client is taking warfarin (Coumadin) and asks the nurse if taking St. Johns wort is acceptable. What
response by the nurse is best?
a. No, it may interfere with the warfarin. b. There isnt any information about that. c. Why would you want to take that?
d. Yes, it is a good supplement for you.

A

ANS: A
Many foods and drugs interfere with warfarin, St. Johns wort being one of them. The nurse should advise the
client against taking it. The other answers are not accurate.

254
Q

A nurse is teaching a larger female client about alcohol intake and how it affects hypertension. The client
asks if drinking two beers a night is an acceptable intake. What answer by the nurse is best?
a. No, women should only have one beer a day as a general rule. b. No, you should not drink any alcohol with hypertension. c. Yes, since you are larger, you can have more alcohol. d. Yes, two beers per day is an acceptable amount of alcohol.

A

ANS: A
Alcohol intake should be limited to two drinks a day for men and one drink a day for women. A drink is
classified as one beer, 1.5 ounces of hard liquor, or 5 ounces of wine. Limited alcohol intake is acceptable with
hypertension. The womans size does not matter.

255
Q
  1. A nurse is caring for four clients. Which one should the nurse see first?
    a. Client who needs a beta blocker, and has a blood pressure of 92/58 mm Hg
    b. Client who had a first dose of captopril (Capoten) and needs to use the bathroom
    c. Hypertensive client with a blood pressure of 188/92 mm Hg
    d. Client who needs pain medication prior to a dressing change of a surgical wound
A

ANS: B
Angiotensin-converting enzyme inhibitors such as captopril can cause hypotension, especially after the first
dose. The nurse should see this client first to prevent falling if the client decides to get up without assistance. The two blood pressure readings are abnormal but not critical. The nurse should check on the client with higher
blood pressure next to assess for problems related to the reading. The nurse can administer the beta blocker as
standards state to hold it if the systolic blood pressure is below 90 mm Hg. The client who needs pain
medication prior to the dressing change is not a priority over client safety and assisting the other client to the
bathroom.

256
Q

A client had a percutaneous transluminal coronary angioplasty for peripheral arterial disease. What
assessment finding by the nurse indicates a priority outcome for this client has been met?
a. Pain rated as 2/10 after medication
b. Distal pulse on affected extremity 2+/4+
c. Remains on bedrest as directed
d. Verbalizes understanding of procedure

A

ANS: B
Assessing circulation distal to the puncture site is a critical nursing action. A pulse of 2+/4+ indicates good
perfusion. Pain control, remaining on bedrest as directed after the procedure, and understanding are all
important, but do not take priority over perfusion.

257
Q

A client is 4 hours postoperative after a femoropopliteal bypass. The client reports throbbing leg pain on
the affected side, rated as 7/10. What action by the nurse takes priority?
a. Administer pain medication as ordered. b. Assess distal pulses and skin color. c. Document the findings in the clients chart. d. Notify the surgeon immediately.

A

ANS: B
Once perfusion has been restored or improved to an extremity, clients can often feel a throbbing pain due to the
increased blood flow. However, it is important to differentiate this pain from ischemia. The nurse should assess
for other signs of perfusion, such as distal pulses and skin color/temperature. Administering pain medication is
done once the nurse determines the clients perfusion status is normal. Documentation needs to be thorough. Notifying the surgeon is not necessary.

258
Q

A client had a femoropopliteal bypass graft with a synthetic graft. What action by the nurse is most
important to prevent wound infection?
a. Appropriate hand hygiene before giving care
b. Assessing the clients temperature every 4 hours
c. Clean technique when changing dressings
d. Monitoring the clients daily white blood cell count

A

ANS: A
Hand hygiene is the best way to prevent infections in hospitalized clients. Dressing changes should be done
with sterile technique. Assessing vital signs and white blood cell count will not prevent infection.

259
Q

. A client is receiving an infusion of alteplase (Activase) for an intra-arterial clot. The client begins to
mumble and is disoriented. What action by the nurse takes priority?
a. Assess the clients neurologic status. b. Notify the Rapid Response Team. c. Prepare to administer vitamin K. d. Turn down the infusion rate.

A

ANS: B
Clients on fibrinolytic therapy are at high risk of bleeding. The sudden onset of neurologic signs may indicate
the client is having a hemorrhagic stroke. The nurse does need to complete a thorough neurological
examination, but should first call the Rapid Response Team based on the clients manifestations. The nurse
notifies the Rapid Response Team first. Vitamin K is not the antidote for this drug. Turning down the infusion
rate will not be helpful if the client is still receiving any of the drug.

260
Q

. A nursing student is caring for a client with an abdominal aneurysm. What action by the student requires
the registered nurse to intervene?
a. Assesses the client for back pain
b. Auscultates over abdominal bruit
c. Measures the abdominal girth
d. Palpates the abdomen in four quadrants

A

ANS: D
Abdominal aneurysms should never be palpated as this increases the risk of rupture. The registered nurse
should intervene when the student attempts to do this. The other actions are appropriate.

261
Q
A nurse is caring for a client with a deep vein thrombosis (DVT). What nursing assessment indicates a
priority outcome has been met?
a. Ambulates with assistance
b. Oxygen saturation of 98%
c. Pain of 2/10 after medication
d. Verbalizing risk factors
A

ANS: B
A critical complication of DVT is pulmonary embolism. A normal oxygen saturation indicates that this has not
occurred. The other assessments are also positive, but not the priority.

262
Q

A client has a deep vein thrombosis (DVT). What comfort measure does the nurse delegate to the
unlicensed assistive personnel (UAP)?
a. Ambulate the client. b. Apply a warm moist pack. c. Massage the clients leg. d. Provide an ice pack

A

ANS: B
Warm moist packs will help with the pain of a DVT. Ambulation is not a comfort measure. Massaging the
clients legs is contraindicated to prevent complications such as pulmonary embolism. Ice packs are not
recommended for DVT.

263
Q

nurse is assessing an obese client in the clinic for follow-up after an episode of deep vein thrombosis. The client has lost 20 pounds since the last visit. What action by the nurse is best?
a. Ask if the weight loss was intended. b. Encourage a high-protein, high-fiber diet. c. Measure for new compression stockings. d. Review a 3-day food recall diary.

A

ANS: C
Compression stockings must fit correctly in order to work. After losing a significant amount of weight, the
client should be re-measured and new stockings ordered if needed. The other options are appropriate, but not
the most important.

264
Q

A nurse wants to provide community service that helps meet the goals of Healthy People 2020 (HP2020)
related to cardiovascular disease and stroke. What activity would best meet this goal?
a. Teach high school students heart-healthy living. b. Participate in blood pressure screenings at the mall. c. Provide pamphlets on heart disease at the grocery store. d. Set up an Ask the nurse booth at the pet store.

A

ANS: B
An important goal of HP2020 is to increase the proportion of adults who have had their blood pressure
measured within the preceding 2 years and can state whether their blood pressure was normal or high. Participating in blood pressure screening in a public spot will best help meet that goal. The other options are all
appropriate but do not specifically help meet a goal.

265
Q

A client has been diagnosed with a deep vein thrombosis and is to be discharged on warfarin (Coumadin). The client is adamant about refusing the drug because its dangerous. What action by the nurse is best?

a. Assess the reason behind the clients fear.
b. Remind the client about laboratory monitoring. c. Tell the client drugs are safer today than before. d. Warn the client about consequences of noncompliance.

A

ANS: A
The first step is to assess the reason behind the clients fear, which may be related to the experience of someone
the client knows who took warfarin. If the nurse cannot address the specific rationale, teaching will likely be
unsuccessful. Laboratory monitoring once every few weeks may not make the client perceive the drug to be
safe. General statements like drugs are safer today do not address the root cause of the problem. Warning the
client about possible consequences of not taking the drug is not therapeutic and is likely to lead to an
adversarial relationship.

266
Q

A client with a history of heart failure and hypertension is in the clinic for a follow-up visit. The client is on
lisinopril (Prinivil) and warfarin (Coumadin). The client reports new-onset cough. What action by the nurse is
most appropriate?
a. Assess the clients lung sounds and oxygenation. b. Instruct the client on another antihypertensive. c. Obtain a set of vital signs and document them. d. Remind the client that cough is a side effect of Prinivil.

A

A client with a history of heart failure and hypertension is in the clinic for a follow-up visit. The client is on
lisinopril (Prinivil) and warfarin (Coumadin). The client reports new-onset cough. What action by the nurse is
most appropriate?
a. Assess the clients lung sounds and oxygenation. b. Instruct the client on another antihypertensive. c. Obtain a set of vital signs and document them. d. Remind the client that cough is a side effect of Prinivil.

267
Q

A nurse is caring for a client with a nonhealing arterial lower leg ulcer. What action by the nurse is best?
a. Consult with the Wound Ostomy Care Nurse. b. Give pain medication prior to dressing changes. c. Maintain sterile technique for dressing changes. d. Prepare the client for eventual amputation.

A

ANS: A
A nonhealing wound needs the expertise of the Wound Ostomy Care Nurse (or Wound Ostomy Continence
Nurse). Premedicating prior to painful procedures and maintaining sterile technique are helpful, but if the
wound is not healing, more needs to be done. The client may need an amputation, but other options need to be
tried first.

268
Q

A client has peripheral arterial disease (PAD). What statement by the client indicates misunderstanding
about self-management activities?
a. I can use a heating pad on my legs if its set on low. b. I should not cross my legs when sitting or lying down. c. I will go out and buy some warm, heavy socks to wear.
d. Its going to be really hard but i will stop smoking

A

ANS: A
Clients with PAD should never use heating pads as skin sensitivity is diminished and burns can result. The
other statements show good understanding of self-management.

269
Q

A client presents to the emergency department with a severely lacerated artery. What is the priority action
for the nurse?
a. Administer oxygen via non-rebreather mask. b. Ensure the client has a patent airway. c. Prepare to assist with suturing the artery. d. Start two large-bore IVs with normal saline

A

ANS: B
Airway always takes priority, followed by breathing and circulation. The nurse ensures the client has a patent
airway prior to providing any other care measures.

270
Q

The nurse is assessing a client on admission to the hospital. The clients leg appears as shown below:
What action by the nurse is best?
a. Assess the clients ankle-brachial index. b. Elevate the clients leg above the heart. c. Obtain an ice pack to provide comfort. d. Prepare to teach about heparin sodium.

A

ANS: A
This client has dependent rubor, a classic finding in peripheral arterial disease. The nurse should measure the
clients ankle-brachial index. Elevating the leg above the heart will further impede arterial blood flow. Ice will
cause vasoconstriction, also impeding circulation and perhaps causing tissue injury. Heparin sodium is not the
drug of choice for this condition.

271
Q
  1. What nonpharmacologic comfort measures should the nurse include in the plan of care for a client with
    severe varicose veins? (Select all that apply.)
    a. Administering mild analgesics for pain
    b. Applying elastic compression stockings
    c. Elevating the legs when sitting or lying
    d. Reminding the client to do leg exercises
    e. Teaching the client about surgical options
A

ANS: B, C, D
The three Es of care for varicose veins include elastic compression hose, exercise, and elevation. Mild
analgesics are not a nonpharmacologic measure. Teaching about surgical options is not a comfort measure.

272
Q

A nurse is preparing a client for a femoropopliteal bypass operation. What actions does the nurse delegate to
the unlicensed assistive personnel (UAP)? (Select all that apply.)
a. Administering preoperative medication
b. Ensuring the consent is signed
c. Marking pulses with a pen
d. Raising the siderails on the bed
e. Recording baseline vital signs

A

ANS: D, E
The UAP can raise the siderails of the bed for client safety and take and record the vital signs. Administering
medications, ensuring a consent is on the chart, and marking the pulses for later comparison should be done by
the registered nurse. This is also often done by the postanesthesia care nurse and is part of the hand-off report.

273
Q

A client has been bedridden for several days after major abdominal surgery. What action does the nurse
delegate to the unlicensed assistive personnel (UAP) for deep vein thrombosis (DVT) prevention? (Select all
that apply.)
a. Apply compression stockings. b. Assist with ambulation. c. Encourage coughing and deep breathing. d. Offer fluids frequently. e. Teach leg exercises.

A

A client has been bedridden for several days after major abdominal surgery. What action does the nurse
delegate to the unlicensed assistive personnel (UAP) for deep vein thrombosis (DVT) prevention? (Select all
that apply.)
a. Apply compression stockings. b. Assist with ambulation. c. Encourage coughing and deep breathing. d. Offer fluids frequently. e. Teach leg exercises.

274
Q

A nurse is caring for a client on IV infusion of heparin. What actions does this nurse include in the clients
plan of care? (Select all that apply.)
a. Assess the client for bleeding. b. Monitor the daily activated partial thromboplastin time (aPTT) results. c. Stop the IV for aPTT above baseline. d. Use an IV pump for the infusion. e. Weigh the client daily on the same scale.

A

ANS: A, B, D
Assessing for bleeding, monitoring aPTT, and using an IV pump for the infusion are all important safety
measures for heparin to prevent injury from bleeding. The aPTT needs to be 1.5 to 2 times normal in order to
demonstrate that the heparin is therapeutic. Weighing the client is not related.

275
Q

A client is being discharged on warfarin (Coumadin) therapy. What discharge instructions is the nurse
required to provide? (Select all that apply.)
a. Dietary restrictions
b. Driving restrictions
c. Follow-up laboratory monitoring
d. Possible drug-drug interactions
e. Reason to take medication

A

ANS: A, C, D, E
The Joint Commissions Core Measures state that clients being discharged on warfarin need instruction on
follow-up monitoring, dietary restrictions, drug-drug interactions, and reason for compliance. Driving is
typically not restricted.

276
Q

Which statements by the client indicate good understanding of foot care in peripheral vascular disease?
(Select all that apply.)
a. A good abrasive pumice stone will keep my feet soft. b. Ill always wear shoes if I can buy cheap flip-flops. c. I will keep my feet dry, especially between the toes. d. Lotion is important to keep my feet smooth and soft. e. Washing my feet in room-temperature water is best.

A

ANS: C, D, E
Good foot care includes appropriate hygiene and injury prevention. Keeping the feet dry; wearing good, comfortable shoes; using lotion; washing the feet in room-temperature water; and cutting the nails straight
across are all important measures. Abrasive material such as pumice stones should not be used. Cheap flipflops may not fit well and wont offer much protection against injury.

277
Q

ANS: C, D, E
Good foot care includes appropriate hygiene and injury prevention. Keeping the feet dry; wearing good, comfortable shoes; using lotion; washing the feet in room-temperature water; and cutting the nails straight
across are all important measures. Abrasive material such as pumice stones should not be used. Cheap flipflops may not fit well and wont offer much protection against injury.

A

ANS: A, B, C
When a client is upset, the nurse should offer self by remaining with the client if desired. Other helpful
measures include determining what and whom the client has for support systems and asking the client to
describe what he or she is feeling. Telling the client how smoking has led to this situation will only upset the
client further and will damage the therapeutic relationship. Telling the client that many people have
amputations belittles the clients feelings.

278
Q

The nurse working in the emergency department knows that which factors are commonly related aneurysm formation? (Select all that apply.)

a. Atherosclerosis
b. Down syndrome
c. Frequent heartburn
d. History of hypertension
e. History of smoking

A

NS: A, D, E
Atherosclerosis, hypertension, hyperlipidemia, and smoking are the most common related factors. Down
syndrome and heartburn have no relation to aneurysm formation.

279
Q

. A client with a known abdominal aortic aneurysm reports dizziness and severe abdominal pain. The nurse
assesses the clients blood pressure at 82/40 mm Hg. What actions by the nurse are most important? (Select all
that apply.)
a. Administer pain medication. b. Assess distal pulses every 10 minutes. c. Have the client sign a surgical consent. d. Notify the Rapid Response Team. e. Take vital signs every 10 minutes.

A

ANS: B, D, E
This client may have a ruptured/rupturing aneurysm. The nurse should notify the Rapid Response team and
perform frequent client assessments. Giving pain medication will lower the clients blood pressure even further. The nurse cannot have the client sign a consent until the physician has explained the procedure.

280
Q

A student is caring for a client who suffered massive blood loss after trauma. How does the student correlate
the blood loss with the clients mean arterial pressure (MAP)?
a. It causes vasoconstriction and increased MAP. b. Lower blood volume lowers MAP. c. There is no direct correlation to MAP. d. It raises cardiac output and MAP.

A

ANS: B

Lower blood volume will decrease MAP. The other answers are not accurate.

281
Q

. A nurse is caring for a client after surgery. The clients respiratory rate has increased from 12 to 18
breaths/min and the pulse rate increased from 86 to 98 beats/min since they were last assessed 4 hours ago. What action by the nurse is best?
a. Ask if the client needs pain medication. b. Assess the clients tissue perfusion further. c. Document the findings in the clients chart. d. Increase the rate of the clients IV infusion.

A

ANS: B
Signs of the earliest stage of shock are subtle and may manifest in slight increases in heart rate, respiratory rate, or blood pressure. Even though these readings are not out of the normal range, the nurse should conduct a
thorough assessment of the client, focusing on indicators of perfusion. The client may need pain medication, but this is not the priority at this time. Documentation should be done thoroug

282
Q

The nurse gets the hand-off report on four clients. Which client should the nurse assess first?

a. Client with a blood pressure change of 128/74 to 110/88 mm Hg
b. Client with oxygen saturation unchanged at 94%
c. Client with a pulse change of 100 to 88 beats/min
d. Client with urine output of 40 mL/hr for the last 2 hours

A

The nurse gets the hand-off report on four clients. Which client should the nurse assess first?

a. Client with a blood pressure change of 128/74 to 110/88 mm Hg
b. Client with oxygen saturation unchanged at 94%
c. Client with a pulse change of 100 to 88 beats/min
d. Client with urine output of 40 mL/hr for the last 2 hours

283
Q

A nurse is caring for a client after surgery who is restless and apprehensive. The unlicensed assistive personnel (UAP) reports the vital signs and the nurse sees they are only slightly different from previous

readings. What action does the nurse delegate next to the UAP?
a. Assess the client for pain or discomfort. b. Measure urine output from the catheter. c. Reposition the client to the unaffected side. d. Stay with the client and reassure him or her.

A

ANS: B
Urine output changes are a sensitive early indicator of shock. The nurse should delegate emptying the urinary
catheter and measuring output to the UAP as a baseline for hourly urine output measurements. The UAP
cannot assess for pain. Repositioning may or may not be effective for decreasing restlessness, but does not take
priority over physical assessments. Reassurance is a therapeutic nursing action, but the nurse needs to do more
in this situation.

284
Q

client is in shock and the nurse prepares to administer insulin for a blood glucose reading of 208 mg/dL. The spouse asks why the client needs insulin as the client is not a diabetic. What response by the nurse is best?
a. High glucose is common in shock and needs to be treated. b. Some of the medications we are giving are to raise blood sugar. c. The IV solution has lots of glucose, which raises blood sugar. d. The stress of this illness has made your spouse a diabetic

A

client is in shock and the nurse prepares to administer insulin for a blood glucose reading of 208 mg/dL. The spouse asks why the client needs insulin as the client is not a diabetic. What response by the nurse is best?
a. High glucose is common in shock and needs to be treated. b. Some of the medications we are giving are to raise blood sugar. c. The IV solution has lots of glucose, which raises blood sugar. d. The stress of this illness has made your spouse a diabetic

285
Q

A nurse caring for a client notes the following assessments: white blood cell count 3800/mm3, blood
glucose level 198 mg/dL, and temperature 96.2 F (35.6 C). What action by the nurse takes priority?
a. Document the findings in the clients chart. b. Give the client warmed blankets for comfort. c. Notify the health care provider immediately. d. Prepare to administer insulin per sliding scale.

A

ANS: C
This client has several indicators of sepsis with systemic inflammatory response. The nurse should notify the
health care provider immediately. Documentation needs to be thorough but does not take priority. The client
may appreciate warm blankets, but comfort measures do not take priority. The client may or may not need
insulin.

286
Q

A nurse works at a community center for older adults. What self-management measure can the nurse teach
the clients to prevent shock?
a. Do not get dehydrated in warm weather. b. Drink fluids on a regular schedule. c. Seek attention for any lacerations.
d. Take medications as prescribed

A

A nurse works at a community center for older adults. What self-management measure can the nurse teach
the clients to prevent shock?
a. Do not get dehydrated in warm weather. b. Drink fluids on a regular schedule. c. Seek attention for any lacerations.

287
Q

A client arrives in the emergency department after being in a car crash with fatalities. The client has a nearly
amputated leg that is bleeding profusely. What action by the nurse takes priority?
a. Apply direct pressure to the bleeding. b. Ensure the client has a patent airway. c. Obtain consent for emergency surgery. d. Start two large-bore IV catheters.

A

ANS: B
Airway is the priority, followed by breathing and circulation (IVs and direct pressure). Obtaining consent is
done by the physician.

288
Q

A client is receiving norepinephrine (Levophed) for shock. What assessment finding best indicates a
therapeutic effect from this drug?
a. Alert and oriented, answering questions
b. Client denial of chest pain or chest pressure
c. IV site without redness or swelling
d. Urine output of 30 mL/hr for 2 hours

A

ANS: A
Normal cognitive function is a good indicator that the client is receiving the benefits of norepinephrine. The
brain is very sensitive to changes in oxygenation and perfusion. Norepinephrine can cause chest pain as an
adverse reaction, so the absence of chest pain does not indicate therapeutic effect. The IV site is normal. The
urine output is normal, but only minimally so.

289
Q

A student nurse is caring for a client who will be receiving sodium nitroprusside (Nipride) via IV infusion. What action by the student causes the registered nurse to intervene?

a. Assessing the IV site before giving the drug
b. Obtaining a programmable (smart) IV pump
c. Removing the IV bag from the brown plastic cover
d. Taking and recording a baseline set of vital signs

A

ANS: C
Nitroprusside degrades in the presence of light, so it must be protected by leaving it in the original brown
plastic bag when infusing. The other actions are correct, although a smart pump is not necessarily required if the facility does not have them available. The drug must be administered via an IV pump, although the programmable pump is preferred for safety.

290
Q

A client has been brought to the emergency department after being shot multiple times. What action should
the nurse perform first?
a. Apply personal protective equipment. b. Notify local law enforcement officials. c. Obtain universal donor blood. d. Prepare the client for emergency surgery.

A

ANS: A
The nurses priority is to care for the client. Since the client has gunshot wounds and is bleeding, the nurse
applies personal protective equipment (i.e., gloves) prior to care. This takes priority over calling law
enforcement. Requesting blood bank products can be delegated. The nurse may or may not have to prepare the
client for emergency surgery.

291
Q

A nurse is caring for several clients at risk for shock. Which laboratory value requires the nurse to
communicate with the health care provider?
a. Creatinine: 0.9 mg/dL
b. Lactate: 6 mmol/L
c. Sodium: 150 mEq/L
d. White blood cell count: 11,000/mm3

A

ANS: B
A lactate level of 6 mmol/L is high and is indicative of possible shock. A creatinine level of 0.9 mg/dL is
normal. A sodium level of 150 mEq/L is high, but that is not related directly to shock. A white blood cell count
of 11,000/mm3 is slightly high but is not as critical as the lactate level.

292
Q

A client in shock is apprehensive and slightly confused. What action by the nurse is best?
a. Offer to remain with the client for awhile. b. Prepare to administer antianxiety medication. c. Raise all four siderails on the clients bed. d. Tell the client everything possible is being done.

A

A client in shock is apprehensive and slightly confused. What action by the nurse is best?
a. Offer to remain with the client for awhile. b. Prepare to administer antianxiety medication. c. Raise all four siderails on the clients bed. d. Tell the client everything possible is being done.

293
Q

A client is being discharged home after a large myocardial infarction and subsequent coronary artery
bypass grafting surgery. The clients sternal wound has not yet healed. What statement by the client most
indicates a higher risk of developing sepsis after discharge?
a. All my friends and neighbors are planning a party for me. b. I hope I can get my water turned back on when I get home. c. I am going to have my daughter scoop the cat litter box. d. My grandkids are so excited to have me coming home!

A

ANS: B
All these statements indicate a potential for leading to infection once the client gets back home. A large party
might include individuals who are themselves ill and contagious. Having litter boxes in the home can expose
the client to microbes that can lead to infection. Small children often have upper respiratory infections and
poor hand hygiene that spread germs. However, the most worrisome statement is the lack of running water for
handwashing and general hygiene and cleaning purposes.

294
Q

A client in shock has been started on dopamine. What assessment finding requires the nurse to
communicate with the provider immediately?
a. Blood pressure of 98/68 mm Hg
b. Pedal pulses 1+/4+ bilaterally
c. Report of chest heaviness
d. Urine output of 32 mL/hr

A

ANS: C
Chest heaviness or pain indicates myocardial ischemia, a possible adverse effect of dopamine. While taking
dopamine, the oxygen requirements of the heart are increased due to increased myocardial workload, and may
cause ischemia. Without knowing the clients previous blood pressure or pedal pulses, there is not enough
information to determine if these are an improvement or not. A urine output of 32 mL/hr is acceptable.

295
Q

The student nurse studying shock understands that the common manifestations of this condition are directly
related to which problems? (Select all that apply.)
a. Anaerobic metabolism
b. Hyperglycemia
c. Hypotension
d. Impaired renal perfusion
e. Increased perfusion

A

ANS: A, C
The common manifestations of shock, no matter the cause, are directly related to the effects of anaerobic
metabolism and hypotension. Hyperglycemia, impaired renal function, and increased perfusion are not
manifestations of shock.

296
Q

The nurse caring for hospitalized clients includes which actions on their care plans to reduce the possibility
of the clients developing shock? (Select all that apply.)
a. Assessing and identifying clients at risk
b. Monitoring the daily white blood cell count
c. Performing proper hand hygiene
d. Removing invasive lines as soon as possible
e. Using aseptic technique during procedures

A

ANS: A, C, D, E
Assessing and identifying clients at risk for shock is probably the most critical action the nurse can take to
prevent shock from occurring. Proper hand hygiene, using aseptic technique, and removing IV lines and
catheters are also important actions to prevent shock. Monitoring laboratory values does not prevent shock but
can indicate a change.

297
Q

he nurse caring frequently for older adults in the hospital is aware of risk factors that place them at a higher
risk for shock. For what factors would the nurse assess? (Select all that apply.)
a. Altered mobility/immobility
b. Decreased thirst response
c. Diminished immune response
d. Malnutrition
e. Overhydration

A

ANS: A, B, C, D
Immobility, decreased thirst response, diminished immune response, and malnutrition can place the older adult
at higher risk of developing shock. Overhydration is not a common risk factor for shock.

298
Q

A client is in the early stages of shock and is restless. What comfort measures does the nurse delegate to the
nursing student? (Select all that apply.)
a. Bringing the client warm blankets
b. Giving the client hot tea to drink
c. Massaging the clients painful legs
d. Reorienting the client as needed
e. Sitting with the client for reassurance

A

ANS: A, D, E
The student can bring the client warm blankets, reorient the client as needed to decrease anxiety, and sit with
the client for reassurance. The client should be NPO at this point, so hot tea is prohibited. Massaging the legs is
not recommended as this can dislodge any clots present, which may lead to pulmonary embolism.

299
Q

. The nurse is caring for a client with suspected severe sepsis. What does the nurse prepare to do within 3
hours of the client being identified as being at risk? (Select all that apply.)
a. Administer antibiotics. b. Draw serum lactate levels. c. Infuse vasopressors. d. Measure central venous pressure. e. Obtain blood cultures.

A

ANS: A, B, E
Within the first 3 hours of suspecting severe sepsis, the nurse should draw (or facilitate) serum lactate levels, obtain blood cultures (or other cultures), and administer antibiotics (after the cultures have been obtained).
Infusing vasopressors and measuring central venous pressure are actions that should occur within the first 6
hours.

300
Q

A client is receiving an infusion of tissue plasminogen activator (t-PA). The nurse assesses the client to be
disoriented to person, place, and time. What action by the nurse is best?
a. Assess the clients pupillary responses. b. Request a neurologic consultation. c. Stop the infusion and call the provider. d. Take and document a full set of vital signs.

A

ANS: C
A change in neurologic status in a client receiving t-PA could indicate intracranial hemorrhage. The nurse
should stop the infusion and notify the provider immediately. A full assessment, including pupillary responses
and vital signs, occurs next. The nurse may or may not need to call a neurologist.

301
Q

A client received tissue plasminogen activator (t-PA) after a myocardial infarction and now is on an
intravenous infusion of heparin. The clients spouse asks why the client needs this medication. What response
by the nurse is best?
a. The t-PA didnt dissolve the entire coronary clot. b. The heparin keeps that artery from getting blocked again. c. Heparin keeps the blood as thin as possible for a longer time. d. The heparin prevents a stroke from occurring as the t-PA wears off.

A

ANS: B
After the original intracoronary clot has dissolved, large amounts of thrombin are released into the
bloodstream, increasing the chance of the vessel reoccluding. The other statements are not accurate. Heparin is
not a blood thinner, although laypeople may refer to it as such.

302
Q

. A client is in the hospital after suffering a myocardial infarction and has bathroom privileges. The nurse
assists the client to the bathroom and notes the clients O2 saturation to be 95%, pulse 88 beats/min, and
respiratory rate 16 breaths/min after returning to bed. What action by the nurse is best?
a. Administer oxygen at 2 L/min. b. Allow continued bathroom privileges. c. Obtain a bedside commode. d. Suggest the client use a bedpan.

A

ANS: B
This clients physiologic parameters did not exceed normal during and after activity, so it is safe for the client to
continue using the bathroom. There is no indication that the client needs oxygen, a commode, or a bedpan.

303
Q

A nursing student is caring for a client who had a myocardial infarction. The student is confused because the
client states nothing is wrong and yet listens attentively while the student provides education on lifestyle
changes and healthy menu choices. What response by the faculty member is best?
a. Continue to educate the client on possible healthy changes. b. Emphasize complications that can occur with noncompliance. c. Tell the client that denial is normal and will soon go away. d. You need to make sure the client understands this illness.

A

ANS: A
Clients are often in denial after a coronary event. The client who seems to be in denial but is compliant with
treatment may be using a healthy form of coping that allows time to process the event and start to use problemfocused coping. The student should not discourage this type of denial and coping, but rather continue providing
education in a positive manner. Emphasizing complications may make the client defensive and more anxious. Telling the client that denial is normal is placing too much attention on the process. Forcing the client to
verbalize understanding of the illness is also potentially threatening to the client.

304
Q

A client undergoing hemodynamic monitoring after a myocardial infarction has a right atrial pressure of 0.5
mm Hg. What action by the nurse is most appropriate?
a. Level the transducer at the phlebostatic axis. b. Lay the client in the supine position. c. Prepare to administer diuretics. d. Prepare to administer a fluid bolus.

A

A client undergoing hemodynamic monitoring after a myocardial infarction has a right atrial pressure of 0.5
mm Hg. What action by the nurse is most appropriate?
a. Level the transducer at the phlebostatic axis. b. Lay the client in the supine position. c. Prepare to administer diuretics. d. Prepare to administer a fluid bolus.

305
Q

A client has hemodynamic monitoring after a myocardial infarction. What safety precaution does the nurse
implement for this client?
a. Document pulmonary artery wedge pressure (PAWP) readings and assess their trends. b. Ensure the balloon does not remain wedged. c. Keep the client on strict NPO status. d. Maintain the client in a semi-Fowlers position.

A

ANS: B
If the balloon remains inflated, it can cause pulmonary infarction or rupture. The nurse should ensure the
balloon remains deflated between PAWP readings. Documenting PAWP readings and assessing trends is an
important nursing action related to hemodynamic monitoring, but is not specifically related to safety. The
client does not have to be NPO while undergoing hemodynamic monitoring. Positioning may or may not affect
readings.

306
Q

A client has intra-arterial blood pressure monitoring after a myocardial infarction. The nurse notes the
clients heart rate has increased from 88 to 110 beats/min, and the blood pressure dropped from 120/82 to
100/60 mm Hg. What action by the nurse is most appropriate?
a. Allow the client to rest quietly. b. Assess the client for bleeding
c. Document the findings in the chart. d. Medicate the client for pain.

A

ANS: B
A major complication related to intra-arterial blood pressure monitoring is hemorrhage from the insertion site. Since these vital signs are out of the normal range, are a change, and are consistent with blood loss, the nurse
should assess the client for any bleeding associated with the arterial line. The nurse should document the
findings after a full assessment. The client may or may not need pain medication and rest; the nurse first needs
to rule out any emergent bleeding.

307
Q

ANS: B
A major complication related to intra-arterial blood pressure monitoring is hemorrhage from the insertion site. Since these vital signs are out of the normal range, are a change, and are consistent with blood loss, the nurse
should assess the client for any bleeding associated with the arterial line. The nurse should document the
findings after a full assessment. The client may or may not need pain medication and rest; the nurse first needs
to rule out any emergent bleeding.

A

ANS: B
A major complication related to intra-arterial blood pressure monitoring is hemorrhage from the insertion site. Since these vital signs are out of the normal range, are a change, and are consistent with blood loss, the nurse
should assess the client for any bleeding associated with the arterial line. The nurse should document the
findings after a full assessment. The client may or may not need pain medication and rest; the nurse first needs
to rule out any emergent bleeding.

308
Q

A client is in the clinic a month after having a myocardial infarction. The client reports sleeping well since
moving into the guest bedroom. What response by the nurse is best?
a. Do you have any concerns about sexuality?
b. Im glad to hear you are sleeping well now. c. Sleep near your spouse in case of emergency. d. Why would you move into the guest roo

A

ANS: A
Concerns about resuming sexual activity are common after cardiac events. The nurse should gently inquire if
this is the issue. While it is good that the client is sleeping well, the nurse should investigate the reason for the
move. The other two responses are likely to cause the client to be defensive.

309
Q

A client in the cardiac stepdown unit reports severe, crushing chest pain accompanied by nausea and

vomiting. What action by the nurse takes priority?
a. Administer an aspirin. b. Call for an electrocardiogram (ECG). c. Maintain airway patency. d. Notify the provider.

A

ANS: C
Airway always is the priority. The other actions are important in this situation as well, but the nurse should stay with the client and ensure the airway remains patent (especially if vomiting occurs) while another person calls
the provider (or Rapid Response Team) and facilitates getting an ECG done. Aspirin will probably be
administered, depending on the providers prescription and the clients current medications.

310
Q

An older adult is on cardiac monitoring after a myocardial infarction. The client shows frequent

dysrhythmias. What action by the nurse is most appropriate?
a. Assess for any hemodynamic effects of the rhythm. b. Prepare to administer antidysrhythmic medication. c. Notify the provider or call the Rapid Response Team. d. Turn the alarms off on the cardiac monitor.

A

ANS: A
Older clients may have dysrhythmias due to age-related changes in the cardiac conduction system. They may
have no significant hemodynamic effects from these changes. The nurse should first assess for the effects of
the dysrhythmia before proceeding further. The alarms on a cardiac monitor should never be shut off. The
other two actions may or may not be needed.

311
Q

The nurse is preparing to change a clients sternal dressing. What action by the nurse is most important?
a. Assess vital signs. b. Don a mask and gown. c. Gather needed supplies. d. Perform hand hygiene.

A

ANS: D
To prevent a sternal wound infection, the nurse washes hands or performs hand hygiene as a priority. Vital
signs do not necessarily need to be assessed beforehand. A mask and gown are not needed. The nurse should
gather needed supplies, but this is not the priority.

312
Q

A client has an intra-arterial blood pressure monitoring line. The nurse notes bright red blood on the clients

sheets. What action should the nurse perform first?
a. Assess the insertion site. b. Change the clients sheets. c. Put on a pair of gloves. d. Assess blood pressure.

A

ANS: C
For the nurses safety, he or she should put on a pair of gloves to prevent blood exposure. The other actions are
appropriate as well, but first the nurse must don a pair of gloves.

313
Q

A nurse is in charge of the coronary intensive care unit. Which client should the nurse see first?

a. Client on a nitroglycerin infusion at 5 mcg/min, not titrated in the last 4 hours
b. Client who is 1 day post coronary artery bypass graft, blood pressure 180/100 mm Hg
c. Client who is 1 day post percutaneous coronary intervention, going home this morning
d. Client who is 2 days post coronary artery bypass graft, became dizzy this a.m. while walking

A

ANS: B
Hypertension after coronary artery bypass graft surgery can be dangerous because it puts too much pressure on
the suture lines and can cause bleeding. The charge nurse should see this client first. The client who became
dizzy earlier should be seen next. The client on the nitroglycerin drip is stable. The client going home can wait
until the other clients are cared for.

314
Q

A client with coronary artery disease (CAD) asks the nurse about taking fish oil supplements. What
response by the nurse is best?
a. Fish oil is contraindicated with most drugs for CAD. b. The best source is fish, but pills have benefits too. c. There is no evidence to support fish oil use with CAD. d. You can reverse CAD totally with diet and supplements.

A

ANS: B
Omega-3 fatty acids have shown benefit in reducing lipid levels, in reducing the incidence of sudden cardiac
death, and for stabilizing atherosclerotic plaque. The best source is fish three times a week or some fish oil
supplements. The other options are not accurate.

315
Q

A client has presented to the emergency department with an acute myocardial infarction (MI). What action
by the nurse is best to meet The Joint Commissions Core Measures outcomes?
a. Obtain an electrocardiogram (ECG) now and in the morning. b. Give the client an aspirin. c. Notify the Rapid Response Team. d. Prepare to administer thrombolytics.

A

ANS: B
The Joint Commissions Core Measures set for acute MI require that aspirin is administered when a client with
MI presents to the emergency department or when an MI occurs in the hospital. A rapid ECG is vital, but
getting another one in the morning is not part of the Core Measures set. The Rapid Response Team is not
needed if an emergency department provider is available. Thrombolytics may or may not be needed.

316
Q

A nurse is caring for four clients. Which client should the nurse assess first?

a. Client with an acute myocardial infarction, pulse 102 beats/min
b. Client who is 1 hour post angioplasty, has tongue swelling and anxiety
c. Client who is post coronary artery bypass, chest tube drained 100 mL/hr
d. Client who is post coronary artery bypass, potassium 4.2 mEq/L

A

ANS: B

The post-angioplasty client with tongue swelling and anxiety is exhibiting manifestations of an allergic
reaction that could progress to anaphylaxis. The nurse should assess this client first. The client with a heart rate
of 102 beats/min may have increased oxygen demands but is just over the normal limit for heart rate. The two
post coronary artery bypass clients are stable.

317
Q

A nurse is caring for a client who is intubated and has an intra-aortic balloon pump. The client is restless
and agitated. What action should the nurse perform first for comfort?
a. Allow family members to remain at the bedside. b. Ask the family if the client would like a fan in the room. c. Keep the television tuned to the clients favorite channel. d. Speak loudly to the client in case of hearing problems.

A

A nurse is caring for a client who is intubated and has an intra-aortic balloon pump. The client is restless
and agitated. What action should the nurse perform first for comfort?
a. Allow family members to remain at the bedside. b. Ask the family if the client would like a fan in the room. c. Keep the television tuned to the clients favorite channel. d. Speak loudly to the client in case of hearing problems.

318
Q

The nurse is caring for a client with a chest tube after a coronary artery bypass graft. The drainage slows

significantly. What action by the nurse is most important?
a. Increase the setting on the suction. b. Notify the provider immediately. c. Re-position the chest tube. d. Take the tubing apart to assess for clots.

A

ANS: B
If the drainage in the chest tube decreases significantly and dramatically, the tube may be blocked by a clot. This could lead to cardiac tamponade. The nurse should notify the provider immediately. The nurse should not
independently increase the suction, re-position the chest tube, or take the tubing apart.

319
Q

. A home health care nurse is visiting an older client who lives alone after being discharged from the hospital
after a coronary artery bypass graft. What finding in the home most causes the nurse to consider additional
referrals?
a. Dirty carpets in need of vacuuming
b. Expired food in the refrigerator
c. Old medications in the kitchen
d. Several cats present in the home

A

ANS: B
Expired food in the refrigerator demonstrates a safety concern for the client and a possible lack of money to
buy food. The nurse can consider a referral to Meals on Wheels or another home-based food program. Dirty
carpets may indicate the client has no household help and is waiting for clearance to vacuum. Old medications
can be managed by the home health care nurse and the client working collaboratively. Having pets is not a
cause for concern.

320
Q

A client is on a dopamine infusion via a peripheral line. What action by the nurse takes priority for safety?
a. Assess the IV site hourly. b. Monitor the pedal pulses. c. Monitor the clients vital signs. d. Obtain consent for a central line

A

ANS: A
Dopamine should be infused through a central line to prevent extravasation and necrosis of tissue. If it needs to
be run peripherally, the nurse assesses the site hourly for problems. When the client is getting the central line, ensuring informed consent is on the chart is a priority. But at this point, the client has only a peripheral line, so
caution must be taken to preserve the integrity of the clients integumentary system. Monitoring pedal pulses
and vital signs give indications as to how well the drug is working.

321
Q

ANS: A
Dopamine should be infused through a central line to prevent extravasation and necrosis of tissue. If it needs to
be run peripherally, the nurse assesses the site hourly for problems. When the client is getting the central line, ensuring informed consent is on the chart is a priority. But at this point, the client has only a peripheral line, so
caution must be taken to preserve the integrity of the clients integumentary system. Monitoring pedal pulses
and vital signs give indications as to how well the drug is working.

A

ANS: C
Poor peripheral pulses and cool skin may be signs of impending cardiogenic shock and should be reported
immediately. A blood pressure drop of 20 mm Hg is not worrisome. An oxygen saturation of 94% is just
slightly below normal. A urine output of 1.2 mL/kg/hr for 4 hours is normal.

322
Q

A client presents to the emergency department with an acute myocardial infarction (MI) at 1500 (3:00 PM). The facility has 24-hour catheterization laboratory abilities. To meet The Joint Commissions Core Measures
set, by what time should the client have a percutaneous coronary intervention performed?
a. 1530 (3:30 PM)
b. 1600 (4:00 PM)
c. 1630 (4:30 PM)
d. 1700 (5:00 PM)

A

ANS: C
The Joint Commissions Core Measures set for MI includes percutaneous coronary intervention within 90
minutes of diagnosis of myocardial infarction. Therefore, the client should have a percutaneous coronary
intervention performed no later than 1630 (4:30 PM).

323
Q

The provider requests the nurse start an infusion of an inotropic agent on a client. How does the nurse explain the action of these drugs to the client and spouse?
a. It constricts vessels, improving blood flow. b. It dilates vessels, which lessens the work of the heart. c. It increases the force of the hearts contractions. d. It slows the heart rate down for better filling.

A

ANS: C
A positive inotrope is a medication that increases the strength of the hearts contractions. The other options are
not correct.

324
Q

. A nurse is assessing a client who had a myocardial infarction. Upon auscultating heart sounds, the nurse
hears the following sound. What action by the nurse is most appropriate?
(Click the media button to hear the audio clip.)
a. Assess for further chest pain. b. Call the Rapid Response Team. c. Have the client sit upright. d. Listen to the clients lung sounds.

A

ANS: D
The sound the nurse hears is an S3 heart sound, an abnormal sound that may indicate heart failure. The nurse
should next assess the clients lung sounds. Assessing for chest pain is not directly related. There is no
indication that the Rapid Response Team is needed. Having the client sit up will not change the heart sound

325
Q

A client had an inferior wall myocardial infarction (MI). The nurse notes the clients cardiac rhythm as
shown below:
What action by the nurse is most important?
a. Assess the clients blood pressure and level of consciousness. b. Call the health care provider or the Rapid Response Team. c. Obtain a permit for an emergency temporary pacemaker insertion. d. Prepare to administer antidysrhythmic medication.

A

ANS: A
Clients with an inferior wall MI often have bradycardia and blocks that lead to decreased perfusion, as seen in
this ECG strip showing sinus bradycardia. The nurse should first assess the clients hemodynamic status,
including vital signs and level of consciousness. The client may or may not need the Rapid Response Team, a
temporary pacemaker, or medication; there is no indication of this in the question.

326
Q
A nursing student learns about modifiable risk factors for coronary artery disease. Which factors does this
include? (Select all that apply.)
a. Age
b. Hypertension
c. Obesity
d. Smoking
e. Stress
A

ANS: B, C, D, E
Hypertension, obesity, smoking, and excessive stress are all modifiable risk factors for coronary artery disease. Age is a nonmodifiable risk factor.

327
Q

ANS: B, C, D, E
Hypertension, obesity, smoking, and excessive stress are all modifiable risk factors for coronary artery disease. Age is a nonmodifiable risk factor.

A

ANS: A, C, E
The nurse can delegate assisting the client to get up in the chair or ambulate to the bathroom, applying TEDs or
sequential compression devices, and taking/recording vital signs. The spirometer should be used every hour the
day after surgery. Assessing pain using a 0-to-10 scale is a nursing assessment, although if the client reports
pain, the UAP should inform the nurse so a more detailed assessment is done.

328
Q

ANS: A, C, E
The nurse can delegate assisting the client to get up in the chair or ambulate to the bathroom, applying TEDs or
sequential compression devices, and taking/recording vital signs. The spirometer should be used every hour the
day after surgery. Assessing pain using a 0-to-10 scale is a nursing assessment, although if the client reports
pain, the UAP should inform the nurse so a more detailed assessment is done.

A

ANS: A, B, D, E
The pain from an MI is often accompanied by shortness of breath and fear or anxiety. It lasts longer than 15
minutes and is not relieved by nitroglycerin. It occurs without a known cause such as exertion.

329
Q

ANS: A, B, D, E
The pain from an MI is often accompanied by shortness of breath and fear or anxiety. It lasts longer than 15
minutes and is not relieved by nitroglycerin. It occurs without a known cause such as exertion.

A

ANS: B, D, E

Nonpharmacologic comfort measures can include positioning, complementary therapies, and splinting the
chest incision. Medications are not nonpharmacologic. Food choices are not comfort measures.

330
Q

A nursing student planning to teach clients about risk factors for coronary artery disease (CAD) would
include which topics? (Select all that apply.)
a. Advanced age
b. Diabetes
c. Ethnic background
d. Medication use
e. Smoking

A

ANS: A, B, C, E

Age, diabetes, ethnic background, and smoking are all risk factors for developing CAD; medication use is not.

331
Q
  1. A nursing student wants to know why clients with chronic obstructive pulmonary disease tend to be
    polycythemic. What response by the nurse instructor is best?
    a. It is due to side effects of medications for bronchodilation. b. It is from overactive bone marrow in response to chronic disease. c. It combats the anemia caused by an increased metabolic rate. d. It compensates for tissue hypoxia caused by lung disea
A

ANS: D
In response to hypoxia, more red blood cells are made so more oxygen can be carried and delivered to tissues. This is a physiologic process in response to the disease; it is not a medication side effect, the result of
overactive bone marrow, or a response to anemia.

332
Q
  1. A client is receiving rivaroxaban (Xarelto) and asks the nurse to explain how it works. What response by the
    nurse is best?
    a. It inhibits thrombin. b. It inhibits fibrinogen. c. It thins your blood. d. It works against vitamin K.
A

ANS: A
Rivaroxaban is a direct thrombin inhibitor. It does not work on fibrinogen or vitamin K. It is not a blood
thinner, although many clients call anticoagulants by this name.

333
Q
The health care provider tells the nurse that a client is to be started on a platelet inhibitor. About what drug
does the nurse plan to teach the client?
a. Clopidogrel (Plavix)
b. Enoxaparin (Lovenox)
c. Reteplase (Retavase)
d. Warfarin (Coumadin)
A

ANS: A
Clopidogrel is a platelet inhibitor. Enoxaparin is an indirect thrombin inhibitor. Reteplase is a fibrinolytic
agent. Warfarin is a vitamin K antagonist.

334
Q

A nurse is assessing a dark-skinned client for pallor. What action is best?
a. Assess the conjunctiva of the eye. b. Have the client open the hand widely. c. Look at the roof of the clients mouth. d. Palpate for areas of mild swelling

A

ANS: A
To assess pallor in dark-skinned people, assess the conjunctiva of the eye or the mucous membranes. Looking
at the roof of the mouth can reveal jaundice. Opening the hand widely is not related to pallor, nor is palpating
for mild swelling.

335
Q

A hospitalized client has a platelet count of 58,000/mm3. What action by the nurse is best?
a. Encourage high-protein foods. b. Institute neutropenic precautions. c. Limit visitors to healthy adults. d. Place the client on safety precautions.

A

ANS: D
With a platelet count between 40,000 and 80,000/mm3, clients are at risk of prolonged bleeding even after
minor trauma. The nurse should place the client on safety precautions. High-protein foods, while healthy, are
not the priority. Neutropenic precautions are not needed as the clients white blood cell count is not low. Limiting visitors would also be more likely related to a low white blood cell count.

336
Q

A client is having a bone marrow biopsy today. What action by the nurse takes priority?
a. Administer pain medication first. b. Ensure valid consent is on the chart. c. Have the client shower in the morning. d. Premedicate the client with sedatives.

A

ANS: B
A bone marrow biopsy is an invasive procedure that requires informed consent. Pain medication and sedation
are important components of care for this client but do not take priority. The client may or may not need or be
able to shower

337
Q

A nurse is caring for four clients. After reviewing todays laboratory results, which client should the nurse
see first?
a. Client with an international normalized ratio of 2.8
b. Client with a platelet count of 128,000/mm3
c. Client with a prothrombin time (PT) of 28 seconds
d. Client with a red blood cell count of 5.1 million/L

A

A normal PT is 11 to 12.5 seconds. This client is at high risk of bleeding. The other values are within normal
limits.

338
Q

A client is having a bone marrow biopsy and is extremely anxious. What action by the nurse is best?
a. Assess client fears and coping mechanisms. b. Reassure the client this is a common test. c. Sedate the client prior to the procedure. d. Tell the client he or she will be asleep

A

ANS: A
Assessing the clients specific fears and coping mechanisms helps guide the nurse in providing holistic care that
best meets the clients needs. Reassurance will be helpful but is not the best option. Sedation is usually used. The client may or may not be totally asleep during the procedure.

339
Q

A client is having a radioisotopic imaging scan. What action by the nurse is most important?
a. Assess the client for shellfish allergies. b. Place the client on radiation precautions. c. Sedate the client before the scan. d. Teach the client about the procedure.

A

ANS: D
The nurse should ensure that teaching is done and the client understands the procedure. Contrast dye is not
used, so shellfish/iodine allergies are not related. The client will not be radioactive and does not need radiation
precautions. Sedation is not used in this procedure.

340
Q

A student nurse learns that the spleen has several functions. What functions do they include? (Select all that

apply. )
a. Breaks down hemoglobin
b. Destroys old or defective red blood cells (RBCs)
c. Forms vitamin K for clotting
d. Stores extra iron in ferritin
e. Stores platelets not circulating

A

ANS: A, B, E
Functions of the spleen include breaking down hemoglobin released from RBCs, destroying old or defective
RBCs, and storing the platelets that are not in circulation. Forming vitamin K for clotting and storing extra iron
in ferritin are functions of the liver.

341
Q

An older client asks the nurse why people my age have weaker immune systems than younger people. What
responses by the nurse are best? (Select all that apply.)
a. Bone marrow produces fewer blood cells. b. You may have decreased levels of circulating platelets. c. You have lower levels of plasma proteins in the blood. d. Lymphocytes become more reactive to antigens. e. Spleen function declines after age 60.

A

ANS: A, C
The aging adult has bone marrow that produces fewer cells and decreased blood volume with fewer plasma
proteins. Platelet numbers remain unchanged, lymphocytes become less reactive, and spleen function stays the
same.

342
Q
. A nursing student learns that many drugs can impair the immune system. Which drugs does this include?
(Select all that apply.)
a. Acetaminophen (Tylenol)
b. Amphotericin B (Fungizone)
c. Ibuprofen (Motrin)
d. Metformin (Glucophage)
e. Nitrofurantoin (Macrobid)
A

ANS: B, C, E
Amphotericin B, ibuprofen, and nitrofurantoin all can disrupt the hematologic (immune) system. Acetaminophen and metformin do not.

343
Q

A nurse works in a gerontology clinic. What age-related changes cause the nurse to alter standard
assessment techniques from those used for younger adults? (Select all that apply.)
a. Dentition deteriorates with more cavities. b. Nail beds may be thickened or discolored. c. Progressive loss of hair occurs with age. d. Sclerae begin to turn yellow or pale. e. Skin becomes dry as the client ages

A

ANS: B, C, E
Common findings in older adults include thickened or discolored nail beds, dry skin, and thinning hair. The
nurse adapts to these changes by altering assessment techniques. Having more dental caries and changes in the
sclerae are not normal age-related changes.

344
Q
A nurse caring for a client with sickle cell disease (SCD) reviews the clients laboratory work. Which finding
should the nurse report to the provider?
a. Creatinine: 2.9 mg/dL
b. Hematocrit: 30%
c. Sodium: 147 mEq/L
d. White blood cell count: 12,000/mm3
A

ANS: A
An elevated creatinine indicates kidney damage, which occurs in SCD. A hematocrit level of 30% is an
expected finding, as is a slightly elevated white blood cell count. A sodium level of 147 mEq/L, although
slightly high, is not concerning.

345
Q

A client hospitalized with sickle cell crisis frequently asks for opioid pain medications, often shortly after
receiving a dose. The nurses on the unit believe the client is drug seeking. When the client requests pain
medication, what action by the nurse is best?
a. Give the client pain medication if it is time for another dose. b. Instruct the client not to request pain medication too early. c. Request the provider leave a prescription for a placebo. d. Tell the client it is too early to have more pain medication.

A

ANS: A
Clients with sickle cell crisis often have severe pain that is managed with up to 48 hours of IV opioid
analgesics. Even if the client is addicted and drug seeking, he or she is still in extreme pain. If the client can
receive another dose of medication, the nurse should pro

346
Q

client in sickle cell crisis is dehydrated and in the emergency department. The nurse plans to start an IV. Which fluid choice is best?

a. 0.45% normal saline
b. 0.9% normal saline
c. Dextrose 50% (D50)
d. Lactated Ringers solution

A

ANS: A
Because clients in sickle cell crisis are often dehydrated, the fluid of choice is a hypotonic solution such as
0.45% normal saline. 0.9% normal saline and lactated Ringers solution are isotonic. D50 is hypertonic and not
used for hydration

347
Q

A client presents to the emergency department in sickle cell crisis. What intervention by the nurse takes
priority?

a. Administer oxygen. b. Apply an oximetry probe. c. Give pain medication. d. Start an IV

A

ANS: A
All actions are appropriate, but remembering the ABCs, oxygen would come first. The main problem in a
sickle cell crisis is tissue and organ hypoxia, so providing oxygen helps halt the process.

348
Q

A client has a serum ferritin level of 8 ng/mL and microcytic red blood cells. What action by the nurse is
best?
a. Encourage high-protein foods. b. Perform a Hemoccult test on the clients stools. c. Offer frequent oral care. d. Prepare to administer cobalamin (vitamin B12)

A

ANS: B
This client has laboratory findings indicative of iron deficiency anemia. The most common cause of this
disorder is blood loss, often from the GI tract. The nurse should perform a Hemoccult test on the clients stools. High-protein foods may help the condition, but dietary interventions take time to work. That still does not
determine the cause. Frequent oral care is not related. Cobalamin injections are for pernicious anemia.

349
Q

A client has Crohns disease. What type of anemia is this client most at risk for developing?

a. Folic acid deficiency
b. Fanconis anemia
c. Hemolytic anemia
d. Vitamin B12 anemia

A

ANS: A
Malabsorption syndromes such as Crohns disease leave a client prone to folic acid deficiency. Fanconis
anemia, hemolytic anemia, and vitamin B12 anemia are not related to Crohns disease.

350
Q

A nurse in a hematology clinic is working with four clients who have polycythemia vera. Which client
should the nurse see first?
a. Client with a blood pressure of 180/98 mm Hg
b. Client who reports shortness of breath
c. Client who reports calf tenderness and swelling
d. Client with a swollen and painful left great toe

A

ANS: B
Clients with polycythemia vera often have clotting abnormalities due to the hyperviscous blood with sluggish
flow. The client reporting shortness of breath may have a pulmonary embolism and should be seen first. The
client with a swollen calf may have a deep vein thrombosis and should be seen next. High blood pressure and
gout symptoms are common findings with this disorder

351
Q

A nursing student is caring for a client with leukemia. The student asks why the client is still at risk for
infection when the clients white blood cell count (WBC) is high. What response by the registered nurse is best?
a. If the WBCs are high, there already is an infection present. b. The client is in a blast crisis and has too many WBCs. c. There must be a mistake; the WBCs should be very low. d. Those WBCs are abnormal and dont provide protection.

A

ANS: D
In leukemia, the WBCs are abnormal and do not provide protection to the client against infection. The other
statements are not accurate.

352
Q

The family of a neutropenic client reports the client is not acting right. What action by the nurse is the
priority?
a. Ask the client about pain. b. Assess the client for infection. c. Delegate taking a set of vital signs. d. Look at todays laboratory results.

A

ANS: B
Neutropenic clients often do not have classic manifestations of infection, but infection is the most common
cause of death in neutropenic clients. The nurse should assess for infection. The nurse should assess for pain
but this is not the priority. The nurse should take the clients vital signs instead of delegating them since the
client has had a change in status. Laboratory results may be inconclusive.

353
Q

A nurse is caring for a client who is about to receive a bone marrow transplant. To best help the client cope
with the long recovery period, what action by the nurse is best?
a. Arrange a visitation schedule among friends and family. b. Explain that this process is difficult but must be endured. c. Help the client find things to hope for each day of recovery. d. Provide plenty of diversionary activities for this time.

A

ANS: C
Providing hope is an essential nursing function during treatment for any disease process, but especially during
the recovery period after bone marrow transplantation, which can take up to 3 weeks. The nurse can help the
client look ahead to the recovery period and identify things to hope for during this time. Visitors are important
to clients, but may pose an infection risk. Telling the client the recovery period must be endured does not
acknowledge his or her feelings. Diversionary activities are important, but not as important as instilling hope.

354
Q

A nursing student is struggling to understand the process of graft-versus-host disease. What explanation by the nurse instructor is best?
a. Because of immunosuppression, the donor cells take over. b. Its like a transfusion reaction because no perfect matches exist. c. The clients cells are fighting donor cells for dominance. d. The donors cells are actually attacking the clients cells.

A

the nurse instructor is best?
a. Because of immunosuppression, the donor cells take over. b. Its like a transfusion reaction because no perfect matches exist. c. The clients cells are fighting donor cells for dominance. d. The donors cells are actually attacking the clients cells.

355
Q

the nurse instructor is best?
a. Because of immunosuppression, the donor cells take over. b. Its like a transfusion reaction because no perfect matches exist. c. The clients cells are fighting donor cells for dominance. d. The donors cells are actually attacking the clients cells.

A

ANS: A
Fatigue is a common problem for clients with leukemia. This client is managing his or her own ADLs using
rest periods, which indicates an understanding of fatigue and how to control it. Helping to plan an activity
schedule is a lesser indicator. Requesting a sleeping pill does not help control fatigue during the day. Asking
visitors to leave when tired is another lesser indicator. Managing ADLs using rest periods demonstrates the
most comprehensive management strategy

356
Q

ANS: A
Fatigue is a common problem for clients with leukemia. This client is managing his or her own ADLs using
rest periods, which indicates an understanding of fatigue and how to control it. Helping to plan an activity
schedule is a lesser indicator. Requesting a sleeping pill does not help control fatigue during the day. Asking
visitors to leave when tired is another lesser indicator. Managing ADLs using rest periods demonstrates the
most comprehensive management strategy

A

NS: C
All teaching topics are important to the client with lymphoma, but for a young male, sperm banking is of
particular concern if the client is going to have radiation to the lower abdomen or pelvis.

357
Q

A client has been admitted after sustaining a humerus fracture that occurred when picking up the family

cat. What test result would the nurse correlate to this condition?
a. Bence-Jones protein in urine
b. Epstein-Barr virus: positive
c. Hemoglobin: 18 mg/dL
d. Red blood cell count: 8.2/mm3

A

ANS: A
This client has possible multiple myeloma. A positive Bence-Jones protein finding would correlate with this condition. The Epstein-Barr virus is a herpesvirus that causes infectious mononucleosis and some cancers. A
hemoglobin of 18 mg/dL is slightly high for a male and somewhat high for a female; this can be caused by
several conditions, and further information would be needed to correlate this value with a specific medical
condition. A red blood cell count of 8.2/mm3 is also high, but again, more information would be needed to
correlate this finding with a specific medical condition.

358
Q

condition. The Epstein-Barr virus is a herpesvirus that causes infectious mononucleosis and some cancers. A
hemoglobin of 18 mg/dL is slightly high for a male and somewhat high for a female; this can be caused by
several conditions, and further information would be needed to correlate this value with a specific medical
condition. A red blood cell count of 8.2/mm3 is also high, but again, more information would be needed to
correlate this finding with a specific medical condition.

A

ANS: D
All the options are drugs used to treat multiple myeloma, but the drug used specifically for bone manifestations
is zoledronic acid (Zometa), which is a bisphosphonate. This drug class inhibits bone resorption and is used to
treat osteoporosis as well.

359
Q

A client with autoimmune idiopathic thrombocytopenic purpura (ITP) has had a splenectomy and returned
to the surgical unit 2 hours ago. The nurse assesses the client and finds the abdominal dressing saturated with
blood. What action is most important?
a. Preparing to administer a blood transfusion
b. Reinforcing the dressing and documenting findings
c. Removing the dressing and assessing the surgical site
d. Taking a set of vital signs and notifying the surgeon

A

A client with autoimmune idiopathic thrombocytopenic purpura (ITP) has had a splenectomy and returned
to the surgical unit 2 hours ago. The nurse assesses the client and finds the abdominal dressing saturated with
blood. What action is most important?
a. Preparing to administer a blood transfusion
b. Reinforcing the dressing and documenting findings
c. Removing the dressing and assessing the surgical site
d. Taking a set of vital signs and notifying the surgeon

360
Q

A client has a platelet count of 9000/mm3. The nurse finds the client confused and mumbling. What action
takes priority?
a. Calling the Rapid Response Team
b. Delegating taking a set of vital signs
c. Instituting bleeding precautions
d. Placing the client on bedrest

A

ANS: A
With a platelet count this low, the client is at high risk of spontaneous bleeding. The most disastrous
complication would be intracranial bleeding. The nurse needs to call the Rapid Response Team as this client
has manifestations of a sudden neurologic change. The nurse should not delegate the vital signs as the client is no longer stable. Bleeding precautions will not address the immediate situation. Placing the client on bedrest or
putting the client back into bed is important, but the critical action is to call for immediate medical attention.

361
Q

A nurse is preparing to administer a blood transfusion. What action is most important?

a. Correctly identifying client using two identifiers
b. Ensuring informed consent is obtained if required
c. Hanging the blood product with Ringers lactate
d. Staying with the client for the entire transfusion

A

ANS: B
If the facility requires informed consent for transfusions, this action is most important because it precedes the
other actions taken during the transfusion. Correctly identifying the client and blood product is a National
Patient Safety Goal, and is the most important action after obtaining informed consent. Ringers lactate is not
used to transfuse blood. The nurse does not need to stay with the client for the duration of the transfusion.

362
Q

A nurse is preparing to hang a blood transfusion. Which action is most important?

a. Documenting the transfusion
b. Placing the client on NPO status
c. Placing the client in isolation
d. Putting on a pair of gloves

A

ANS: D
To prevent bloodborne illness, the nurse should don a pair of gloves prior to hanging the blood. Documentation
is important but not the priority at this point. NPO status and isolation are not needed.

363
Q

A client receiving a blood transfusion develops anxiety and low back pain. After stopping the transfusion, what action by the nurse is most important?

a. Documenting the events in the clients medical record
b. Double-checking the client and blood product identification
c. Placing the client on strict bedrest until the pain subsides
d. Reviewing the clients medical record for known allergies

A

ANS: B
This client had a hemolytic transfusion reaction, most commonly caused by blood type or Rh incompatibility. The nurse should double-check all identifying information for both the client and blood type. Documentation
occurs after the client is stable. Bedrest may or may not be needed. Allergies to medications or environmental
items is not related.

364
Q

A client has thrombocytopenia. What client statement indicates the client understands self-management of his condition?
a. I brush and use dental floss every day. b. I chew hard candy for my dry mouth. c. I usually put ice on bumps or bruises. d. Nonslip socks are best when I walk.

A

ANS: C
The client should be taught to apply ice to areas of minor trauma. Flossing is not recommended. Hard foods
should be avoided. The client should wear well-fitting shoes when ambulating.

365
Q

A client has a sickle cell crisis with extreme lower extremity pain. What comfort measure does the nurse
delegate to the unlicensed assistive personnel (UAP)?
a. Apply ice packs to the clients legs. b. Elevate the clients legs on pillows. c. Keep the lower extremities warm. d. Place elastic bandage wraps on the clients legs

A

A client has a sickle cell crisis with extreme lower extremity pain. What comfort measure does the nurse
delegate to the unlicensed assistive personnel (UAP)?
a. Apply ice packs to the clients legs. b. Elevate the clients legs on pillows. c. Keep the lower extremities warm. d. Place elastic bandage wraps on the clients legs

366
Q

. A client admitted for sickle cell crisis is distraught after learning her child also has the disease. What
response by the nurse is best?
a. Both you and the father are equally responsible for passing it on. b. I can see you are upset. I can stay here with you a while if you like. c. Its not your fault; there is no way to know who will have this disease. d. There are many good treatments for sickle cell disease these days.

A

ANS: B
The best response is for the nurse to offer self, a therapeutic communication technique that uses presence. Attempting to assign blame to both parents will not help the client feel better. There is genetic testing
available, so it is inaccurate to state there is no way to know who will have the disease. Stating that good
treatments exist belittles the clients feelings.

367
Q

A cell disease (SCD) takes hydroxyurea (Droxia). The client presents to the clinic
reporting an increase in fatigue. What laboratory result should the nurse report immediately?
a. Hematocrit: 25%
b. Hemoglobin: 9.2 mg/dL
c. Potassium: 3.2 mEq/L
d. White blood cell count: 38,000/mm3

A

ANS D

Although individuals with SCD often have elevated white blood cell (WBC) counts, this extreme elevation
could indicate leukemia, a complication of taking hydroxyurea. The nurse should report this finding
immediately. Alternatively, it could indicate infection, a serious problem for clients with SCD. Hematocrit and
hemoglobin levels are normally low in people with SCD. The potassium level, while slightly low, is not as
worrisome as the WBCs.

368
Q

Although individuals with SCD often have elevated white blood cell (WBC) counts, this extreme elevation
could indicate leukemia, a complication of taking hydroxyurea. The nurse should report this finding
immediately. Alternatively, it could indicate infection, a serious problem for clients with SCD. Hematocrit and
hemoglobin levels are normally low in people with SCD. The potassium level, while slightly low, is not as
worrisome as the WBCs.

A

ANS: A
The client who had two bloody diarrhea stools that morning may be hemorrhaging in the gastrointestinal (GI)
tract and should be assessed first. The client with the change in respiratory rate may have an infection or
worsening anemia and should be seen next. The other two clients are not a priority at this time.

369
Q

A client has frequent hospitalizations for leukemia and is worried about functioning as a parent to four
small children. What action by the nurse would be most helpful?
a. Assist the client to make sick day plans for household responsibilities. b. Determine if there are family members or friends who can help the client. c. Help the client inform friends and family that they will have to help out. d. Refer the client to a social worker in order to investigate respite child care

A

ANS: A
While all options are reasonable choices, the best option is to help the client make sick day plans, as that is
more comprehensive and inclusive than the other options, which focus on a single item.

370
Q

A client has been treated for a deep vein thrombus and today presents to the clinic with petechiae. Laboratory results show a platelet count of 42,000/mm3. The nurse reviews the clients medication list to
determine if the client is taking which drug?
a. Enoxaparin (Lovenox)
b. Salicylates (aspirin)
c. Unfractionated heparin
d. Warfarin (Coumadin)

A

ANS: C
This client has manifestations of heparin-induced thrombocytopenia. Enoxaparin, salicylates, and warfarin do
not cause this condition.

371
Q

The nurse assesses a clients oral cavity and makes the discovery shown in the photo below:
What action by the nurse is most appropriate?
a. Encourage the client to have genetic testing. b. Instruct the client on high-fiber foods. c. Place the client in protective precautions. d. Teach the client about cobalamin therapy.

A

ANS: D
This condition is known as glossitis, and is characteristic of B12 anemia. If the anemia is a pernicious anemia,
it is treated with cobalamin. Genetic testing is not a priority for this condition. The client does not need highfiber foods or protective precautions.

372
Q

A nurse working with clients with sickle cell disease (SCD) teaches about self-management to prevent
exacerbations and sickle cell crises. What factors should clients be taught to avoid? (Select all that apply.)
a. Dehydration
b. Exercise
c. Extreme stress
d. High altitudes
e. Pregnancy

A

ANS: A, C, D, E
Several factors cause red blood cells to sickle in SCD, including dehydration, extreme stress, high altitudes, and pregnancy. Strenuous exercise can also cause sickling, but not unless it is very vigoro

373
Q

ANS: A, C, D, E
Several factors cause red blood cells to sickle in SCD, including dehydration, extreme stress, high altitudes, and pregnancy. Strenuous exercise can also cause sickling, but not unless it is very vigoro

A

NS: A, C, E
Chemical and ionizing radiation exposure and viral infections are known risk factors for developing leukemia. Eating genetically modified food and receiving vaccinations are not known risk factors.

374
Q
A client has Hodgkins lymphoma, Ann Arbor stage Ib. For what manifestations should the nurse assess the
client? (Select all that apply.)
a. Headaches
b. Night sweats
c. Persistent fever
d. Urinary frequency
e. Weight loss
A

ANS: B, C, E
In this stage, the disease is located in a single lymph node region or a single nonlymph node site. The client
displays night sweats, persistent fever, and weight loss. Headache and urinary problems are not related.

375
Q

A client has a platelet count of 25,000/mm3. What actions does the nurse delegate to the unlicensed assistive
personnel (UAP)? (Select all that apply.)
a. Assist with oral hygiene using a firm toothbrush. b. Give the client an enema if he or she is constipated. c. Help the client choose soft foods from the menu. d. Shave the male client with an electric razor. e. Use a lift sheet when needed to re-position the client.

A

ANS: C, D, E
This client has thrombocytopenia and requires bleeding precautions. These include oral hygiene with a soft- bristled toothbrush or swabs, avoiding rectal trauma, eating soft foods, shaving with an electric razor, and
using a lift sheet to re-position the client

376
Q

A student nurse is helping a registered nurse with a blood transfusion. Which actions by the student are most
appropriate? (Select all that apply.)
a. Hanging the blood product using normal saline and a filtered tubing set
b. Taking a full set of vital signs prior to starting the blood transfusion
c. Telling the client someone will remain at the bedside for the first 5 minutes
d. Using gloves to start the clients IV if needed and to handle the blood product
e. Verifying the clients identity, and checking blood compatibility and expiration time

A

A student nurse is helping a registered nurse with a blood transfusion. Which actions by the student are most
appropriate? (Select all that apply.)
a. Hanging the blood product using normal saline and a filtered tubing set
b. Taking a full set of vital signs prior to starting the blood transfusion
c. Telling the client someone will remain at the bedside for the first 5 minutes
d. Using gloves to start the clients IV if needed and to handle the blood product
e. Verifying the clients identity, and checking blood compatibility and expiration time

377
Q

A student nurse is learning about blood transfusion compatibilities. What information does this include?
(Select all that apply.)
a. Donor blood type A can donate to recipient blood type AB. b. Donor blood type B can donate to recipient blood type O. c. Donor blood type AB can donate to anyone. d. Donor blood type O can donate to anyone. e. Donor blood type A can donate to recipient blood type

A

ANS: a, d
Blood type A can be donated to people who have blood types A or AB. Blood type O can be given to anyone. Blood type B can be donated to people who have blood types B or AB. Blood type AB can only go to
recipients with blood type AB.

378
Q

Blood type A can be donated to people who have blood types A or AB. Blood type O can be given to anyone. Blood type B can be donated to people who have blood types B or AB. Blood type AB can only go to
recipients with blood type AB.

A

ANS: B, D
Darbepoetin alfa and epoetin alfa are both red blood cell colony-stimulating factors that will help increase the
production of red blood cells. Azacitidine and decitabine are used for myelodysplastic syndromes. Methylprednisolone is a steroid and would not be used for this prob

379
Q

A nurse is preparing to administer a blood transfusion to an older adult. Understanding age-related changes, what alterations in the usual protocol are necessary for the nurse to implement? (Select all that apply.)
a. Assess vital signs more often. b. Hold other IV fluids running. c. Premedicate to prevent reactions. d. Transfuse smaller bags of blood. e. Transfuse each unit over 8 hours.

A

ANS: A, B
The older adult needs vital signs monitored as often as every 15 minutes for the duration of the transfusion
because changes may be the only indication of a transfusion-related problem. To prevent fluid overload, the
nurse obtains a prescription to hold other running IV fluids during the transfusion. The other options are not
warranted.

380
Q

A client has heparin-induced thrombocytopenia (HIT). The student nurse asks how this is treated. About
what drugs does the nurse instructor teach? (Select all that apply.)
a. Argatroban (Argatroban)
b. Bivalirudin (Angiomax)
c. Clopidogrel (Plavix)
d. Lepirudin (Refludan)
e. Methylprednisolone (Solu-Medrol)

A

ANS: A, B, D
The standard drugs used to treat HIT are argatroban, bivalirudin, and lepirudin. The other drugs are not used. Clopidogrel is an antiplatelet agent used to reduce the likelihood of stroke or myocardial infarction. Methylprednisolone is a steroid used to reduce inflammation.

381
Q

A client has received a bone marrow transplant and is waiting for engraftment. What actions by the nurse
are most appropriate? (Select all that apply.)
a. Not allowing any visitors until engraftment
b. Limiting the protein in the clients diet
c. Placing the client in protective precautions
d. Teaching visitors appropriate hand hygiene
e. Telling visitors not to bring live flowers or plants

A

ANS: C, D, E
The client waiting for engraftment after bone marrow transplant has no white cells to protect him or her against
infection. The client is on protective precautions and visitors are taught hand hygiene. No fresh flowers or
plants are allowed due to the standing water in the vase or container that may harbor organisms. Limiting
protein is not a healthy option and will not promote engraftment.